You are on page 1of 342

Second Edition

(2022)

SurgiNotes

EGYPTIAN BOARD OF GENERAL SURGERY


[FIRST PART PREVIOUS MCQS EXAMS 2010-2022]

ELEMAM
‫بسم هللا الرحمن الرحيم‬
‫الملف ده فيه امتحانات خاصه بالجزء األول للزمالة المرصية يف الجراحة العامة و تخصصاتها‪.‬‬ ‫­‬
‫تم تجميعه بفضل هللا من كذا مصدر (روابط عىل تليجرام و جوجل درايف و ميديا فاير) جزا هللا‬ ‫­‬
‫خي الجزاء‪.‬‬ ‫قائمي عىل رفع الملفات دي ر‬
‫ر‬ ‫ال‬
‫الملف ده عباره عنه ‪ 13‬امتحان ‪( MCQ‬وىه كل االمتحانات المتاحة من ‪ 2010‬ل ‪:)2022‬‬ ‫•‬
‫‪1. 2010 January - Paper (1) MCQ.‬‬
‫‪2. 2013 June - Paper (1) MCQ.‬‬
‫‪3. 2014 March - Paper (1) MCQ.‬‬
‫‪4. 2015 March - Paper (1) MCQ.‬‬
‫‪5. 2017 February - Paper (1) MCQ.‬‬
‫‪6. 2018 February - Paper (1) MCQ.‬‬
‫‪7. 2018 July - Paper (1) MCQ.‬‬
‫‪8. 2019 July - Paper (1) MCQ.‬‬
‫‪9. 2020 October - Paper (1) MCQ.‬‬
‫‪10.2020 October - Paper (2) MCQ.‬‬
‫‪11.2021 March - Paper (2) MCQ.‬‬
‫‪12.2021 September - Paper (1) MCQ.‬‬
‫‪13.2022 March - Paper (1) MCQ.‬‬
‫اإلجابات موجوده مجمعه يف اخركل امتحان يف صوره ‪( Answer sheet‬علشان كان فيه بعض الناس‬ ‫•‬
‫اقيحت "تعديل عىل النسخة اىل فاتت" ان كل سؤال يكون مش متجاوب تحته علشان يعطوا مساحه‬ ‫ر‬
‫للتفكي يف اإلجابات)‪.‬‬
‫ر‬ ‫لنفسهم‬
‫امتحاني‬
‫ر‬ ‫األصىل اىل ناقل منه االمتحانات وده يف‬
‫ي‬ ‫فيه بعض األسئلة لألسف مش موجوده يف الورق‬ ‫•‬
‫بس الحمد هلل‪:‬‬
‫‪1. 2021 September - Paper (1) MCQ:‬‬
‫‪a. 13 – 14 – 15 – 16 – 17.‬‬
‫‪2. 2018 July - Paper (1) MCQ:‬‬
‫‪a. 1 – 2 – 3 – 4.‬‬
‫‪b. 9 – 10 – 11 – 12 – 13.‬‬
‫‪c. 40 – 41 – 42 – 43 – 44.‬‬
‫‪d. 90.‬‬
‫‪e. 93 – 94 – 95 – 96 – 97.‬‬
‫غي‬‫يف النهاية انا جمعت االمتحانات واألسئلة دي بغرض ان احنا نساعد بعض ومش محتاج منكم ر‬ ‫­‬
‫دعوه بظهر الغيب ولو حد ليه أي تعديل عىل أي حاجه او أي حد قدر يوصل ل األسئلة او الكلمات‬
‫اىل ناقصه ما رييددش انه يقول عليها ‪......‬‬

‫بالتوفيق لينا كلنا يا شباب ‪......‬‬


SurgiNotes 2nd Ed. (2022)

Egyptian Board Of General Surgery


1st Part Exam – Paper (1) MCQ
January 2010
1. Which of the following muscles is NOT supplied by the femoral nerve?
a. Sartorius.
b. Rectus femoris.
c. Gracilis.
d. Pectineus.
e. Vastus lateralis.

2. Which of the following is NOT a branch of the femoral artery?


a. Deep external pudendal artery.
b. Superficial circumflex-femoral artery.
c. Deep circumflex femoral artery.
d. Superficial epigastric artery.
e. Superficial external pudendal artery.

3. Which of the following is NOT a branch of the axillary artery?


a. Lateral thoracic artery.
b. Thoraco-acromial artery.
c. Suprascapular artery.
d. Anterior circumflex humeral artery.
e. Subscapular artery.

4. Which of the following is NOT true about the adductor canal?


a. It starts at the apex of the femoral triangle.
b. The roof is formed by the deep fascia.
c. It contains the femoral artery, vein and nerve.
d. It terminates at the adductor hiatus.
e. The floor is foined by the adductor longus and magnus.
P a g e 1 | 26
SurgiNotes 2nd Ed. (2022)

5. The venous drainage of the caudate lobe of the liver is to:


a. The left hepatic vein.
b. The portal vein.
c. The right hepatic vein.
d. The inferior vena cava.
e. The middle hepatic vein.

6. Which of the following is NOT present in ulnar nerve injury at the wrist?
a. Atrophy of the interossei.
b. Inability to adduct the thumb.
c. Loss of sensation over the medial 1/3 of the dorsum of the hand.
d. Partial claw hand.
e. Inability to abduct the fingers.

7. Which of the following is NOT present in sciatic nerve injury?


a. Loss of sensation of the sole of the foot.
b. Inability to dorsiflex the ankle.
c. Loss of sensation of the medial side of the foot.
d. Inability to abduct the foot.
e. Trophic ulcers of the sole of the foot.

8. Which of the following is wrong about the scalenus anterior muscle?


a. It arises from the anterior tubercles of C3-C7 vertebrae.
b. It is inserted into the scalene tubercle of the first rib.
c. It is crossed by the vagus nerve.
d. The subclavian vein passes in front of the muscle.
e. The subclavian artery passes posterior to the muscle.
P a g e 2 | 26
SurgiNotes 2nd Ed. (2022)

9. Which of the following is NOT true about latissimus dorsi muscle?


a. It is inserted into the floor of the bicipital groove of the humerus.
b. The nerve supply is derived from the posterior cord of the brachial plexus.
c. The arterial supply is derived from the second part of the axillary artery.
d. It is an adductor and medial rotator of the upper limb.
e. It takes origin from the lumbar fascia.

10.All the following regarding the femoral canal are correct EXPECT:
a. The femoral sheath invests the femoral vessels and the femoral branch of
the genitofemoral nerve.
b. The femoral nerve courses down lateral to the femoral artery outside the
femoral sheath.
c. The femoral hernia descends down within the femoral sheath.
d. The femoral canal is a fatty cushion between the femoral artery and vein.
e. The pectineus muscle lies behind the femoral sheath.

11.Which of the following muscles is NOT supplied by the obturator nerve?


a. Gracilis.
b. Adductor longus.
c. Adductor brevis.
d. Pectineus.
e. Adductor magnus.

12.Which of the following is NOT present in femoral nerve injury?


a. Loss of sensations over the medial side of the foot.
b. Loss of the knee jerk reflex.
c. Loss of the cremasteric reflex.
d. Loss of sensations over the anterior aspect of the thigh.
e. Atrophy of the sartorius muscle.
P a g e 3 | 26
SurgiNotes 2nd Ed. (2022)

13.Which of the following is NOT supplied by the trigeminal nerve?


a. The muscles of mastication.
b. The buccinator muscle.
c. The temporomandibular joint.
d. The conjunctiva.
e. The mucous membrane of the maxillary air sinus.

14.Which of the following nerves is NOT derived from the cervical plexus?
a. Auriculotemporal nerve.
b. Great auricular nerve.
c. Transverse cervical nerve.
d. Supraclavicular nerves.
e. Phrenic nerve.

15.Which of the following is false regarding the flexor retinaculum at the wrist?
a. Attached to the scaphoid and trapezium laterally and to the pisiform and
hamate medially.
b. Gives attachments to the thenar and hypothenar eminences muscles.
c. The median nerve enters the palm beneath its midpoint.
d. Ulnar nerve and artery run deep to it medially.
e. Gives an insertion to the palmaris longus muscle.

16.Regarding the inguinal canal all the following statements are true EXCEPT:
a. The internal ring lies midway between the symphysis pubis and anterior
superior iliac spine.
b. The internal ring lies lateral to the inferior epigastric vessels.
c. The external oblique aponeurosis forms the anterior boundary.
d. The inguinal ligament forms the inferior boundary.
e. The conjoint tendon forms the lateral part of the posterior wall.
P a g e 4 | 26
SurgiNotes 2nd Ed. (2022)

17.Taste from the posterior one third of tongue is provided by which of the
following nerves?
a. Glossopharyngeal.
b. Facial.
c. Lingual.
d. Vagus.
e. Hypoglossal.

18.All the following are branches of the internal iliac artery EXCEPT:
a. Superior gluteal artery.
b. Inferior vesical artery.
c. Superior rectal artery.
d. Inferior rectal artery.
e. Uterine artery.

19.Which of the following structures does NOT pass through the aortic opening
of the diaphragm?
a. Azygos vein.
b. Thoracic duct.
c. The right phrenic nerve.
d. The aorta.
e. Lymph vessels from the thorax.

20.Which of the following nerves is MOST liable to be injured in fracture of the


surgical neck of the humerus?
a. Median nerve.
b. Ulnar.
c. Circumflex nerve.
d. Radial nerve.
e. Nerve to latissimus dorsi muscle.
P a g e 5 | 26
SurgiNotes 2nd Ed. (2022)

21.Which of the following is NOT true about the femoral ring?


a. It is bounded anteriorly by the inguinal ligament.
b. It is bounded posteriorly by the pectineal ligament.
c. It is bounded medially by the lacunar ligament.
d. It is bounded laterally by the femoral artery.
e. It contains the lymph node of Cloquet.

22.Which of the following is NOT true about the parotid gland?


a. The capsule is formed from the investing layer of cervical fascia.
b. The secretory supply is by the auriculotemporal nerve.
c. The external carotid artery transverses the gland.
d. Branches of the facial nerve are deep to the retromandibular vein.
e. Lymph nodes may be within the gland.

23.Which of the following does NOT pass in the adductor (Hunter's) canal?
a. Femoral artery.
b. Saphenous nerve.
c. Nerve to vastus medialis.
d. Obturator nerve.
e. Femoral vein.

24.Which of the following is NOT true about the anatomy of the ureter?
a. It crosses the bifurcation of the common iliac artery.
b. It passes superficial to the uterine artery.
c. The ductus deferens crosses superficial to the ureter near its termination.
d. It receives blood supply from the renal, aortic and vesical arteries.
e. It is supplied by sympathetic autonomic nerves T11-L1.
P a g e 6 | 26
SurgiNotes 2nd Ed. (2022)

25.Which of the following is TRUE about the ilioinguinal nerve?


a. It supplies the cremasteric muscle.
b. It supplies the lower part of the rectus abdominis muscle.
c. It supplies the skin on the inner side of the upper part of the thigh.
d. It represents the collateral branch of the second lumbar nerve.
e. In the inguinal canal it has posterior to the spermatic cord.

26.Which of the following is NOT present in injury of the common peroneal


nerve?
a. Loss of eversion of the foot.
b. Loss of extension of the foot.
c. Loss of sensations at the sides of the third toe.
d. Loss of sensations at the lateral aspect of the leg.
e. Loss of planter flexion of the foot.

27.Which of the following is NOT a structure of the spermatic cord?


a. Testicular artery.
b. Testicular veins.
c. Genital branch of the genito-femoral nerve.
d. Ilioinguinal nerve.
e. Sympathetic fibres from the aortic sympathetic plexus.

28.Which of the following is NOT true about the inguinal canal?


a. The inferior wall is reinforced in its medial part by the lacunar ligament.
b. The anterior wall is reinforced in its lateral third by the fibres of the
internal oblique muscle.
c. The superior wall is formed by the lowest arching fibres of the internal
oblique and transverses abdominis.
d. The deep inguinal ring is lateral to the inferior epigastric vessels.
e. The iliohypogastric nerve runs superficial to the spermatic cord.
P a g e 7 | 26
SurgiNotes 2nd Ed. (2022)

29.Which of the following tests will be affected after intake of non-steroidal anti-
inflammatory drugs?
a. Coagulation time.
b. PTT.
c. INR.
d. Bleeding time.
e. Thrombin time.

30.Which of the following statements is NOT correct about hypokalemia?


a. It may occur in Conn's syndrome.
b. It presents by malaise and muscle weakness.
c. It may occur in an Addisonian crisis.
d. It may lead to paralytic ileus.
e. ECG shows elevation of the ST segment.

31.Which of the following is NOT true about neurogenic shock?


a. It may follow spinal fractures.
b. There is tachycardia.
c. The extremities are warm.
d. There is bradycardia and hypotension.
e. Vasopressors may be useful.

32.Which of the following statements is NOT true about haemophilia?


a. It is an X-linked disorder.
b. It may lead to recurrent hemarthrosis.
c. Prothrombin time is increased.
d. It may give rise to severe abdominal pain.
e. It is safe to operate on a hemophilic patient if the concentration of the
specific factor is 30%.
P a g e 8 | 26
SurgiNotes 2nd Ed. (2022)

33.All the following are complications of massive blood transfusion EXCEPT:


a. Hypothermia.
b. Hypocalcemia.
c. Hypokalaemia.
d. Acidosis.
e. DIC.

34.Regarding hyperkalemia all are true EXCEPT:


a. Is mostly the result of renal failure.
b. Is common with massive tissue destruction.
c. Muscle rigidity is a manifestation of severe hyperkalemia.
d. Causes a peaked T-wave on the electrocardiogram.
e. Urgent treatment is by Ca gluconate and Na bicarbonate.

35.The major source of protein for multi-trauma patients during the catabolic
phase is:
a. Plasma protein.
b. Fat.
c. Liver.
d. Skeletal muscle.
e. Kidney.

36.Which of the following cell types is essential for normal wound healing?
a. Leukocytes.
b. Monocytes.
c. Platelets.
d. Erythrocytes.
e. Lymphocytes.
P a g e 9 | 26
SurgiNotes 2nd Ed. (2022)

37.Which of the following is the MOST sensitive liver function:


a. Prothrombin time and concentration.
b. Partial thromboplastin time.
c. Level of fibrinogen.
d. Coagulation time.
e. AST.

38.Low molecular weight heparin produces its effects due to inhibition of which
factor?
a. IIa.
b. IXa.
c. Xa.
d. XI.
e. XII.

39.The following statements are correct EXCEPT:


a. Hypoproteinaemia impairs wound healing.
b. Immobilization of the wounded part helps wound healing.
c. Corticosteroid intake impairs wound healing.
d. The intake of cancer chemotherapy agents delays wound healing.
e. Irradiation of a wound promotes healing.

40.Which of the following statements is NOT true about potassium?


a. Plasma level is raised in metabolic acidosis.
b. The plasma level is a reliable index of total body potassium.
c. Plasma potassium represents 0.4% of total body potassium.
d. In Conn's syndrome there is hypokalemia.
e. Calcium infusion can be used for the treatment of hyperkalaemia.
P a g e 10 | 26
SurgiNotes 2nd Ed. (2022)

41.Which of the following statements is NOT true about secretin?


a. It is secreted mainly in the duodenum.
b. The main stimulus for secretion is the presence of acid in the duodenum.
c. It stimulates the secretion of thick enzyme rich pancreatic juice.
d. It decreases gastric acid section.
e. It stimulates gall bladder contraction.

42.Intravenous infusion of epinephrine will cause all the following EXCEPT:


a. Tachycardia.
b. Hyperglycaemia.
c. Bronchodilatation.
d. Increased splanchnic blood flow.
e. Decreased bowel mobility.

43.Which of the following stimulates glycogenesis?


a. Cortisol.
b. Insulin.
c. Glucagon.
d. Aldosterone.
e. Thyroxine.

44.Which of the following is the best index for the severity of shock?
a. Hb%.
b. Creatinine level.
c. Lactate level.
d. O2 saturation of arterial blood.
e. Plasma sodium concentration.
P a g e 11 | 26
SurgiNotes 2nd Ed. (2022)

45.Which of the following is NOT associated with acute response to injury?


a. Increased plasma cortisol.
b. Raised plasma fatty acids.
c. Increased glucagon level.
d. Reduced liver glycogen.
e. Reduced aldosterone level.

46.Which of the following does NOT cause increased gastrin release from the
pyloric antrum?
a. Antral distension.
b. Vagal stimulation
c. Products of protein digestion.
d. Antral acidity.
e. Antral alkalinity.

47.Which of the following is NOT a cause of hypokalemia?


a. Pyloric obstruction.
b. Alkalosis.
c. Metabolic acidosis.
d. Villous papilloma of the colon.
e. The use of carbonic anhydrase inhibitors.

48.Which of the following is NOT true about hypokalemia?


a. It may be present in paralytic ileus.
b. It may be present after insulin therapy.
c. The reflexes are decreased.
d. ECG shows a wide QRS complex.
e. It is present in alkalosis.
P a g e 12 | 26
SurgiNotes 2nd Ed. (2022)

49.Which of the following is NOT true about enteral nutrition?


a. It may lead to diarrhea.
b. There is a risk of gastric reflux and pulmonary complications.
c. It is not suitable for long-term nutrition.
d. It is superior to parenteral nutrition.
e. It may cause abdominal colic.

50.Which of the following is NOT correct about the catabolic response to trauma?
a. Main source of glucose is the liver glycogen.
b. There is loss of the muscle mass.
c. Intravenous hyperalimentation diminishes but cannot prevent the
catabolic response.
d. It is related to the severity of the trauma.
e. The response is initiated by catecholamines.

51.Which of the following is NOT true about the physiology of the thyroid gland?
a. Thyroxine is mainly bound to thyroid binding globulin.
b. T3 is 3-4 times more active than T4.
c. Cells of thyroid follicles have prominent endoplasmic reticulum.
d. The half-life of thyroxine is about four days.
e. Thiouracil blocks the binding of iodine to tyrosine.

52.The MOST important difference between interstitial fluid and plasma is the:
a. Osmolarity.
b. Potassium concentration.
c. Sodium concentration.
d. Concentration of proteins.
e. Chloride concentration.
P a g e 13 | 26
SurgiNotes 2nd Ed. (2022)

53.Which of the following is the major stimulus for aldosterone secretion?


a. Angiotensin II.
b. ACTH.
c. Hypokalaemia.
d. Hypernatraemia.
e. Cortisol.

54.The effects of glucocorticoid hormones include all the following EXCEPT:


a. Increased hepatic glycogen synthesis.
b. Decreased glucose uptake by the adipose tissue.
c. Increased hepatic gluconeogenesis.
d. Decreased protein synthesis in the skeletal muscles.
e. Decreased glucose uptake by the skeletal muscles.

55.In a healthy adult man weighing 70 Kg:


a. The body water content is 75%.
b. The plasma volume is approximately 5 litres.
c. Approximately 1.5 litres of water are lost daily from the lungs.
d. The intracellular fluid volume is approximately two-thirds of the total
body water.
e. The daily potassium requirement is approximately 3.5-5 mmol/Kg.

56.During prolonged starvation the brain's main fuel is:


a. Lactose.
b. Amino acid.
c. Glucose.
d. Short-chain fatty acid.
e. Ketones.
P a g e 14 | 26
SurgiNotes 2nd Ed. (2022)

57.Which of the following statements is NOT true about DVT?


a. The clinical picture is not accurate in the diagnosis.
b. The incidence of DVT following hip surgery is about 10%.
c. Duplex scanning is highly accurate in the diagnosis.
d. Malignancy is an important predisposing factor.
e. Venography is not commonly performed for the diagnosis nowadays.

58.Which of the following is NOT true in DIC?


a. It may occur in septic shock.
b. Fibrinogen level is low.
c. FDP are elevated.
d. PT and PTT are elevated.
e. There is elevated platelet count.

59.Which of the following is a contraindication to heparin therapy?


a. Closed head injury two weeks ago.
b. Third trimester of pregnancy.
c. Heparin-induced thrombocytopenia.
d. Subclavian vein thrombosis.
e. Superior mesenteric artery embolism.

60.All the following may lead to DIC EXCEPT:


a. Accidental hemorrhage.
b. Burn.
c. Metastatic carcinoma of the prostate.
d. Von-Willebrand's disease.
e. Incompatible blood transfusion.
P a g e 15 | 26
SurgiNotes 2nd Ed. (2022)

61.The first important event in hemostasis following severe tissue injury is:
a. Blood coagulation.
b. Formation of a platelet plug.
c. Vascular spasm.
d. Formation of thromboplastin.
e. Formation of prothrombin activator.

62.Which of the following is associated with a defect of the platelet function?


a. Epistaxis.
b. Haemorthrosis.
c. Soft tissue hemorrhages.
d. Normal bleeding time.
e. Prolonged prothrombin time.

63.Which of the following is NOT true regarding warfarin?


a. They inhibit the synthesis of biologically active forms of coagulation
factors II, VII, IX and X.
b. They exert their action after 2-3 days.
c. The dose is controlled by the partial thromboplastin time.
d. The dose needs to be reduced if there is concomitant intake of aspirin.
e. Vitamin K is the antidote to warfarin.

64.Which of the following is NOT transmitted through blood transfusion?


a. Malaria.
b. Hepatitis C.
c. Human immunodeficiency virus.
d. Cytomegalovirus.
e. Clostridium difficile.
P a g e 16 | 26
SurgiNotes 2nd Ed. (2022)

65.You were obligated to perform cholecystectomy for a cirrhotic patient. During


surgery there was excessive bleeding. What would you order?
a. Fresh blood.
b. Fresh platelets.
c. IV vitamin K.
d. Fresh frozen plasma.
e. IV factor.

66.You were obliged to operate for a strangulated inguinal hernia in a patient


who was receiving 150 mg aspirin daily. During surgery there was excessive
bleeding. What would you advise?
a. Fresh blood.
b. Fresh frozen plasma.
c. Desmopressin.
d. IV vitamin K.
e. Fresh platelets.

67.You are planning to perform cholecystectomy for a female patient who is


receiving warfarin 5 mg/daily because she had mitral valve replacement 4
years ago. What is your pre-operative management?
a. Stop warfarin and substitute it by heparin for 3 days preoperatively.
b. Stop warfarin and substitute it by heparin for 10 days preoperatively.
c. Continue warfarin and give fresh frozen plasma intraoperatively.
d. Continue warfarin and give IV vitamin K intraoperatively.
e. Continue warfarin and give fresh platelets intraoperatively.

68.Which of the following statements is NOT correct about tumour necrosis


factor (TNF-alpha)?
a. It is released mainly from macrophages.
b. It can cause multiple organs failure.
c. High concentration is present in cases of septic shock.
d. It is one of the interferons.
e. It cause anorexia, tachycardia and fever.
P a g e 17 | 26
SurgiNotes 2nd Ed. (2022)

69.Multisystem failure in septic shock is caused by the following EXCEPT:


a. Release of chemical mediators (cytokines) by macrophages.
b. Damage of vascular endothelium.
c. Defective oxygen delivery to the cells.
d. Marked anemia.
e. Dysfunction of intestinal mucosa allows more bacteria to enter the blood
stream.

70.which of the following is wrong about hyperacute rejection?


a. It is due to the presence of preformed antibodies.
b. It occurs within hours of transplantation.
c. It can be treated by corticosteroids.
d. It is best treated by removal of the graft.
e. Cyclosporin A did not reduce its incidence.

71.Regarding necrotizing fasciitis:


a. Skin may show hemorrhagic bullae.
b. It is commonly a single microbial infection.
c. The muscles are usually involved.
d. Broad spectrum antibiotics are the most essential measure of treatment.
e. It commonly occurs in previously healthy people.

72.Which of the following leucocytes will be involved in the development of a


stitch granuloma?
a. Basophils.
b. Eosinophils.
c. Monocytes.
d. Neutrophils.
e. Lymphocyte.
P a g e 18 | 26
SurgiNotes 2nd Ed. (2022)

73.Which of the following statements regarding wound healing is NOT true?


a. A dehisced wound that is re-sutured gains strength faster than primary
wound.
b. Wounds heal faster at 39°C than at 37 °C.
c. Wound healing is accelerated in a low tissue oxygen environment.
d. Synthesis of new collagen is blocked in ascorbic acid deficiency.
e. Wound healing will not occur normally in the absence of monocytes.

74.The following measures are important contributing factors in controlling


bacterial wound contamination in the operating room EXCEPT:
a. Appropriately timed preoperative antibiotic prophylaxis.
b. Scrubbing, gowning and gloving.
c. Surgical site hair shaving in the evening prior to operation.
d. Antimicrobial skin preparation.
e. Refined and aseptic techniques.

75.All of the following statements regarding the use of systemic prophylactic


antibiotics are true EXCEPT:
a. The goal is to attain high tissue level at time of incision.
b. Should be as broad-spectrum as possible in most cases.
c. Are usually given as a single dose.
d. They are not effective if given 3 hours after making the incision.
e. Are not effective in reducing postoperative respiratory infections.

76.Which of the following is NOT true about systemic inflammatory response


(SIRS)?
a. It may be caused by peritonitis, burns and acute pancreatitis.
b. The white leucocytic count may be reduced below 4000.
c. There is metabolic acidosis.
d. The temp may drop to 36 °C.
e. There is reduced output of catecholamines.
P a g e 19 | 26
SurgiNotes 2nd Ed. (2022)

77.Which of the following is an early manifestation of sepsis?


a. Decreased cardiac output.
b. Hypoglycaemia.
c. Cutaneous vasodilatation.
d. Increased arteriovenous O2 difference.
e. Respiratory acidosis.

78.Which of the following is the MOST likely cause of pyrexia occurring 48 hours
after an abdominal operation?
a. Wound infection.
b. DVT.
c. Chest infection.
d. Pulmonary embolism.
e. Leaking intestinal anastomosis.

79.Which of the following is NOT true about gas gangrene?


a. It is mainly caused by clostridium welchii.
b. There is usually crepitus and color changes in the affected area.
c. The main effects are due to bacterial endotoxin.
d. Anti-gas gangrene serum is no more used in the treatment.
e. Debridement of the affected tissues is vital in the treatment.

80.One of the following is an anaerobic organism:


a. Staphylococcus aureus.
b. E. coli.
c. Klebsiella.
d. Pseudomonas aeruginosa.
e. Bacteroides.
P a g e 20 | 26
SurgiNotes 2nd Ed. (2022)

81.The organism that is famous for production of the enzyme penicillinase and
resists penicillin therapy is:
a. Staphylococcus aureus.
b. Haemolytic streptococci.
c. Klebsiella.
d. Pseudomonas aeruginosa.
e. Bacteroides.

82.The main source of staphylococci in the body is:


a. The urinary tract.
b. The stomach, duodenum and upper jejunum.
c. The colon.
d. The nasal cavity and skin.
e. None of the above.

83.Which of the following is NOT correct about erysipelas?


a. Caused by methicillin resistant staph. aureus (MRSA).
b. Sensitive to penicillin.
c. Infectious.
d. Spreading infection with no pus formation.
e. Rose pink skin patches with vesicles at the edge.

84.In pseudomembranous enterocolitis the causing organism is:


a. Clostridium perfringens.
b. Clostridium septicum.
c. Clostridium difficile.
d. Clostridium tetani.
e. Clostridium oedematiens.
P a g e 21 | 26
SurgiNotes 2nd Ed. (2022)

85.Which of the following is true regarding antibiotics?


a. Penicillins act by disrupting the peptidoglycan of the bacterial cell wall.
b. Ampicillin in effective against pseudomonas infections.
c. Vancomycin is the first choice for treatment of infections with staph.
aureus.
d. Aminoglycosides may cause hepatotoxicity.
e. Cephalosporins are rarely used for prophylaxis.

86.What is MOST important in the management of a lacerated contaminated


wound?
a. Removal of foreign and dead tissues.
b. Local antibiotics powder.
c. Insertion of a drain.
d. Skin graft.
e. Hyperbaric O2.

87.Clavulanic acid is given in combination with amoxicillin in order to:


a. Reduce the risk of allergic reactions.
b. Prolong amoxicillin half-life.
c. Reduce the severity of diarrhea.
d. Extend amoxicillin antibacterial spectrum.
e. Protect against gastric acidity.

88.The aminoglycoside MOST likely to remain a useful therapeutic agent in the


event of gentamicin resistance is:
a. Streptomycin.
b. Amikacin.
c. Neomycin.
d. Tobramycin.
e. Kanamycin
P a g e 22 | 26
SurgiNotes 2nd Ed. (2022)

89.The MOST frequent serious complication of treatment with gentamycin is:


a. Serum sickness.
b. Kidney damage.
c. Reactivation of tuberculosis.
d. Blood dyscrasias.
e. Intravascular thrombosis.

90.The effectiveness of prophylactic antibiotics in surgery is MOSTLY related to


the:
a. Use of broad-spectrum agents.
b. Continuation of antibiotics for 24 hours after surgery.
c. Timing of initial administration.
d. Use of two synergistic antibiotics.
e. Use of bactericidal agents.

91.The MOST deleterious effect of excessive glucose in the formula of


longstanding TPN is:
a. Hyperglycemia.
b. Excessive diuresis.
c. Hepatic steatosis.
d. Hyponatremia.
e. Hypoglycemia.

92.Arterial blood gas analysis revealed a pH 7.6, P(O2) 85 mmHg, P(CO2) 46 mmHg
and HCO3- 44 mmol/L. This denotes:
a. Metabolic acidosis.
b. Respiratory alkalosis.
c. Respiratory acidosis.
d. Metabolic alkalosis.
e. Combined respiratory and metabolic alkalosis.
P a g e 23 | 26
SurgiNotes 2nd Ed. (2022)

93.The best initial therapy for major pulmonary embolism is:


a. Aspirin.
b. Heparin.
c. Warfarin.
d. A fibrinolytic agent.
e. Corticosteroids.

94.Arterial blood gas values of a patient are:


PH 7.36.
PO2 45 mmHg.
PCO2 60 mmHg.
HCO3- 34 mmol/L.
Base excess 5 mmol/L.
These values are consistent with:
a. Acute morphine overdosage.
b. Sepsis.
c. Hypovolemic shock.
d. Lobar atelectasis.
e. Severe chronic obstructive airway disease.

95.Which of the following is an appropriate definition of the shock state?


a. Low blood pressure to maintain normal metabolic and nutritional
functions.
b. Low cardiac output to maintain metabolic and nutritional functions.
c. Shock index greater than 0.9 to maintain metabolic and nutritional
functions.
d. Inadequate tissue perfusion to maintain normal metabolic and nutritional
functions.
e. Abnormal vascular resistance to maintain normal metabolic and
nutritional functions.

96.Which of the following biochemical changes is NOT seen in shocked patients?


a. Hyperglycemia.
b. Negative nitrogen balance.
c. Hyperlacticacidemia.
d. Metabolic alkalosis.
e. Hyperkalemia.
P a g e 24 | 26
SurgiNotes 2nd Ed. (2022)

97.Among the physiologic responses to acute injury one is true:


a. Increased secretion of insulin.
b. Increased secretion of thyroxine.
c. Decreased secretion of vasopressin (ADH).
d. Decreased secretion of glucagon.
e. Decreased secretion of aldosterone.

98.A 20-year-old woman has the following arterial blood results: pH 7.49, PCO2
47 mmHg, HCO3- 35 mmol/L and oxygen saturation 98% on air. Her plasma
potassium concentration is 2.5 mmol. The physiologic status can best be
described as which of the following?
a. There is a respiratory alkalosis.
b. There is a metabolic alkalosis.
c. Her urine is likely to be acidic.
d. Pulmonary embolism is a likely diagnosis.
e. There is respiratory acidosis.

99.The LEAST safe artery to insert an arterial line into is the:


a. Radial.
b. Ulnar.
c. Brachial.
d. Axillary.
e. Femoral.

100. Which of the following is MOST accurate in the diagnosis of pulmonary


embolism?
a. ECG.
b. Chest X-ray.
c. D dimmer test.
d. CT angiography.
e. Ventilation perfusion lung scan.
P a g e 25 | 26
SurgiNotes 2nd Ed. (2022)

Egyptian Board Of General Surgery


1st Part Exam – Paper (1) MCQ "ANSWER"
January 2010

1 c 26 e 51 d 76 e
2 c 27 d 52 d 77 c
3 c 28 e 53 a 78 c
4 c 29 d 54 a 79 c
5 d 30 e 55 d 80 e
6 d 31 b 56 e 81 a
7 c 32 c 57 b 82 d
8 c 33 c 58 e 83 a
9 c 34 c 59 c 84 c
10 d 35 d 60 d 85 a
11 d 36 b 61 c 86 a
12 c 37 a 62 a 87 d
13 b 38 c 63 c 88 b
14 a 39 e 64 e 89 b
15 d 40 b 65 d 90 c
16 e 41 e 66 c 91 a
17 a 42 d 67 a 92 d
18 c 43 b 68 d 93 d
19 c 44 c 69 d 94 e
20 c 45 e 70 c 95 d
21 d 46 d 71 a 96 d
22 d 47 c 72 c 97 a
23 d 48 d 73 c 98 b
24 b 49 c 74 c 99 d
25 c 50 a 75 e 100 d

P a g e 26 | 26
SurgiNotes 2nd Ed. (2022)

Egyptian Board Of General Surgery


1st Part Exam – Paper (1) MCQ
June 2013
1. A 30-year-old male patient had a road traffic accident. Examination revealed loss of
contour of the shoulder and the arm was held in an adducted position. The patient
could not move his shoulder. Which of the following nerves was MOST likely injured
in this patient?
a. Axillary (circumflex).
b. Median.
c. Musculocutaneous.
d. Radial.
e. Ulnar.
2. A 42-year-old woman presents to her doctor with shoulder pain. On examination,
shoulder abduction is weak and painful. Which muscle is likely to be affected?
a. Infraspinatus.
b. Subscapularis.
c. Supraspinatus.
d. Teres major
e. Teres minor.
3. A 32-year-old man is brought to the emergency department following a fall. An X-
ray reveals that he has a fracture of the surgical neck of his humerus. Neurological
examination reveals paraesthesia over the upper lateral arm (overlying the deltoid
muscle). Which nerve is MOST likely to have been damaged?
a. Axillary.
b. Media.
c. Musculocutaneous.
d. Radial.
e. Ulnar.
4. While assessing an elderly woman with severe neck arthritis, you note she has
weakness of finger abduction and adduction. This is MOST likely to be due to
compression of which spinal nerve root of the brachial plexus?
a. Fifth cervical.
b. Sixth cervical.
c. Seventh cervical.
d. Eighth cervical.
e. First thoracic.
P a g e 1 | 26
SurgiNotes 2nd Ed. (2022)

5. Which one of the following muscles is an extensor of the hip?


a. Adductor longus.
b. Gracilis.
c. Iliopsoas.
d. Pectineus.
e. Semitendinosus.

6. In L5 nerve root compression from a herniated intervertebral disc, altered


cutaneous sensation is predominantly in the:
a. Dorsum of foot.
b. Heel of foot.
c. Medial border of foot.
d. Outer border of foot.
e. Sole of foot.

7. A 47-year-old woman undergoes a modified radical mastectomy for a T2N2


infiltrating ductal carcinoma. She arrives at her first postoperative visit
complaining of hypoesthesia of the upper posteromedial aspect of the
ipsilateral arm. What might explain this finding?
a. Lymphatic fibrosis.
b. Medial pectoral pedicle injury.
c. Second intercostal brachial cutaneous nerve injury.
d. Axillary vein thrombosis.
e. Thoraco-dorsal pedicle injury.

8. Which of the following nerves is liable to be injured during opening of the


inguinal canal in herniorrhaphy operation?
a. Iliohypogastric nerve.
b. Obturator nerve.
c. Lateral femoral cutaneous nerve.
d. Ilioinguinal nerve.
e. Pudendal nerve.
P a g e 2 | 26
SurgiNotes 2nd Ed. (2022)

9. An 82-year-old man has complete occlusion of his inferior mesenteric artery


on angiography but no symptoms or signs of colonic ischemia. Which of the
following arteries is the MOST likely additional source of blood supply to the
territory of the inferior mesenteric artery?
a. Left colic.
b. Left gastroepiploic.
c. Middle colic.
d. Splenic.
e. Superior rectal.

10.A 70-year-old woman undergoes a left mastectomy and axillary clearance. At


a follow-up appointment she was noted to have winging of the left scapula.
Which muscle has been paralysed as a result of the dissection of the axilla?
a. Latissimus dorsi.
b. Pectoralis major.
c. Serratus anterior.
d. Teres major.
e. Trapezius.

11.A 42-year-old construction worker noted a swelling in the right submandibular


region. Biopsy reveals malignancy and surgical excision is advised. The patient
is informed that one of the risks of this operation is which of the following?
a. Horner syndrome.
b. Excessive sweating in the temporal region.
c. Deformity of the angle of the mouth.
d. Submandibular duct calculus.
e. Trismus.

12.After undergoing a surgical procedure on the small (short) saphenous vein, a


patient complains of pain and numbness on the lateral aspect of the foot. The
nerve MOST likely to be affected is the:
a. Deep peroneal (fibular).
b. Lateral plantar.
c. Saphenous.
d. Superficial peroneal (fibular).
e. Sural.
P a g e 3 | 26
SurgiNotes 2nd Ed. (2022)

13.A 35-year-old woman is seen at the surgical clinic with a history of faecal
incontinence since the birth of her second child 18 months previously. The first
stage of labour had been prolonged and difficult. Physical examination reveals
a relatively lax anal sphincter. Which nerve is likely to have been damaged in
labour?
a. Genitofemoral nerve.
b. Lumbosacral trunk.
c. Obturator nerve.
d. Pelvic splanchnic nerve.
e. Pudendal nerve.
14.A 27-year-old man is admitted to the emergency department after a car crash.
Physical examination reveals weakness in medial rotation and adduction of
the humerus. Which of the following nerves was MOST probably injured?
a. Thoracodorsal.
b. Axillary.
c. Dorsal scapular.
d. Spinal accessory.
e. Radial.
15.A 5-year-old male entered the emergency department with a complaint of
severe abdominal pain. During physical examination it is observed that his
cremasteric reflex is absent. Which of the following nerves is responsible for
the efferent limb of the cremasteric reflex?
a. Ilioinguinal.
b. Iliohypogastric.
c. Genitofemoral.
d. Pudendal.
e. Ventral ramus of T12.
16.A 45-year-old woman is admitted to the emergency department with a
complaint of severe abdominal pain. CT scan and MRI examinations reveal a
tumor of the head of the pancreas involving the uncinate process. Which of
the following vessels is MOST likely to be occluded?
a. Common hepatic artery.
b. Cystic artery and vein.
c. Superior mesenteric artery.
d. Inferior mesenteric artery.
e. Portal vein.
P a g e 4 | 26
SurgiNotes 2nd Ed. (2022)

17.Upon removal of a leg cast, a 15-year-old boy complains of numbness of the


dorsum of his right foot and inability to dorsiflex and evert his foot. Which is
the MOST probable site of the nerve compression that resulted in these
symptoms?
a. Popliteal fossa.
b. Neck of the fibula.
c. Lateral compartment of the leg.
d. Anterior compartment of the leg.
e. Medial malleolus.

18.A 42-year-old male fell from a height. Radiographic examination reveals


fracture of the proximal femur. Which of the following arteries supplies the
proximal part of the femur?
a. Deep circumflex iliac.
b. Acetabular branch of obturator.
c. Lateral circumflex femoral.
d. A branch of profunda femoris.
e. Medial circumflex femoral.

19.which of the following is TRUE regarding the secretory functions of the


pancreas?
a. Secretin releases fluid rich in enzymes.
b. Secretin releases fluid rich mainly in electrolytes and bicarbonate.
c. Cholecystokinin releases fluid, predominantly rich in electrolytes and
bicarbonate.
d. All pancreatic enzymes are secreted in an inactive form.
e. The pancreas produces proteolytic enzymes only.

20.A 63-year-old man undergoes a peripheral vascular procedure under general


anesthesia. A decrease in urine formation and excretion is noted. Decreased
urine flow under general anesthesia occurs because of which of the following?
a. Vasopressin.
b. Aldosterone suppression.
c. Depression of glucocorticoid.
d. Depression of thyroid function.
e. Specific effect of anesthesia on renal tubules.
P a g e 5 | 26
SurgiNotes 2nd Ed. (2022)

21.In compensating for respiratory alkalosis, the body excretes more:


a. Ammonium ions.
b. Bicarbonate ions.
c. Dihydrogen phosphate ions.
d. Carbonic acid.
e. Hydrogen ions.

22.A 50-year-old female patient complains of thirst, polyuria and dehydrated


tongue due to uncontrolled diabetes mellitus. What is the cause of metabolic
acidosis in this patient?
a. High glucose levels depress the respiratory center in the medulla.
b. Glucose is an acidic substance.
c. Glucose is osmotically active and for every water molecule retained, a
hydrogen ion is also retained.
d. Most diabetics have chronic diarrhea, which leads to excessive loss of
bicarbonate ions.
e. Increased rate of lipolysis and ketogenesis.

23.A patient whose blood pH is 7.47, whose PCO2 is 31 mmHg in arterial blood
and whose levels of bicarbonate ion in arterial blood are 23 mEq/liter is in:
a. Compensated metabolic alkalosis.
b. Uncompensated respiratory acidosis.
c. Uncompensated respiratory alkalosis.
d. Uncompensated metabolic acidosis.
e. Uncompensated metabolic alkalosis.

24.What is the MOST serious physiological abnormality in acute renal failure?


a. Fluid overload.
b. ↑ Creatinine.
c. ↑ Urea.
d. Acidosis.
e. Raised blood urea N2.
P a g e 6 | 26
SurgiNotes 2nd Ed. (2022)

25.Which of the following is a function of the parasympathetic system?


a. Increase the blood supply of the heart.
b. Increase the blood supply of skeletal muscles.
c. Increase in the heart rate.
d. Stimulation of glycogenolysis.
e. Increased motility of the gastrointestinal tract.

26.A fit 30-year-old man donates 500 ml of blood. Which one of the following is
the MOST likely physiological change?
a. A fall in blood pressure.
b. Activation of renin-angiotensin system.
c. Reduced urine output.
d. Sweating.
e. Tachypnoea.

27.A 65-year-old man undergoes trans-sphenoidal surgery for a pituitary macro-


adenoma. On the first postoperative day he is noted to be confused. The
MOST likely cause is:
a. Hyperkalaemia.
b. Hyperuricaemia.
c. Hypoglycaemia.
d. Hyponatraemia.
e. Hypoxia.

28.A 78-year-old woman with emphysema receiving 28% oxygen by mask has the
following blood gas results:
PH PO2 PaCO2 Bicarbonate Base excess
Finding: 7.28 70 mmHg 48 mmHg 36 mmol/L +5
The MOST likely interpretation is:
a. Mixed respiratory and metabolic acidosis.
b. Partially compensated metabolic acidosis.
c. Partially compensated respiratory acidosis.
d. Uncompensated metabolic acidosis.
e. Uncompensated respiratory acidosis.
P a g e 7 | 26
SurgiNotes 2nd Ed. (2022)

29.A 40-year-old woman had the anterior lobe of the pituitary removed because
of a tumour. Without postoperative supplements, which of the following
could occur?
a. Failure to produce adequate amounts of thyroxine.
b. Failure to produce parathyroid hormone in response to hypocalcaemia.
c. Failure to secrete catecholamines in response to stress.
d. Failure to secrete insulin in hyperglycaemia.
e. Inability to concentrate urine in response to water deprivation.

30.A 70-year-old patient with diabetes and paraplegia is undergoing an elective


laparoscopic cholecystectomy after an episode of biliary pancreatitis. Shortly
after induction, blood pressure is normal, but ECG shows peaked P waves and
a widened QRS complex. The MOST likely diagnosis is:
a. Ketoacidosis.
b. Hyperkalemia.
c. Hypoglycemia.
d. Hypocalcemia.
e. Acute myocardial infarction.

31.A 48-year-old woman presents to the emergency department with persistent


diarrhea. On examination, she is apyrexial and has an irregularly irregular
pulse of 140 beats/minute. The MOST likely cause of the abnormal cardiac
rhythms is:
a. Hypernatraemia.
b. Hypokalaemia.
c. Hypovolaemia.
d. Myocardial ischaemia.
e. Sepsis.

32.A 70-year-old man with chronic obstructive pulmonary disease is admitted for
elective hemicolectomy. What is a preoperative arterial blood gas analysis
likely to show?
Arterial PCO2 Bicarbonate
a Decrease Decreased
b Decreased Increased
c Decreased Normal
d Increased Decreased
e Increased Increased
P a g e 8 | 26
SurgiNotes 2nd Ed. (2022)

33.Which of the following cells is an example of a permanent cell NOT capable of


division?
a. Acinar cells of the pancreas.
b. Colonic mucosal cells.
c. Erythrocytes.
d. Hepatocytes.
e. Osteocytes.

34.In patients with reflux esophagitis. What does the presence of columnar cells
in the esophageal mucosa represent?
a. Carcinoma in situ.
b. Carcinoma.
c. Dysplasia.
d. Hyperplasia.
e. Metaplasia.

35.Which of the following cells are commonly found in tissues undergoing chronic
inflammation?
a. Eosinophils.
b. Lymphocytes.
c. Mast cells.
d. Platelets.
e. Polymorphonuclear leucocytes.

36.With regard to protein loss after injury, which of the following statements is
TRUE?
a. It can be prevented by total parenteral nutrition.
b. It occurs primarily from skeletal muscle.
c. It occurs primarily from the site of injury.
d. It results from impaired synthesis.
e. It results from significant decrease intake.
P a g e 9 | 26
SurgiNotes 2nd Ed. (2022)

37.Which of the following can decrease the adverse effects of steroids on wound
healing?
a. Copper.
b. Vitamin A.
c. Vitamin C.
d. Vitamin D.
e. Vitamin E.

38.Histopathological examination of a cavitating apical lung mass from a 45-year-


old man shows necrotic material surrounded by epithelioid cells and
occasional multinucleated giant cells. Acid fast bacilli are identified. Which cell
type gives rise to the giant cells?
a. Basophil.
b. Eosinophil.
c. Lymphocyte.
d. Macrophage.
e. Neutrophil.

39.With regard to the healing process which of the following statements is


correct?
a. Collagen content reaches a maximum at approximately one week after
injury.
b. Monocytes are essential for normal wound healing.
c. Fibroblasts appear in the wound within 24 to 36 h after the injury.
d. The function of monocytes in wound healing is limited to phagocytosis of
bacteria and debris.
e. Early in wound healing, type I collagen is predominant.

40.A 40-year-old male patient had renal transplantation and is receiving


immunosuppression. Few days following the operation the patient developed
persistent fever. Which of the following infectious microorganisms is currently
the MOST likely cause of this fever?
a. Candidiasis.
b. Coli sepsis.
c. Cytomegalovirus sepsis.
d. Pneumococcal sepsis.
e. Streptococci.

P a g e 10 | 26
SurgiNotes 2nd Ed. (2022)

41.A 30-year-old female patient developed postoperative wound infection after


a thyroidectomy operation. What is the MOST likely causative organism?
a. Bacteroides.
b. E. coli.
c. Proteus.
d. Staph. aureus.
e. Streptococci.

42.Which of the following is the mode of action of amikacin?


a. Blockage of bacterial DNA replication.
b. Competitive inhibition of the enzyme dihydrofolate reductase.
c. Inhibition of bacterial wall synthesis.
d. Inhibition of protein synthesis by ribosomes.
e. Inhibition of translocation of peptides.

43.Which of the following organisms is the commonest to cause urinary tract


infection following catheterization?
a. Clostridium difficile.
b. E. coli.
c. Pseudomonas aeruginosa.
d. Staph. aureus.
e. Strept. pneumonia.

44.Which is the initial management of antibiotic associated diarrhea?


a. Amoxycillin.
b. Discontinuing the causative antimicrobial.
c. Metronidazole.
d. Oral erythromycin.
e. Vancomycin.
P a g e 11 | 26
SurgiNotes 2nd Ed. (2022)

45.The use of prophylactic antibiotics has become commonplace. Which of the


following statement(s) is TRUE concerning the prophylactic use of antibiotics?
a. Prophylactic antibiotics should be administered for all surgical
procedures.
b. Continuing thee antibiotic into the postoperative period has led to
improved results in antibiotic prophylaxis.
c. The appropriate use of prophylactic antibiotics must include the initiation
of the agent prior to the surgical procedure.
d. The prophylactic administration of broad-spectrum agents (Third-
generation cephalosporins) has been shown to be particularly
advantageous.
e. The topical use of antimicrobial agents is of no advantage in the
prophylactic setting.

46.Which of the following is the MOST common serious infectious complication


of blood transfusion?
a. Acquired immunodeficiency syndrome.
b. Cytomegalovirus.
c. Malaria.
d. Virus A hepatitis.
e. Virus C hepatitis.

47.What is the commonest organism to cause infections following splenectomy?


a. Haemophilus influenza.
b. Escherichia coli.
c. Neisseria meningitidis.
d. Staphylococcus aureus.
e. Streptococcus pneumonia.

48.Which of the following cells is responsible for cellular mediated immunity?


a. B-lymphocytes.
b. Dendritic cells.
c. Monocytes.
d. Natural killer cells.
e. T-lymphocytes.
P a g e 12 | 26
SurgiNotes 2nd Ed. (2022)

49.The main mode of action of cyclosporine is interference of:


a. Antibody production by plasma cells.
b. Cytotoxic T-cell function.
c. Identification of antigen.
d. Interleukin-2 production.
e. Macrophage.

50.Which of the following cancers is MOST common in organ transplant


recipients?
a. Bronchogenic carcinoma.
b. Renal cell carcinoma.
c. Pancreatic cancer.
d. Prostatic cancer.
e. Skin cancer.

51.Pathogenic mechanisms involved in tuberculosis can be primarily attributed


to which of the following?
a. Cell-mediated hypersensitivity.
b. Clogging of alveoli by large numbers of acid-fast mycobacteria.
c. Humoral immunity
d. Specific cell adhesion sites.
e. Toxic production by the mycobacteria.

52.Which of the following allogeneic grafts does NOT require


immunosuppression?
a. Kidney.
b. Heart.
c. Liver.
d. Bone marrow.
e. Cartilage.
P a g e 13 | 26
SurgiNotes 2nd Ed. (2022)

53.Five days after an uneventful cholecystectomy, an asymptomatic middle aged


woman is found to have a serum sodium level of 120 mEq/L. What is the
proper management?
a. Administration of hypertonic saline solution.
b. Administration of Ringer's lactate solution.
c. Aggressive diuresis with furosemide.
d. Plasma ultra-filtration.
e. Restriction of free water.

54.What is the MOST common fluid or electrolyte disorder in the surgical patient?
a. Extracellular fluid deficit.
b. Hyperkalemia.
c. Hyponatremia.
d. Metabolic: acidosis.
e. Metabolic alkalosis.

55.The osmolarity of the extracellular fluid space is determined primary by the


concentration of:
a. Bicarbonate.
b. Chloride ion.
c. Phosphate radicals.
d. Potassium ions.
e. Sodium ion.

56.Which of the following is characteristic of neurogenic shock?


a. Cool moist skin.
b. Decreased blood volume.
c. Increased peripheral vascular resistance.
d. Increased cardiac output.
e. Bradycardia.
P a g e 14 | 26
SurgiNotes 2nd Ed. (2022)

57.What is the MOST common symptom after major pulmonary embolism?


a. Cough.
b. Dyspnea.
c. Hemoptysis.
d. Pleural pain.
e. Palpitation.

58.A 24-year-old woman has acute renal failure following postpartum


hemorrhage. Laboratory studies showed serum glucose, 150 mg/dl; sodium,
135 mEq/L; potassium, 6.5 mEq/L; chloride, 105 mEq/L and bicarbonate, 15
mEq/L. Which of the following is recommended?
a. Decrease potassium chloride to 10 mEq/day.
b. Intravenous 0.9% sodium chloride.
c. 100 ml of 50% glucose water with 10 U insulin.
d. Intravenous calcitonin.
e. Intravenous magnesium sulfate.

59.A 55-year-old man sustains numerous injuries involving the abdomen and
lower extremities. During the intra- and postoperative periods, he is
resuscitated with 10 L of Ringer's lactate and 2 U of packed red blood cells
(RBC). After initial improvement, he has severe dyspnea on the second
postoperative day. The MOST useful initial diagnostic test is which of the
following?
a. Electrocardiogram.
b. Analysis of arterial blood gases.
c. Insertion of a central venous line.
d. Ventilation-perfusion scan.
e. Computed tomography (CT) scan of abdomen.

60.Which of the following metabolic effects is present in shock?


a. Increase in sodium and water excretion.
b. Increase in renal perfusion.
c. Decrease in cortisol levels.
d. Hyperkalemia.
e. Hypoglycemia.
P a g e 15 | 26
SurgiNotes 2nd Ed. (2022)

61.Which of the following is the commonest cause of ARDS?


a. Sepsis syndrome.
b. Aspiration.
c. Acute pancreatitis.
d. DIC.
e. Fat embolism.

62.A 60-year-old man had undergone exploratory laparotomy for perforated


gastric ulcer with severe peritoneal contamination. Six hours after surgery, he
is tachycardic, hypertensive and has shallow respirations. Intubation and
institution of ventilatory support is indicated in the presence of which of the
following?
a. Respiratory rate of 23 breaths/min.
b. PaCO2 of 45 mmHg.
c. PaO2 of 55 mmHg on room air.
d. Heart rate of 140 bpm.
e. BP of 150/100 mmHg.

63.What is the primary source of calories during acute starvation (< 5 days
fasting)?
a. Skeletal muscle proteins.
b. Body stored fat.
c. Liver glycogen.
d. Ketone bodies.
e. Muscle glycogen.

64.Which of the following types of shock is associated with high pulmonary


wedge pressure?
a. Hypovolemic shock.
b. Cardiogenic shock.
c. Early septic shock.
d. Late septic shock.
e. Neurogenic shock.
P a g e 16 | 26
SurgiNotes 2nd Ed. (2022)

65.A common drug that can trigger an episode of malignant hyperthermia:


a. Propofol.
b. Dantrolene.
c. Ketamine.
d. Succinylcholine.
e. Neostigmine.

66.Which of the following techniques does NOT provide a definitive airway?


a. Cricothyroidotomy.
b. Tracheostomy.
c. Nasotracheal tube.
d. Laryngeal mask airway.
e. Endotracheal tube.

67.Choose the TRUE statement about septic shock:


a. Gram-positive organisms do not cause septic shock because they lack the
cell-wall endotoxin.
b. Leucopenia is not a feature of septic shock.
c. The second most common source of gram-negative bacteremia is the
urinary tract.
d. Escherichia coli is the second most common organism identified in gram-
negative bacteremia.
e. Gram-negative endotoxemia appears to be the result of the abnormal
activation of normal physiologic pathways.

68.A 38-year-old man in end-stage renal failure resulting from polycystic kidney
disease receives a cadaveric renal transplant. Good renal function is
established but four weeks later deteriorates, the serum creatinine rising by
25%. Which of the following processes is MOST likely to be responsible for this
deterioration?
a. B-cell mediated rejection.
b. Circulating immune complex disease.
c. IgG antibody mediated rejection.
d. Post-transplant lymphoproliferative disorder.
e. T-cell mediated rejection.
P a g e 17 | 26
SurgiNotes 2nd Ed. (2022)

69.A 55-year-old male patient is receiving 150 mg of aspirin daily because he has
coronary artery disease. Which of the following tests will be affected in this
patient?
a. Bleeding time.
b. Coagulation time.
c. INR.
d. PTT.
e. Thrombin time.

70.A 30-year-old male patient was admitted to the casualty department due to a
car accident. The patient had fracture of the pelvis and the right femur and he
received 5 liters of blood following which he started to have bleeding from his
nose and mouth. What is the main cause of this bleeding tendency?
a. Decrease in fibrinogen.
b. Decrease in prothrombin.
c. Decrease of calcium.
d. Increased fibrinolytic activity.
e. Platelet depletion.

71.A patient with a life threatening pulmonary embolus is receiving heparin. She
developed serious vaginal bleeding and a major retroperitoneal haematoma
after 5 days of heparin therapy. What is the recommended treatment?
a. Reverse heparin and evacuate the haematoma.
b. Reverse heparin by protamine sulphate and insert a vena caval filter.
c. Stop heparin and closely observe the patient.
d. Stop heparin, give fresh frozen plasma and start warfarin therapy.
e. Switch to low-dose heparin.

72.What is the main mode of action of heparin?


a. It increases the level of protein C.
b. It is a cofactor of antithrombin III.
c. It prevents clot retraction.
d. It prevents platelets aggregation.
e. It prevents the synthesis of fibrinogen.
P a g e 18 | 26
SurgiNotes 2nd Ed. (2022)

73.What is the principal mechanism of the antithrombotic action of administering


aspirin in low dose?
a. Induction of capillary vasodilatation.
b. Induction of endothelial cell prostacyclin production.
c. Induction of endothelial heparin production.
d. Inhibition of factor V production by the liver.
e. Inhibition of platelet production of thromboxane A2.

74.You were obliged to operate for a strangulated inguinal hernia in a patient


who was receiving 300 mg aspirin daily. During surgery there was excessive
bleeding. What would you advise?
a. Desmopressin.
b. Fresh blood.
c. Fresh frozen plasma.
d. Fresh platelets.
e. IV vitamin K.

75.You are planning to perform cholecystectomy for a female patient who is


receiving warfarin 5 mg/daily because she had mitral valve replacement 4
years ago. What is your pre-operative management?
a. Continue warfarin and give fresh frozen plasma intraoperatively.
b. Continue warfarin and give fresh platelets intraoperatively.
c. Continue warfarin and give IV vitamin K intraoperatively.
d. Stop warfarin and substitute it by heparin for 10 days preoperatively.
e. Stop warfarin and substitute it by heparin for 5 days preoperatively.

76.Which of the following is TRUE regarding a male patient who has haemophilia?
a. He has frequent attacks of spontaneous mucocutaneous bleeding.
b. His sisters are affected as well.
c. The partial thromboplastin time is normal.
d. The prothrombin time is prolonged.
e. Transfusion of factor VIII concentrate is helpful.
P a g e 19 | 26
SurgiNotes 2nd Ed. (2022)

77.A cirrhotic patient with abnormal haemostatic studies requires an urgent


cholecystectomy. A transfusion of fresh frozen plasma is planned to minimize
the risk of bleeding. What is the optimal timing of this transfusion?
a. In the recovery room.
b. Intraoperatively.
c. On call to surgery.
d. The night before surgery.
e. Two days before surgery.

78.A fully heparinized patient develops a condition requiring emergency surgery.


After stopping the heparin, what else should be done to prepare the patient?
a. 2 units of cryoprecipitates.
b. Administration of protamine sulphate 1 mg for every 100 units of heparin
most recently administered.
c. Immediate fresh frozen plasma.
d. Transfusion of 10 units of platelets.
e. Vitamin K intravenously.

79.Which of the following mechanisms initiates the normal haemostatic


mechanism?
a. Fibrin formation.
b. Fibrin stabilization.
c. Fibrinolysis.
d. Platelet plug formation.
e. Vascular constriction.

80.You are planning to perform colectomy for a 55-year-old male patient who
had a previous CABG operation and is receiving 150 mg of aspirin daily. What
will be your policy?
a. Continue aspirin and administer fresh frozen plasma intraoepratively.
b. Continue aspirin and administer fresh platelets intraoperatively.
c. Stop aspirin for 5 days preoperatively and substitute it by warfarin.
d. Stop aspirin for 10 days preoperatively and substitute it by subcutaneous
heparin.
e. Stop aspirin for 3 days preoperatively.
P a g e 20 | 26
SurgiNotes 2nd Ed. (2022)

81.Which of the following is the best method of identifying the liability to


bleeding during a surgical procedure?
a. Platelet count.
b. A complete history and physical examination.
c. Bleeding time.
d. Lee-White clotting time.
e. Prothrombin time (PT).

82.What is the treatment of a hemophilic patient who had massive hemarthrosis


of the knee following a fall on his knee?
a. Penicillamine.
b. Fresh frozen plasma.
c. Transfusion of factor VIII to 10% of normal factor levels.
d. Platelet transfusion.
e. Exploration of joint.

83.Which of the following denotes a haemolytic transfusion reaction during


anesthesia?
a. Shaking chills and muscle spasms.
b. Fever and oliguria.
c. Hyperpyrexia and hypotension.
d. Tachycardia and cyanosis.
e. Bleeding and hypotension.

84.A 50-year-old female patient bas chronic renal failure and has been
maintained on chronic dialysis. The patient underwent cholecystectomy. Post-
operatively she had severe bleeding. What is the MOST likely cause for this
bleeding?
a. Elevated PT.
b. Elevated PTT.
c. Low platelet count.
d. Decreased platelet aggregation.
e. Sepsis.
P a g e 21 | 26
SurgiNotes 2nd Ed. (2022)

85.A 70-year-old female patient is receiving warfarin because she has AF and had
previous thrombosis. Which of the following statements regarding warfarin is
correct?
a. The dose of warfarin is adjusted according to the partial thromboplastin
time.
b. Warfarin takes about 8 hours to exert its effects.
c. Warfarin acts by inhibiting factor XII.
d. If the patient is also taking aspirin, the dose of warfarin should be
reduced.
e. Protamine sulphate is the antidote to warfarin.
86.A mother mentions that her 8-year-old boy gets recurrent attacks of
hemarthrosis following a minimal trauma. Investigations revealed normal
platelet count and prothrombin time, but the partial thromboplastin time is
prolonged. Which of the following statements regarding this clinical condition
is TRUE?
a. There is an underlying liver problem.
b. The sisters of this boy are usually having the same problem.
c. The Loy has had repeated episodes of epistaxis.
d. There is no family history in this condition.
e. Transfusion of factor VIII concentrate is helpful.
87.A 40-year-old male patient is complaining of persistent headache, recurrent
attacks of palpitation, sweating and visual disturbances. Blood pressure of the
patient is 180/110 mmHg. Which of the following is MOST accurate in the
diagnosis of the problem?
a. 24-hour urinary catecholamines.
b. 24-hour urinary VMA.
c. CT scan of the abdomen.
d. Plasma aldosterone level.
e. Plasma metanephrine level.
88.A 30-year-old pregnant female patient (first trimester) developed palpitation,
tremors, excessive sweating, nervousness and failure to gain weight. Which of
the following investigations is the MOST accurate to diagnose her condition?
a. Free thyroxine level.
b. Level of thyroid peroxidase enzyme.
c. Technetium scan.
d. Thyroglobulin level.
e. Total thyroxine level.

P a g e 22 | 26
SurgiNotes 2nd Ed. (2022)

89.What is the hormone that acts on the intestines to increase calcium


absorption?
a. Calcitonin.
b. Corticotrophin releasing factor (CRF).
c. Pancreatic polypeptide.
d. Parathormone.
e. Thyroxine.

90.Regarding the anatomy of the thyroid gland, which of the following is TRUE?
a. In about 80% of persons, the recurrent laryngeal nerve traverses anterior
to the inferior thyroid artery.
b. The recurrent laryngeal nerve has an oblique course around the
subclavian artery on the left side.
c. The superior laryngeal nerve provides both sensory and motor function
to the larynx.
d. The thyroid gland is innervated only by parasympathetic fibers from the
vagus nerve.
e. Unilateral recurrent laryngeal nerve injury usually results in airway
compromise that necessitates tracheotomy.

91.Which of the following is the MOST potent stimulus for aldosterone secretion?
a. ACTH.
b. Antidiuretic hormone.
c. Hyperkalaemia.
d. Hypernatraemia.
e. Rennin angiotensin system.

92.Which of the following abnormalities indicates the possibility of


hyperaldosteronism?
a. Hyperkalemia, hyponatremia, hypochloremia.
b. Hyperkalemia, hypernatremia, low pH.
c. Hyperkalemia, hyponatremia, hyperglycemia.
d. Hypokalemia, hypernatremia, high pH.
e. Hypokalemia, hypochloremia, high pH.
P a g e 23 | 26
SurgiNotes 2nd Ed. (2022)

93.A 23-year-old woman undergoes total thyroidectomy for carcinoma of the


thyroid gland. On the second postoperative day, she begins to complain of
tingling sensation in her hands. She appears quite anxious and later complains
of muscle cramps. What is the initial therapy?
a. 10 ml of 10% magnesium sulfate intravenously.
b. 22-dihydrotachysterol orally.
c. Continuous infusion of calcium gluconate.
d. Oral calcium gluconate.
e. Oral vitamin D.

94.What is the recommended treatment of acute adrenal insufficiency?


a. Normal saline, potassium and glucose.
b. Hypertonic saline and potassium.
c. Normal saline and potassium.
d. Intravenous mineralocorticoids.
e. Normal saline, glucose and intravenous glucocorticoids.

95.A 50-year-old woman presents with lethargy, weight gain, cold intolerance
and loss of interest for the past six months. Which is the MOST appropriate
initial investigation?
a. Erythrocyte sedimentation rate estimation.
b. Radioactive isotope scan of thyroid.
c. Thyroid antibodies screen.
d. Thyroid stimulating hormone estimation.
e. Ultrasound scan of thyroid gland.

96.A 48-year-old woman with breast cancer diagnosed five years ago is admitted
with a two week history of back pain, nausea, fatigue and constipation. Her
serum calcium is 14 mg/dl. Which of the following is the correct first line
treatment for this patient?
a. Bisphosphonate infusion.
b. Calcitonin infusion.
c. Dextrose/insulin infusion.
d. Forced diuresis with furosemide.
e. Intravenous fluids.
P a g e 24 | 26
SurgiNotes 2nd Ed. (2022)

97.Which of the following is the MOST effective way in preventing surgical


infection?
a. Antibiotic prophylaxis.
b. Bowel preparation.
c. Drains and irrigation.
d. Skin preparation.
e. Surgical technique.

98.Which of the following is the MOST likely cause of pyrexia occurring 48 hours
after an abdominal operation?
a. Chest infection.
b. DVT.
c. Leaking intestinal anastomosis.
d. Pulmonary embolism.
e. Wound infection.

99.Which of the following is an anaerobic organism?


a. Bacteroides.
b. E. coli.
c. Klebsiella.
d. Pseudomonas aeruginosa.
e. Staphylococcus aureus.

100. A patient came with weakness in extension of hand and pronation. On


examination there is no wasting of hand muscles. Flexion of the wrist is
normal, biceps and triceps are normal. No brachioradialis jerk. Where is the
lesion?
a. Median nerve.
b. Radial nerve.
c. Ulnar nerve.
d. Musculocutaneous nerve.
e. Nerve to anterior interosseous.
P a g e 25 | 26
SurgiNotes 2nd Ed. (2022)

Egyptian Board Of General Surgery


1st Part Exam – Paper (1) MCQ "ANSWER"
June 2013

1 a 26 b 51 a 76 e
2 c 27 d 52 e 77 c
3 a 28 c 53 e 78 b
4 e 29 a 54 a 79 e
5 e 30 b 55 e 80 d
6 a 31 b 56 e 81 b
7 c 32 e 57 b 82 c
8 d 33 c 58 c 83 e
9 c 34 e 59 b 84 d
10 c 35 b 60 d 85 d
11 c 36 b 61 a 86 e
12 e 37 b 62 c 87 e
13 e 38 d 63 b 88 a
14 a 39 b 64 b 89 d
15 c 40 c 65 d 90 c
16 c 41 d 66 d 91 e
17 b 42 d 67 e 92 d
18 d 43 b 68 e 93 c
19 b 44 b 69 a 94 e
20 a 45 b 70 e 95 d
21 b 46 e 71 b 96 e
22 e 47 e 72 b 97 a
23 c 48 e 73 e 98 a
24 d 49 d 74 a 99 a
25 e 50 e 75 e 100 b

P a g e 26 | 26
SurgiNotes 2nd Ed. (2022)

Egyptian Board Of General Surgery


1st Part Exam – Paper (1) MCQ
March 2014
1. Which of the following is the main blood supply to the breast?
a. Acromiothoracic artery.
b. Intercostal arteries.
c. Internal thoracic artery.
d. Lateral thoracic artery.
e. Superior thoracic artery.

2. Taste from the posterior one third of tongue is provided by which of the
following nerves?
a. Facial.
b. Glossopharyngeal.
c. Hypoglossal.
d. Lingual.
e. Vagus.

3. Which of the following nerves is MOST likely to be injured following fracture


shaft of the humerus?
a. Medial cord of the brachial plexus.
b. Median nerve.
c. Musculocutaneous nerve.
d. Radial nerve.
e. Ulnar nerve.

4. What is the site of drainage of the inferior thyroid veins?


a. External jugular vein.
b. Innominate vein.
c. Internal jugular vein.
d. Subclavian vein.
e. Vertebral vein.
P a g e 1 | 26
SurgiNotes 2nd Ed. (2022)

5. Which of the following nerves is liable to be injured during opening of the


inguinal canal in herniorrhaphy operation?
a. Iliohypogastric nerve.
b. Obturator nerve.
c. Lateral femoral cutaneous nerve.
d. Ilioinguinal nerve.
e. Pudendal nerve.
6. A 25-year-old male patient is involved in a serious car accident. Examination
reveals that his right leg is shortened and internally rotated. The patient is
unable to dorsiflex or plantarflex his foot. All sensations below the knee are
lost apart from the medial side of the leg and foot and upper back of the calf.
Which of the following is the MOST likely injured nerve?
a. Common peroneal nerve.
b. Tibial nerve.
c. Obturator nerve.
d. Sciatic nerve.
e. Femoral nerve.
7. A 20-year-old man presented to the emergency department with a stab injury
to the anterior aspect of the right wrist. On examination he was found to have
a deep 2-сm long transverse laceration at the front of the right wrist with loss
of sensation in the ring and little fingers. The patient was unable to abduct and
adduct his fingers on the affected side. Radial pulse was palpable. What was
the MOST likely structure to have been injured?
a. Anterior interosseous nerve.
b. Median nerve.
c. Musculocutaneous nerve.
d. Radial nerve.
e. Ulnar nerve.
8. During the surgical repair of a perforated duodenal ulcer in a 47-year-old male
patient, the gastroduodenal artery is ligated. A branch of which of the
following arteries will continue to supply blood to the pancreas in this patient?
a. Inferior mesenteric.
b. Left gastric.
c. Right gastric.
d. Proper hepatic.
e. Superior mesenteric.
P a g e 2 | 26
SurgiNotes 2nd Ed. (2022)

9. A 19-year-old woman attends the emergency department after being shot in


the neck with an air-gun pellet. In the course of surgical exploration of the
posterior triangle, a nerve is injured. The patient is unable to shrug her left
shoulder or fully abduct her left arm. Which is the MOST likely structure to
have been injured?
a. Axillary nerve.
b. Dorsal scapular nerve.
c. Long thoracic nerve.
d. Spinal accessory nerve.
e. Suprascapular nerve.

10.A 34-year-old man is seen in the emergency department with a fibular fracture
following a football match. On examination he is noted to have loss of foot
eversion. Which area of skin should be examined to confirm loss of the
cutaneous distribution of the affected nerve?
a. Along the lateral aspect of the foot.
b. Along the medial aspect of the foot.
c. Between the hallux and the second digit.
d. On the dorsal surface of the foot.
e. On the plantar surface of the foot.

11.Which of the following is the sensory supply to the skin of the tip of the index
finger?
a. Radial nerve only.
b. Radial and median nerves.
c. Median and ulnar nerves.
d. Ulnar nerve only.
e. Median nerve only.

12.Which one of the following structures provides the greatest support for the
liver?
a. The falciform ligament.
b. The coronary ligament.
c. The ligamentum teres.
d. The hepatic veins joining the inferior vena cava.
e. The ligamentum venosum.
P a g e 3 | 26
SurgiNotes 2nd Ed. (2022)

13. A 65-year-old man presents with haematuria and right loin pain. Computerized
tomography demonstrates a right renal tumour and he undergoes a right radical
nephrectomy. The right testicular vein drains into which of the following?
a. Inferior mesenteric vein.
b. Inferior vena cava.
c. Right adrenal vein.
d. Right lumbar vein.
e. Right renal vein.

14. A 62-year-old man is admitted to the emergency department due to retention of


urine. Physical examination reveals prostatic hypertrophy. After several
unsuccessful attempts to catheterize the penile urethra, the urologist orders
drainage of the urinary bladder by the least invasive procedure, avoiding entry
into the peritoneal cavity or the injury of any major vessels or organs. Which of
the following spaces needs to be traversed by the needle to reach the bladder?
a. Ischioanal fossa.
b. Perineal body.
c. Retropubic space (of Retzius).
d. Superficial perineal cleft.
e. Deep perineal pouch.

15. A 20-year-old male while walking slipped on the wet pavement and injured his
right arm. Radiographic images showed a midshaft fracture of the humerus.
Which pair of structures was MOST likely injured at the fracture site?
a. Median nerve and brachial artery.
b. Axillary nerve and posterior humeral circumflex artery.
c. Radial nerve and deep brachial artery.
d. Suprascapular nerve and artery.
e. Long thoracic nerve and lateral thoracic artery.

16. A 29-year-old female is examined in the emergency department after falling from
her balcony. Radiographic examination reveals that she has suffered a broken
clavicle, with associated internal bleeding. Which of the following vessels is
MOST likely to be injured in clavicular fractures?
a. Subclavian artery.
b. Cephalic vein.
c. Lateral thoracic artery.
d. Subclavian vein.
e. Internal thoracic artery.
P a g e 4 | 26
SurgiNotes 2nd Ed. (2022)

17.Which of the following tests will be affected after intake of non-steroidal anti-
inflammatory drugs?
a. Bleeding time.
b. Coagulation time.
c. INR.
d. PTT.
e. Thrombin time.

18.Which of the following is a function of the parasympathetic system?


a. Increase the blood supply of the heart.
b. Increase the blood supply of skeletal muscles.
c. C. Increase in the heart rate.
d. Stimulation of glycogenolysis.
e. Increased motility of the gastrointestinal tract.

19.A 48-year-old woman presents to the emergency department with persistent


diarrhea. On examination, she is apyrexial and has an irregularly irregular
pulse of 140 beats/minute. The MOST likely cause of the abnormal cardiac
rhythms is:
a. Hypernatraemia.
b. Hypokalaemia.
c. Hypovolaemia.
d. Myocardial ischaemia.
e. Sepsis.

20.Stress related hyperglycemia is thought to be due to increased release of all of


the following EXCEPT:
a. Glucocorticoids.
b. Growth hormone.
c. Thyroid-stimulating hormone (TSH).
d. Glucagon.
e. Epinephrine.
P a g e 5 | 26
SurgiNotes 2nd Ed. (2022)

21.All of the following activate the sympathoadrenal and hypothalamic-pituitary


axes during stress or injury EXCEPT:
a. Pain.
b. Hypovolemia.
c. Acidosis.
d. Hypercapnia.
e. Acetylcholine.

22.Which of the following are effects of epinephrine in response to injury?


a. It enhances the adherence of leukocytes to vascular endothelial
membranes.
b. It stimulates the release of aldosterone.
c. It inhibits the secretion of thyroid hormones.
d. It increases glucagon secretion.
e. It decreases lipolysis in adipose tissue.

23.With regard to potassium, which of the following statements is NOT true?


a. Normal dietary intake of potassium is 50 to 100 mEq/day.
b. In patients with normal renal function, most ingested potassium is
excreted in urine.
c. More than 90% of the potassium in the body is located in the extracellular
compartment.
d. Critical hyperkalemia (> 6 mEq/L) is rarely encountered if renal function
is normal.
e. Administration of sodium bicarbonate shifts potassium from the
extracellular space (ECF) to the intracellular space (ICP).

24.Norepinephrine (noradrenaline) causes an increase in the systemic vascular


resistance. In which condition, its use is MOST appropriate:
a. Cardiogenic shock.
b. Hypovolemic shock.
c. Neurogenic shock.
d. Obstructive shock (e.g. pulmonary embolism).
e. Septic shock.
P a g e 6 | 26
SurgiNotes 2nd Ed. (2022)

25.A 60-year-old woman had a total pancreatectomy three months ago. She now
complains of offensive, foul smelling stools that are difficult to flush. Lack of
which enzyme is responsible?
a. Amylase.
b. Cholecystokinin.
c. Lipase.
d. Secretin.
e. Trypsin.

26.Respiratory alkalosis can occur as a result of:


a. Asphyxia.
b. Asthma.
c. Severe emphysema.
d. Hyperventilation.
e. Hypoventilation.

27.Anti-diuretic hormone (ADH) is secreted in response to shock and remains


elevated for approximately 1 week. Which of the following is seen as a result
of this increased level of ADH?
a. Decreased water permeability in the distal tubules.
b. Increased sodium loss in the distal tubules.
c. Mesenteric vasoconstriction.
d. Mesenteric vasodilatation.
e. Increased hepatic glycolysis.

28.Pancreatic acini secrete enzyme that play an important role in the digestion of
proteins. What is the MOST potent stimulant of pancreatic acinar cells?
a. Acetylcholine.
b. Cholecystokinin (CCK).
c. Peptide.
d. Secretin.
e. Trypsinogen.
P a g e 7 | 26
SurgiNotes 2nd Ed. (2022)

29.Which of the following is the major stimulus for aldosterone secretion?


a. ACTH.
b. Angiotensin II.
c. Cortisol.
d. Hypernatraemia.
e. Hypokalaemia.

30.Which of the following cells is responsible for the secretion of pepsinogen?


a. Chief cells.
b. Enterochromaffin cells.
c. Mucous cells.
d. Paneth cells.
e. Parietal cells.

31.Which of the following is the MOST potent stimulus for contraction of the gall
bladder?
a. Acetylcholine.
b. Cholecystokinin.
c. Intravenous hyperalimentation.
d. Secretin.
e. Vagal stimulation.

32.A 45-year-old female patient complains of reflux oesophagitis secondary to a


sliding hiatus hernia. Which of the following decreases the tone of the lower
esophageal sphincter?
a. Fatty meal.
b. Gastric alkalinization.
c. Gastric distension.
d. Gastrin.
e. Proteins.
P a g e 8 | 26
SurgiNotes 2nd Ed. (2022)

33.A 40-year-old male patient was injured in the thigh and a bleeding point was
ligated by a silk suture. The patient got recurrent attacks of inflammation at
the site of the wound. The surgeon explored the wound and removed an
inflammatory reaction mass around the silk suture. Which of the following
cells will be the predominant cell detected in this mass?
a. Basophils.
b. Eosinophils.
c. Lymphocyte.
d. Macrophages.
e. Neutrophils.

34.Which of the following cells is an example of a permanent cell NOT capable of


division?
a. Acinar cells of the pancreas.
b. Colonic mucosal cells.
c. Erythrocytes.
d. Hepatocytes.
e. Osteocytes.

35.Which of the following microscopic features MOST likely indicates that a


neoplasm is malignant?
a. Atypia.
b. Increased nuclear : cytoplasmic ratio.
c. Invasion.
d. Necrosis.
e. Pleomorphism.

36.Which of the following can minimize effects of steroids on wound healing?


a. Copper.
b. Vitamin A.
c. Vitamin C.
d. Vitamin D.
e. Vitamin E.
P a g e 9 | 26
SurgiNotes 2nd Ed. (2022)

37.A 45-year-old woman is admitted with a chronic cough and weight loss. On
examination, crepitations are heard over the apex of the right lung. A chest X-
ray reveals cavitation in the right apex. What is a lung biopsy MOST likely to
show?
a. Abundant plasma cells.
b. Aggregates of activated macrophages.
c. Eosinophils.
d. Mast cells.
e. Neutrophilic infiltrates with abscess formation.

38.Which of the following is an anaerobic organism?


a. Bacteroides.
b. E. coli.
c. Klebsiella.
d. Pseudomonas aeruginosa.
e. Staphylococcus aureus.

39.A 40-year-old male patient had renal transplantation and is receiving


immunosuppression. Few days following the operation the patient developed
persistent fever. Which of the following infectious microorganisms is currently
the MOST likely cause of this fever?
a. Candidiasis.
b. E. coli.
c. Cytomegalovirus.
d. Pneumococci.
e. Streptococci.

40.Which of the following is the mode of action of amikacin?


a. Blockage of bacterial DNA replication.
b. Competitive inhibition of the enzyme dihydrofolate reductase.
c. Inhibition of bacterial wall synthesis.
d. Inhibition of protein synthesis by ribosomes.
e. Inhibition of translocation of peptides.
P a g e 10 | 26
SurgiNotes 2nd Ed. (2022)

41.Which of the following organisms is the commonest to cause urinary tract


infection following catheterization?
a. Clostridium difficile.
b. E. coli.
c. Pseudomonas aeruginosa.
d. Staph. aureus.
e. Strept. pneumonia.

42.Which of the following has the greatest influence for impaired healing of a
wound?
a. Anaemia.
b. Diabetes mellitus.
c. Intake of corticosteroids.
d. Local wound infection.
e. Malnutrition.

43.A 30-year-old woman sustained a puncture wound to the foot. The patient has
been on a therapeutic dose of steroids for the past 5 years for ulcerative colitis.
Her last tetanus toxoid booster was 8 years ago. What should the patient
receive?
a. Tetanus toxoid booster.
b. Human immunoglobulin, antibiotics with anaerobic coverage.
c. Tetanus toxoid plus human immunoglobulin.
d. Tetanus toxoid plus human immunoglobulin and antibiotics with aerobic
and anaerobic coverage.
e. Wide debridement of the wound.

44.The clinical course of the majority of patients with HCV infection is


characterized by which one of the following?
a. Acute constitutional symptoms and jaundice.
b. Acute fulminant hepatic failure.
c. Development of chronic hepatitis.
d. Progression to cirrhosis.
e. Development of hepatocellular carcinoma.
P a g e 11 | 26
SurgiNotes 2nd Ed. (2022)

45.Which of the following statements about the treatment of necrotizing fasciitis


is TRUE?
a. The underlying tissue necrosis is reflected by the extent of skin necrosis.
b. Intravenous immune globulin (IVIG) is the first line of therapy.
c. Penicillin provides sufficient antibiotic coverage.
d. Hyperbaric oxygen has been shown to improve survival.
e. Exploratory incisions over normal-appearing skin are effective in
determining the extent of the necrosis.

46.Which of the following statements about the usage of antibiotics for the
prevention of surgical site infection is NOT true?
a. Antibiotics should be administered within 60 minutes of incision.
b. The therapeutic dose of the antibiotic should be administered
intravenously.
c. Adequate tissue concentrations should be maintained during operation
by re-dosing as necessary.
d. Antibiotics should be continued for 48 hours after operation.
e. Vancomycin is the agent of choice for patients from nursing homes
undergoing hip replacement.

47.The earliest manifestations of serious gram-negative infection may consist of


a triad of signs that includes:
a. Tachypnea, hypotension and an altered sensorium.
b. Tachypnea, hypotension and lactic acidosis.
c. Thrombocytopenia, hypotension and lactic acidosis
d. Mild hyperventilation, respiratory alkalosis and an altered sensorium.
e. Tachycardia, hypotension and metabolic acidosis.

48.A 24-year-old HIV positive man presents to the emergency room with acute
onset pain and redness in his scrotum, penis and perineum. Upon
examination, you feel crepitus over the erythematous area described, which
emits a foul-smelling grey discharge. What is the MOST appropriate
management for this patient's illness?
a. Apply topical polymycin ointment.
b. Give hydrocortisone infusion.
c. Initiate highly active antiretroviral treatment.
d. Initiate penicillin G infusion.
e. Surgical debridement of affected tissue.
P a g e 12 | 26
SurgiNotes 2nd Ed. (2022)

49.After being struck by a moving truck, a 23-year-old woman undergoes


splenectomy during diagnostic laparotomy. She leaves the hospital against
medical advice on postoperative day 4, after refusing vaccination. Infection
with which of the following organisms is MOST likely to result in her
developing sepsis?
a. Beta-hemolytic streptococcus.
b. Candida albicans.
c. Clostridium difficile.
d. Escherichia coli.
e. Pseudomonas aeruginosa.

50.Severe sepsis is differentiated from sepsis by:


a. A history of premorbid conditions such as diabetes.
b. Positive blood culture for bacteria and fungus.
c. Acute organ failure such as renal insufficiency.
d. Prolonged arterial hypotension.
e. Temperature more than 38.5 °C.

51.Septic shock is characterized by:


a. Increased capillary permeability.
b. Vasoconstriction
c. A low cardiac output.
d. A high systemic vascular resistance.
e. Bradycardia.

52.A blue-green discharge from an ulcer indicates infection with:


a. Candida albicans.
b. Haemophilus influenza.
c. Pseudomonas pyocyaneus.
d. Staphylococcus aureus.
e. Streptococcus viridans.
P a g e 13 | 26
SurgiNotes 2nd Ed. (2022)

53.A 30-year-old female patient developed postoperative wound infection after


a thyroidectomy operation. What is the MOST likely causative organism?
a. Bacteroides.
b. E. coli.
c. Proteus.
d. Staph. aureus.
e. Streptococci.

54.Which of the following is the mode of action of ciprofloxacin?


a. Blockage of bacterial DNA replication.
b. Competitive inhibition of the enzyme dihydrofolate reductase.
c. Inhibition of bacterial wall synthesis.
d. Inhibition of protein synthesis by ribosomes.
e. Inhibition of translocation of Peptides.

55.Which of the following measures is MOST likely to reduce the risk of


postoperative wound infection with MRSA?
a. 5 days of broad spectrum prophylactic antibiotics.
b. Ensure that the patient showers with chlorhexidine wash prior to surgery.
c. A policy of staff handwashing between patients.
d. Screening patients for MRSA carriage prior to surgery.
e. Preoperative shaving the area of incision.

56.A 45-year-old male patient had renal transplantation operation. Thirty


minutes after the operation the urinary output markedly dropped. The blood
pressure of was 120/80 mmHg. Duplex scanning of the transplanted kidney
revealed patent renal vessels and normal pelvicalyceal system. Which of the
following is TRUE regarding the present problem?
a. It is less liable to occur in females who had repeated pregnancies.
b. It is more liable to occur in patients who receive liver transplantation.
c. Stimulated CD4 lymphocytes are mainly responsible for the problem.
d. The patient needs immediate removal of the transplanted kidney.
e. The problem can be corrected by increasing the dose of cyclosporin.
P a g e 14 | 26
SurgiNotes 2nd Ed. (2022)

57.Which of the following is a potential sequela of cytomegalic virus infection?


a. Cholecystitis.
b. Intra-abdominal abscess.
c. Pancreatitis.
d. Parotitis.
e. Pyelonephritis.

58.Which of the following cancers is MOST common in organ transplant


recipients?
a. Bronchogenic carcinoma.
b. Colon cancer.
c. Pancreatic cancer.
d. Prostatic cancer.
e. Skin cancer.

59.What is the single MOST important factor in determining whether to perform


a transplant between a specific donor and recipient?
a. ABO blood types of the donor and recipient.
b. Closeness of relationship between the donor and recipient.
c. HLA types of the donor and recipient.
d. Mixed lymphocyte culture assays of the donor and recipient.
e. Peripheral T-cell count of the recipient.

60.A 40-year-old male patient underwent a renal transplant from a cadaveric


donor. Immediately after finishing the vascular anastomosis, the kidney
became cyanotic and flaccid. Histological examination revealed depositions of
immunoglobulins and complement in the vessels walls. What is the
immunological background of the problem?
a. Donor cytotoxic T-lymphocytes directed against the host antigens.
b. Donor natural killer cells directed against host antigens.
c. Host natural killer cells against donor antigens.
d. Preformed donor antibodies against the host antigens.
e. Preformed host antibodies against the donor antigens.
P a g e 15 | 26
SurgiNotes 2nd Ed. (2022)

61.A patient is reported to have a potassium level of 7.3 mEq/L. What is the first
diagnostic or therapeutic maneuver for this patient?
a. An ECG and administration of 1 g of 10% calcium gluconate.
b. An ECG-and infusion of 45 mEq/L NaHCO3.
c. An ECG and infusion of glucose and insulin intravenously.
d. Kayexalate enema.
e. Transfer to an intensive care unit for careful cardiac monitoring.

62.The arterial blood gas analysis of a patient was pH 7.5, PCO2 47 mmHg and
HCO3- 35 mmol/L. This patient MOST likely is suffering from:
a. Chronic obstructive pulmonary disease.
b. Diabetic ketoacidosis.
c. Persistent diarrhea.
d. Profound vomiting.
e. Salicylate poisoning.

63.Five days after uneventful cholecystectomy, asymptomatic middle aged


woman is found to have a serum sodium level of 120 mEq/L. What is the
proper management?
a. Administration of hypertonic saline solution.
b. by Administration of Ringer's lactate solution.
c. Aggressive diuresis with furosemide.
d. Plasma ultra-filtration.
e. Restriction of free water.

64.What is the MOST common fluid disorder in the surgical patient?


a. Extracellular fluid deficit.
b. Hyperkalemia.
c. Hyponatremia.
d. Metabolic acidosis.
e. Metabolic alkalosis.
P a g e 16 | 26
SurgiNotes 2nd Ed. (2022)

65.The osmolarity of the extracellular fluid space is determined primary by the


concentration of:
a. Bicarbonate.
b. Chloride ions.
c. Phosphate radicals.
d. Potassium ions.
e. Sodium ions.

66.In the presence of acute blood loss, what is the initial mechanism to maintain
an adequate preload to the heart?
a. Development of tachycardia.
b. Hormonal effects of angiotensin.
c. Hormonal effects of rennin.
d. Increase in systemic vascular resistance.
e. Increased cortisol secretion.

67.Which of the following is characteristic of neurogenic shock?


a. Cool moist skin.
b. Decreased blood volume.
c. Increased peripheral vascular resistance.
d. Increased cardiac output.
e. Bradycardia.

68.What is the MOST common symptom after major pulmonary embolism?


a. Cough.
b. Dyspnea.
c. Hemoptysis.
d. Pleural pain.
e. Palpitation.
P a g e 17 | 26
SurgiNotes 2nd Ed. (2022)

69.A 60-year-old male patient had a colectomy operation. Post-operatively the


patient had tachycardia, hypertension and shallow respiration. Which of the
following is an indication to intubate the patient?
a. PaCO2 of 45 mmHg.
b. Respiratory rate 25/min.
c. PO2 of 55 mmHg at room temperature.
d. Heart rate of 130/min.
e. High pulmonary capillary wedge pressure.

70.Which of the following is the commonest cause of ARDS?


a. Sepsis syndrome.
b. Aspiration.
c. Acute pancreatitis.
d. DIC.
e. Fat embolism.

71.Which of the following characterizes protein metabolism during trauma?


a. Decreased liver gluconeogenesis.
b. Inhibition of skeletal muscle breakdown by interleukin-1 and tumor
necrosis factor (TNF, cachectin).
c. Decreased urinary nitrogen loss.
d. Hepatic synthesis of acute-phase reactants.
e. Decreased glutamine consumption by fibroblasts, lymphocytes and
intestinal epithelial cells.

72.What is the correct management of the commonest acid-base imbalance seen


in long standing or severe hemorrhagic shock?
a. Intravenous sodium bicarbonate.
b. Component blood therapy.
c. Increased fluid administration.
d. Vasopressors.
e. Hyperventilation.
P a g e 18 | 26
SurgiNotes 2nd Ed. (2022)

73. A 38-year-old man in end-stage renal failure resulting from polycystic kidney disease
receives a cadaveric renal transplant. Good renal function is established but four
weeks later deteriorates, the serum creatinine rising by 25%. Which of the following
processes is MOST likely to be responsible for this deterioration?
a. B-cell mediated rejection.
b. Circulating immune complex disease.
c. IgG antibody mediated rejection.
d. Post-transplant lymphoproliferative disorder.
e. T-cell mediated rejection.
74. A 78-year-old woman with emphysema receiving 28% oxygen by mask has the
following blood gas results:
PH PO2 PaCO2 Bicarbonate Base excess
Finding: 7.28 70 mmHg 48 mmHg 36 mmol/L +5
Normal: 7.35-7.45 90-110 35-45 22-26 -2 to +2
The MOST likely interpretation is:
a. Mixed respiratory and metabolic acidosis.
b. Partially compensated metabolic acidosis.
c. Partially compensated respiratory acidosis.
d. Uncompensated metabolic acidosis.
e. Uncompensated respiratory acidosis.
75. A 28-year-old motorcyclist is admitted following a road traffic accident, having
sustained bilateral femoral fractures and a ruptured spleen. Three days
postoperatively he is noted to be agitated, hypoxaemic and difficult to ventilate. His
observations show a blood pressure of 120/80 mmHg, regular pulse of 88
beats/minute and he is apyrexial. A chest X-ray shows bilateral diffuse lung
infiltrates. What is the MOST likely underlying diagnosis?
a. Adult respiratory distress syndrome (ARDS).
b. Atelectasis.
c. Bronchopneumonia.
d. Pulmonary oedema.
e. Pulmonary thrombo-embolism.
76. A 22-year-old man involved in a motor vehicle accident is found to have a thoracic
spine fracture (T6) and paraplegia. The patient is hypotensive with systolic BP of 70
mmHg, is bradycardiac with a pulse of 48 beats/min and is breathing comfortably.
Which of the following would the MOST appropriate initial treatment?
a. Isotonic fluid administration.
b. Steroid administration within 24 hours of the injury.
c. Immediate intubation.
d. Alpha-agonist administration.
e. Immediate magnetic resonance imaging.
P a g e 19 | 26
SurgiNotes 2nd Ed. (2022)

77.What is the primary source of calories during prolonged starvation (> 5 days
fasting)?
a. Skeletal muscle proteins.
b. Body stored fat.
c. Liver glycogen.
d. Ketone bodies.
e. Muscle glycogen.

78.A 40-year-old man is confused and restless the second day after upper
abdominal surgery and repair of a hiatus hernia. What is the MOST probable
cause of his condition?
a. Pulmonary embolism.
b. Narcotic overdose.
c. Pulmonary atelectasis.
d. Electrolyte imbalance.
e. Starvation ketosis.

79.Which of the following techniques does NOT provide a definitive airway?


a. Cricothyroidotomy.
b. Tracheostomy.
c. Nasotracheal tube.
d. Laryngeal mask airway.
e. Endotracheal tube.

80.Which of the following is a contraindication to heparin therapy?


a. Closed head injury two weeks ago.
b. Heparin-induced thrombocytopenia.
c. Subclavian vein thrombosis.
d. Superior mesenteric artery embolism.
e. Third trimester of pregnancy.
P a g e 20 | 26
SurgiNotes 2nd Ed. (2022)

81.A 30-year-old male patient was admitted to the casualty department due to a
car accident. The patient had fracture of the pelvis and the right femur and he
received 5 liters of blood following which he started to have bleeding from his
nose and mouth. What is the main cause of this bleeding tendency?
a. Decrease in fibrinogen.
b. Decrease in prothrombin.
c. Decrease of calcium.
d. Increased fibrinolytic activity.
e. Platelet depletion.

82.What is the main mode of action of heparin?


a. It increases the level of protein C.
b. It is a cofactor of antithrombin III.
c. It prevents clot retraction.
d. It prevents platelets aggregation.
e. It prevents the synthesis of fibrinogen.

83.What is the principal mechanism of the antithrombotic action of administering


aspirin in low dose?
a. Induction of capillary vasodilatation.
b. Induction of endothelial cell prostacyclin production.
c. Induction of endothelial heparin production.
d. Inhibition of factor V production by the liver.
e. Inhibition of platelet production of thromboxane A2.

84.You were obliged to perform cholecystectomy for a cirrhotic patient. During


surgery there was excessive bleeding. What would you order?
a. Fresh blood.
b. Fresh frozen plasma.
c. Fresh platelets.
d. IV vitamin K.
e. IV factor VIII.
P a g e 21 | 26
SurgiNotes 2nd Ed. (2022)

85.A fully heparinized patient develops a condition requiring emergency surgery.


After stopping the heparin, what else should be done to prepare the patient?
a. 2 units of cryoprecipitates.
b. Administration of protamine sulphate 1 mg for every 100 units of heparin
most recently administered.
c. Immediate fresh frozen plasma.
d. Transfusion of 10 units of platelet.
e. Vitamin K intravenously.
86.A 5-year-old boy slipped and hurted his right knee while walking. He presents
with a tender, swollen, warm knee with significant hemarthrosis. His PT is 12
(normal 13 seconds), PTT is over 100 (normal 25 seconds), platelet count is
300,000/mm3 and bleeding time is normal. Initial management should consist
of which of the following?
a. Fresh-frozen plasma.
b. Aspiration of knee.
c. Factor VIII concentrate.
d. Passive exercise.
e. Long-leg cast.
87.A 50-year-old female patient has chronic renal failure and has been
maintained an chronic dialysis. The patient underwent cholecystectomy. Post-
operatively she had severe bleeding. What is the MOST likely cause for this
bleeding?
a. Elevated PT.
b. Elevated PTT.
c. Low platelet count.
d. Decreased platelet aggregation.
e. Sepsis.
88.A mother mentions that her 8-year-old boy gets recurrent attacks of
hemarthrosis following a minimal trauma. Investigations revealed normal
platelet count and prothrombin time, but the partial thromboplastin time is
prolonged. Which of the following statements regarding this clinical condition
is TRUE?
a. There is an underlying liver problem.
b. The sisters of this boy are usually having the same problem.
c. he boy has had repeated episodes of epistaxis.
d. There is no family history in this condition.
e. Transfusion of factor VIII concentrate is helpful.
P a g e 22 | 26
SurgiNotes 2nd Ed. (2022)

89.In the awake, non-anesthetized patient suspected of having a hemolytic post-


transfusion reaction, the MOST characteristic signs are:
a. Nausea and vomiting.
b. Fever and chills.
c. Oliguria and hemoglobinuria.
d. Cyanosis and dyspnea.
e. Tenderness of the renal angle.

90.What is the hormone that acts on the intestines to increase calcium


absorption?
a. Calcitonin.
b. Corticotrophin releasing factor (CRF).
c. Pancreatic polypeptide.
d. Parathormone.
e. Thyroxine.

91.A 30-year-old female patient complains of fatigue, generalized bony aches and
depression. The patient gives a history of 2 previous operations for urinary
stones. Which of the following laboratory tests is MOST accurate in the
diagnosis?
a. 24 hours urinary calcium.
b. Ionizable serum calcium.
c. Total serum calcium.
d. Plasma chloride.
e. Serum phosphate.

92.Which of the following abnormalities indicates the possibility of


hyperaldosteronism?
a. Hyperkalemia, hyponatremia, hypochloremia.
b. Hyperkalemia, hypernatremia, low pH.
c. Hyperkalemia, hyponatremia, hyperglycemia.
d. Hypokalemia, hypernatremia, high pH.
e. Hypokalemia, hypochloremia, high pH.
P a g e 23 | 26
SurgiNotes 2nd Ed. (2022)

93.A 23-year-old woman undergoes total thyroidectomy for carcinoma of the


thyroid gland. On the second postoperative day, she begins to complain of
tingling sensation in her hands. She appears quite anxious and later complains
of muscle cramps. What is the initial therapy?
a. 10 ml of 10% magnesium sulfate intravenously.
b. 22-dihydrotachysterol orally.
c. Intravenous infusion of calcium gluconate.
d. Oral calcium gluconate.
e. Oral vitamin D.

94.A 55-year-old female patient who has metastatic breast cancer presents with
weakness, anorexia, malaise, constipation and back pain and lethargy.
Laboratory studies include a normal chest X-ray; serum albumin 3.2 mg/dl;
serum calcium 14 mg/dl; serum phosphorus 2.6 mg/dl; serum chloride 108
mg/dl; BUN 32 mg/dl and creatinine 2.0 mg/dl. What is the appropriate initial
management?
a. Intravenous normal saline infusion.
b. Administration of thiazide diuretics.
c. Administration of intravenous phosphorus.
d. Use of mithramycin.
e. Neck exploration and parathyroidectomy.

95.What is the recommended treatment of acute adrenal insufficiency?


a. Normal saline, potassium and glucose.
b. Hypertonic saline and potassium.
c. Normal saline and potassium.
d. Intravenous mineralocorticoids.
e. Normal saline, glucose and intravenous glucocorticoids.

96.A 45-year-old woman is found to be hypertensive. Further blood testing


reveals serum sodium 145 mmol/L, serum potassium 2.8 mmol/L and serum
and bicarbonate 30 mmol/L. The MOST likely cause for her symptoms is over
secretion of which of the following?
a. ACTH.
b. Adrenaline.
c. Aldosterone.
d. Cortisol.
e. Norepinephrine (noradrenaline).
P a g e 24 | 26
SurgiNotes 2nd Ed. (2022)

97. Which of the following is the MOST effective way in preventing surgical infection?
a. Antibiotic prophylaxis.
b. Bowel preparation.
c. Drains and irrigation.
d. Skin preparation.
e. Surgical technique.

98. A 28-year-old motorcyclist is admitted following a road traffic accident, having


sustained bilateral femoral fractures and a ruptured spleen. Three days
postoperatively he is noted to be agitated, hypoxaemic and difficult to ventilate.
His observations show a blood pressure of 120/80 mmHg, regular pulse of 88
beats/minute and he is apyrexial. A chest X-ray shows bilateral diffuse lung
infiltrates. What is the MOST likely underlying diagnosis?
a. Adult respiratory distress syndrome (ARDS).
b. Atelectasis.
c. Bronchopneumonia.
d. Pulmonary oedema.
e. Pulmonary thrombo-embolism.

99. A 22-year-old man involved in a motor vehicle accident is found to have a thoracic
spine fracture (T6) and paraplegia. The patient is hypotensive with a systolic BP
of 70 mmHg, is bradycardiac with a pulse of 48 beats/min and is breathing
comfortably. Which of the following would the MOST appropriate initial
treatment?
a. Isotonic fluid administration.
b. Steroid administration within 24 hours of the injury.
c. Immediate intubation.
d. Alpha-agonist administration.
e. Immediate magnetic resonance imaging.

100. A 26-year-old man with history of chronic duodenal ulcer presents with
repeated vomiting for 2 days. Insertion of Foley’s catheter in yields 130 ml of
concentrated urine. Serum electrolytes showed sodium 128 mEq/L, potassium
2.9 mEq/L and chloride 64 mEq/L. Blood pH is 7.53. The MOST appropriate initial
management is:
a. 5% dextrose in 0.45 normal saline.
b. 0.45 normal saline plus potassium chloride 60 mEq/L.
c. Lactated Ringer's solution plus potassium chloride 60 mEq/L.
d. Ammonium chloride to correct blood pH.
e. IV furosemide (lasix) to correct urine output.
P a g e 25 | 26
SurgiNotes 2nd Ed. (2022)

Egyptian Board Of General Surgery


1st Part Exam – Paper (1) MCQ "ANSWER"
March 2014

1 c 26 d 51 a 76 a
2 b 27 c 52 c 77 d
3 d 28 b 53 d 78 c
4 b 29 b 54 a 79 d
5 d 30 a 55 c 80 b
6 d 31 b 56 c 81 e
7 e 32 a 57 c 82 b
8 e 33 d 58 e 83 e
9 d 34 c 59 a 84 b
10 d 35 c 60 e 85 b
11 e 36 b 61 a 86 c
12 d 37 b 62 d 87 d
13 b 38 a 63 e 88 e
14 c 39 c 64 a 89 b
15 c 40 d 65 e 90 d
16 d 41 b 66 d 91 b
17 a 42 d 67 e 92 d
18 e 43 d 68 b 93 c
19 b 44 c 69 c 94 a
20 c 45 e 70 a 95 e
21 e 46 d 71 d 96 c
22 d 47 d 72 c 97 a
23 c 48 e 73 e 98 a
24 e 49 a 74 c 99 a
25 c 50 c 75 a 100 c

P a g e 26 | 26
SurgiNotes 2nd Ed. (2022)

Egyptian Board Of General Surgery


1st Part Exam – Paper (1) MCQ
March 2015
1. A 30-year-old male patient had a road traffic accident. Examination revealed loss of
contour of the shoulder and the arm was held in an adducted position. The patient could
not move his shoulder. Which of the following nerves was MOST likely injured in this
patient?
a. Axillary (circumflex).
b. Median.
c. Musculocutaneous.
d. Radial.
e. Ulnar.
2. Which of the following nerves is liable to be injured during opening of the inguinal canal
in herniorrhaphy operation?
a. Iliohypogastric nerve.
b. Obturator nerve.
c. Lateral femoral cutaneous nerve.
d. Ilioinguinal nerve.
e. Pudendal nerve.
3. A 25-year-old male patient is involved in a serious car accident. Examination reveals
that his right leg is shortened and internally rotated. The patient is unable to dorsiflex
or planter flex his foot. All sensations below the knee are lost apart from the medial
side of the leg and foot and upper back of the calf. Which of the following is the MOST
likely injured nerve?
a. Common peroneal nerve.
b. Tibial nerve.
c. Obturator nerve.
d. Sciatic nerve.
e. Femoral nerve.
4. A 40-year-old male patient was stabbed in the right gluteal region. The wound was
sutured in the casualty department and the patient discharged. The patient noted that
his gait was not proper. Exam revealed a waddling gait and Trendelenburg’s test was
positive. What was the injured nerve?
a. Superior gluteal nerve.
b. Nerve to quadratus femoris.
c. Inferior gluteal nerve.
d. Nerve to obturator externus.
e. Obturator nerve.
P a g e 1 | 26
SurgiNotes 2nd Ed. (2022)

5. A 65-year-old man presents with haematuria and left loin pain. Computerized
tomography confirms a left renal tumour. The normal left renal hilum lies at
which vertebral level?
a. T11.
b. T12.
c. L1.
d. L2.
e. L3.

6. A 74-year-old woman presents to the outpatient clinic with pelvic pain. A CT


scan reveals enlarged para-aortic lymph nodes. These are MOST likely to be
involved in secondary spread from a tumour in which of the following organs?
a. Cervix.
b. Ovary.
c. Uterus.
d. Vagina.
e. Vulva.

7. A 64-year-old woman is reviewed in the emergency department with an acute


right groin swelling, below and lateral to the pubic tubercle. Which of the
following forms the medial boundary of the ring through which this hernia is
protruding?
a. Conjoint tendon.
b. Inferior epigastric artery.
c. Inguinal ligament.
d. Lacunar ligament.
e. Pectineal ligament.

8. An 82-year-old man has complete occlusion of his inferior mesenteric artery


on angiography but no symptoms or signs of colonic ischaemia. Which of the
following arteries is the MOST likely additional source of blood supply to the
territory of the inferior mesenteric artery?
a. Left colic.
b. Left gastroepiploic.
c. Middle colic.
d. Splenic.
e. Superior rectal.

P a g e 2 | 26
SurgiNotes 2nd Ed. (2022)

9. An 80-year-old man presents with a prostatic malignancy. Imaging reveals a


sclerotic lesion in a thoracic vertebra. Which is the MOST likely vascular route
of metastatic spread?
a. Cremasteric vein.
b. Inferior mesenteric vein.
c. Internal iliac vein.
d. Testicular vein.
e. Vertebral veins.

10.Which of the following statements regarding splenic anatomy is TRUE?


a. The splenic ligaments are all avascular.
b. The tail of the pancreas is often contained in the splenorenal ligament.
c. The average weight of the adult spleen is 300 g.
d. The first branches of the splenic artery are the short gastric arteries.
e. Accessory spleens are most commonly found in the greater omentum.

11.A CT scan of a 63-year-old man reveals that the left renal vein is occluded as it
crosses the aorta. The occlusion is due to compression by an arterial aneurysm
anterior to the vein. What is the MOST likely location of this aneurysm?
a. Coeliac artery.
b. Inferior mesenteric artery.
c. Left colic artery.
d. Middle colic artery.
e. Superior mesenteric artery.

12.A 70-year-old woman undergoes a left mastectomy and axillary clearance. At


a follow-up appointment she was noted to have winging of the left scapula.
Which muscle has been paralysed as a result of the dissection of the axilla?
a. Latissimus dorsi.
b. Pectoralis major.
c. Serratus anterior.
d. Teres major.
e. Trapezius.
P a g e 3 | 26
SurgiNotes 2nd Ed. (2022)

13.A 44-year-old man is admitted to the emergency department with excessive


vomiting and dehydration. Radiographic images demonstrate that part of the
bowel is being compressed between the abdominal aorta and the superior
mesenteric artery. Which of the following intestinal structures is MOST likely
being compressed?
a. Second part of duodenum.
b. Transverse colon.
c. Third part of duodenum.
d. First part of duodenum.
e. Jejunum.

14.A 22-year-old man is admitted to the emergency department after falling from
his bicycle. Radiograph examination reveals a fracture of the tibia above ankle.
MRI and physical examination reveal the tibial nerve is severed on the
posterior aspect of tibia. Which of the following signs will MOST likely present
during physical examination?
a. Sensory loss of the dorsum of the foot.
b. Sensory loss on the sole of the foot.
c. Foot drop.
d. Paralysis of the extensor digitorum brevis.
e. Sensory loss of the entire foot.

15.Which of the following is the main action of secretin?


a. It augments the action of cholecystokinin.
b. It inhibits acid secretion by the stomach.
c. It stimulates the production of enzyme rich pancreatic juice.
d. It stimulates the production of alkaline pancreatic juice.
e. It stimulates the secretion of bile.

16.Which of the following statements is correct regarding potassium


homeostasis?
a. About 60% of the total body potassium is in the intracellular
compartment.
b. Aldosterone stimulates potassium reabsorption in the distal convoluted
tubules.
c. Hypokalaemia causes depression of the ST segment.
d. In alkalosis there is usually hyperkalaemia.
e. Insulin causes potassium to leave the cell.
P a g e 4 | 26
SurgiNotes 2nd Ed. (2022)

17.What is the main brain's fuel during prolonged starvation?


a. Amino acid.
b. Glucose.
c. Ketones.
d. Lactose.
e. Short-chain fatty acid.

18.A 45-year-old female patient complains of reflux oesophagitis secondary to a


sliding hiatus hernia. Which of the following decreases the tone of the lower
esophageal sphincter?
a. Fatty meal.
b. Gastric alkalization.
c. Gastric distension.
d. Gastrin.
e. Proteins.

19.Which of the following is TRUE about epinephrine?


a. It decreases lipolysis in adipose tissues.
b. It decreases lipolysis in the liver.
c. It decreases the secretion of thyroid hormone.
d. It increases glucagon secretion.
e. It increases insulin release.

20.The majority of the blood volume at rest is contained within the:


a. Arterial system.
b. Capillary bed.
c. Portal circulation.
d. Pulmonary circulation.
e. Venous system.
P a g e 5 | 26
SurgiNotes 2nd Ed. (2022)

21.The hypothalamus contains cells which are sensitive to:


a. Partial pressure of oxygen.
b. Arterial blood pressure.
c. Hydrogen ions concentration.
d. Thyroid-stimulating hormone concentration.
e. Plasma volume.

22.In compensating for respiratory alkalosis, the body excretes more:


a. Ammonium ions.
b. Bicarbonate ions.
c. Dihydrogen phosphate ions.
d. Carbonic acid.
e. Hydrogen ions.

23.A fit 30-year-old man donates 500 ml of blood. Which one of the following is
the MOST likely physiological change?
a. A fall in blood pressure.
b. Activation of renin-angiotensin system.
c. Reduced urine output.
d. Sweating.
e. Tachypnoea.

24.Which of the following gastrointestinal secretions has the highest bicarbonate


level?
a. Gastric.
b. Pancreatic.
c. Colonic.
d. Ileal.
e. Saliva.
P a g e 6 | 26
SurgiNotes 2nd Ed. (2022)

25. During the second (proliferative) phase of wound healing the predominant cells
in the wound site are fibroblasts. This cell is of mesenchymal origin and produces
the matrix and collagen needed to strengthen the scar. Cross linkage of collagen
requires hydroxy-proline and hydroxy-lysine residues, which requires a specific
vitamin to be available in sufficient quantities. Deficiency of which vitamin
results in collagen that is unstable?
a. Vitamin B2 (riboflavin).
b. Vitamin B6 (pyridoxine).
c. Vitamin C (ascorbic acid).
d. Vitamin D (cholecalciferol).
e. Vitamin E (tocopherol).
26. A 78-year-old woman with emphysema receiving 28% oxygen by mask has the
following blood gas results:
PH PO2 PaCO2 Bicarbonate Base excess
Finding: 7.28 70 mmHg 48 mmHg 36 mmol/L +5
The MOST likely interpretation is:
a. Mixed respiratory and metabolic acidosis.
b. Partially compensated metabolic acidosis.
c. Partially compensated respiratory acidosis.
d. Uncompensated metabolic acidosis.
e. Uncompensated respiratory acidosis.
27. A 70-year-old patient with diabetes and paraplegia is undergoing an elective
laparoscopic cholecystectomy after an episode of biliary pancreatitis. Shortly
after induction, blood pressure is normal, but ECG shows peaked P waves and a
widened QRS complex. The MOST likely diagnosis is:
a. Ketoacidosis.
b. Hyperkalemia.
c. Hypoglycemia.
d. Hypocalcemia.
e. Acute myocardial infarction.
28. A 70-year-old man with chronic obstructive pulmonary disease is admitted for
elective hemicolectomy. What is a preoperative arterial blood gas analysis likely
to show?
Arterial PCO2 Bicarbonate.
a. Decreased Decreased.
b. Decreased Increased.
c. Decreased Normal.
d. Increased Decreased.
e. Increased Increased.
P a g e 7 | 26
SurgiNotes 2nd Ed. (2022)

29.Which of the following is TRUE regarding the release of tumour necrosis


factor-α?
a. Can be effectively blocked by anti-TNF-α antibodies to halt systemic
inflammatory response syndrome (SIRS).
b. Does not have any beneficial effects in the early phases of the
inflammatory response.
c. Is primarily from leukocytes.
d. Promotes polymorphonuclear (PMN) cell adherence and further
cytokines release.
e. Is always deleterious.

30.Which one of the following is LEAST useful in the immediate treatment of


hyperkalemia?
a. Calcium salts.
b. Sodium bicarbonate.
c. Potassium-binding resins.
d. Glucose and insulin.
e. Hemodialysis.

31.Which pancreatic islet cell type produces a hormonal peptide to stimulate


glycogenolysis and gluconeogenesis?
a. Alpha cell.
b. Beta cell.
c. Delta cell.
d. F cell.
e. PP cell.

32.A 15-year-old female patient presented by enlargement of the upper deep


cervical lymph nodes which had been present for 4-months. The patient did
not respond-to-multiple courses of antibiotics. The temperature was 37.8 oC.
The lymph nodes were matted together and they were slightly tender. Which
of the following cells will be predominant on histopathological examination?
a. Epithelioid cells.
b. Fibroblasts.
c. Lymphocytes.
d. Plasma cells.
e. Polymorphonuclear leucocytes.

P a g e 8 | 26
SurgiNotes 2nd Ed. (2022)

33.Which of the following cells is an example of a permanent cell NOT capable of


division?
a. Acinar cells of the pancreas.
b. Colonic mucosal cells.
c. Erythrocytes.
d. Hepatocytes.
e. Osteocytes.

34.Which of the following phases of cell cycle is MOST resistant to


chemotherapeutic therapy?
a. G0.
b. S.
c. G1.
d. M
e. G2.

35.A 45-year-old woman is admitted with a chronic cough and weight loss. On
examination, crepitations are heard over the apex of the right lung. A chest X-
ray reveals cavitation in the right apex. What is a lung biopsy MOST likely to
show?
a. Abundant plasma cells.
b. Aggregates of activated macrophages.
c. Eosinophils.
d. Mast cells.
e. Neutrophilic infiltrates with abscess formation.

36.With regard to the healing process which of the following statements is


correct?
a. Collagen content reaches a maximum at approximately one week after
injury.
b. Monocytes are essential for normal wound healing.
c. Fibroblasts appear in the wound within 24 to 36 h after the injury.
d. The function of monocytes in wound healing is limited to phagocytosis of
bacteria and debris.
e. Early in wound healing, type I collagen is predominant.
P a g e 9 | 26
SurgiNotes 2nd Ed. (2022)

37.In the presence of acute inflammation, when an abscess has formed, which of
the following cell types is MOST likely to be primarily involved in phagocytic
activity?
a. Erythrocytes.
b. Lymphocytes.
c. Macrophages.
d. Mast cells.
e. Plasma cells.

38.A 45-year-old man had a 12-year history of gastro-oesophageal reflux disease.


The lower third of his oesophagus was reddened and biopsies revealed
columnar epithelium. What was the process affecting the tissue?
a. Anaplasia.
b. Dysplasia.
c. Hyperplasia.
d. Hypertrophy.
e. Metaplasia.

39.Which of the following is the MOST likely cause of pyrexia occurring 48 hours
after an abdominal operation?
a. Chest infection.
b. DVT.
c. Leaking intestinal anastomosis.
d. Pulmonary embolism.
e. Wound infection.

40.Which of the following organism is famous for production of the enzyme


penicillinase and resists penicillin therapy?
a. Bacteroides.
b. Hemolytic streptococci.
c. Klebsiella.
d. Pseudomonas aeruginosa.
e. Staphylococcus aureus.
P a g e 10 | 26
SurgiNotes 2nd Ed. (2022)

41.Which of the following is TRUE regarding antibiotics?


a. Aminoglycosides may cause hepatotoxicity.
b. Ampicillin in effective against pseudomonas infections.
c. Cephalosporins are rarely used for prophylaxis.
d. Penicillins act by disrupting the peptidoglycan of the bacterial cell wall.
e. Vancomycin is the first choice for treatment of infections with Staph.
aureus.

42.Which of the following operations is a clean contaminated operation?


a. Colectomy for a prepared colon.
b. Drainage of a pericolic abscess.
c. Inguinal herniectomy.
d. Perforated appendicitis.
e. Thyroidectomy.

43.A 45-year-old diabetic male patient complains of severe pain in the thigh.
Examination reveals spreading oedema, swelling and tenderness of the thigh.
A provisional diagnosis of necrotizing fasciitis is made. Which of the following
is MOST important in the treatment?
a. Antifungal agents.
b. Antitoxin.
c. Hyperbaric O2.
d. Immunoglobulins.
e. Wide surgical debridement.

44.The effectiveness of prophylactic antibiotics in surgery is MOSTLY related to


the:
a. Continuation of antibiotics for 24 hours after surgery.
b. Timing of initial administration.
c. Use of bactericidal agents.
d. Use of broad-spectrum agents.
e. Use of two synergistic antibiotics.
P a g e 11 | 26
SurgiNotes 2nd Ed. (2022)

45.A 40-year-old male patient had renal transplantation and is receiving


immunosuppression. Few days following the operation the patient developed
persistent fever. Which of the following infectious microorganisms is currently
the MOST likely cause of this fever?
a. Candidiasis.
b. Coli sepsis.
c. Cytomegalovirus sepsis.
d. Pneumococcal sepsis.
e. Streptococci.

46.A 30-year-old female patient developed postoperative wound infection after


a thyroidectomy operation. What is the MOST likely causative organism?
a. Bacteroides.
b. E. coli.
c. Proteus.
d. Staph. aureus.
e. Streptococci.

47.Which of the following is the mode of action of amikacin?


a. Blockage of bacterial DNA replication.
b. Competitive inhibition of the enzyme dihydrofolate reductase.
c. Inhibition of bacterial wall synthesis.
d. Inhibition of protein synthesis by ribosomes.
e. Inhibition of translocation of peptides.

48.What of the following is the MOST common serious infectious complication of


blood transfusion?
a. Acquired immunodeficiency syndrome.
b. Cytomegalovirus.
c. Malaria.
d. Virus A hepatitis.
e. Virus C hepatitis.
P a g e 12 | 26
SurgiNotes 2nd Ed. (2022)

49.Which of the following organisms produces an endotoxin?


a. Clostridium botulinum.
b. Clostridium tetani.
c. E. coli.
d. Pneumococci.
e. Staphylococcus aureus.

50.A 30-year-old woman sustained a puncture wound to the foot. The patient has
been on a therapeutic dose of steroids for the past 5 years for ulcerative colitis.
Her last tetanus toxoid booster was 8 years ago. What should the patient
receive?
a. Tetanus toxoid booster.
b. Human immunoglobulin, antibiotics with anaerobic coverage.
c. Tetanus toxoid plus human immunoglobulin.
d. Tetanus toxoid plus human immunoglobulin and antibiotics with aerobic
and anaerobic coverage.
e. Wide debridement of the wound.

51.Which of the following groups of antibiotics will be effective against


methicillin resistant staph organisms (MRSA)?
a. Third generation cephalosporins.
b. Macrolides.
c. Fluroquinolones.
d. Glycopeptides.
e. Ampicillin + Clavulanic acid.

52.An 80-year-old woman who lives in a nursing home and who had just finished
a 10-day course of antibiotics has abdominal pain and profuse diarrhea. Her
stool is tested and comes back positive for clostridium difficile. What is the
MOST appropriate initial management?
a. Oral vancomycin.
b. Intravenous vancomycin.
c. Metronidazole.
d. Vancomycin enemas.
e. Supportive treatment only.
P a g e 13 | 26
SurgiNotes 2nd Ed. (2022)

53.Which of the following cells is responsible for cellular mediated immunity?


a. B-lymphocytes.
b. Dendritic cells.
c. Monocytes.
d. Natural killer cells.
e. T-lymphocytes.

54.A 64-year-old man underwent transplantation, which was complicated by


graft-versus-host reaction. He had undergone a transplantation of which of
the following?
a. Kidney.
b. Skin.
c. Bone marrow.
d. Cornea.
e. Liver.

55.A 40-year-old male patient underwent a renal transplant from a cadaveric


donor. Immediately after finishing the vascular anastomosis, the kidney
became cyanotic and flaccid. Histological examination revealed deposition of
immunoglobulins and complement in the vessels walls. What is the
immunological background of the problem?
a. Donor cytotoxic T-lymphocytes directed against the host antigens.
b. Donor natural killer cells directed against host antigens.
c. Host natural killer cells against donor antigens.
d. Preformed donor antibodies against the host antigens.
e. Preformed host antibodies against the donor antigens.

56.Which of the following is the MOST commonly used-primary maintenance


immunosuppressive agent?
a. Tacrolimus (prograf).
b. Azathioprine (imuran)
c. Mycophenolate (cellcept).
d. Rapamycin (sirolimus).
e. Corticosteroids.
P a g e 14 | 26
SurgiNotes 2nd Ed. (2022)

57.A patient is reported to have a potassium level of 7.3 mEq/L. What is the first
diagnostic or therapeutic maneuver for this patient?
a. An ECG and administration of 1 g of 10% calcium gluconate.
b. An ECG and infusion of 45 mEq/L NaHCO3.
c. An ECG and infusion of glucose and insulin intravenously.
d. Kayexalate enema.
e. Transfer to an intensive care unit for careful cardiac monitoring.

58.The arterial blood gas analysis of a patient was pH 7.5, P(CO2) 47 mmHg, HCO3-
35 mmol/L. This patient MOST likely is suffering from:
a. Chronic obstructive pulmonary disease.
b. Diabetic ketoacidosis.
c. Persistent diarrhea.
d. Profound vomiting.
e. Salicylate poisoning.

59.Five days after an uneventful cholecystectomy, an asymptomatic middle aged


woman is found to have a serum sodium level of 120 mEq/L. What is the
proper management?
a. Administration of hypertonic saline solution.
b. Administration of Ringer's lactate solution.
c. Aggressive diuresis with furosemide.
d. Plasma ultra-filtration.
e. Restriction of free water.

60.What is the MOST common symptom after major pulmonary embolism?


a. Cough.
b. Dyspnea.
c. Hemoptysis.
d. Pleural pain.
e. Palpitation.
P a g e 15 | 26
SurgiNotes 2nd Ed. (2022)

61.A 45-year-old male with a known history of alcoholism is admitted with acute
pancreatitis. His serum calcium is 7 mg/dl. Management is based upon which
of the following?
a. One-fourth of calcium in scrum is ionized.
b. Alkalosis increases the ionized calcium component.
c. Hypocalcemia may cause polyuria and polydipsia.
d. Determination of serum albumin is necessary for proper determination of
serum calcium level.
e. 20% of serum calcium is bound to citrates.

62.A 75-year-old woman who is in the ICU after undergoing colectomy is


hypotensive and tachycardic. Pulmonary capillary wedge pressure (PCWP) is
elevated to 18 mmHg and cardiac output is 3 L/min. She is in shock best
described as which of the following?
a. Hypovolemic shock.
b. Septic shock.
c. Cardiogenic shock.
d. Anaphylactic shock.
e. Neurogenic shock.

63.Which of the following metabolic effects is present in shock?


a. Increase in sodium and water excretion.
b. Increase in renal perfusion.
c. Decrease in cortisol levels.
d. Hyperkalemia.
e. Hypoglycemia.

64.Features of adult respiratory distress syndrome (ARDS) include all the


following EXCEPT:
a. Increased lung compliance.
b. Diffuse pulmonary infiltrates on chest X-ray.
c. Increase alveolar arterial oxygen gradient.
d. Diuretics may help in recovery of these patients.
e. Artificial ventilation when required must be based on pressure based
ventilation.
P a g e 16 | 26
SurgiNotes 2nd Ed. (2022)

65.A 40-year-old man is confused and restless the second day after upper
abdominal surgery and repair of a hiatus hernia. What is the MOST probable
cause of his condition?
a. Pulmonary embolism.
b. Narcotic overdose.
c. Pulmonary atelectasis.
d. Electrolyte imbalance.
e. Starvation ketosis.

66.Which of the following is the commonest cause of ARDS?


a. Sepsis syndrome.
b. Aspiration.
c. Acute pancreatitis.
d. DIC.
e. Fat embolism.

67.A 39-year-old woman is making a slow but adequate recovery after sustaining
a 40% surface area burn injury. On the sixth postoperative day she becomes
unwell. She vomits intermittently, has painless abdominal distension and
starts to hiccup. What is the MOST likely cause of these symptoms?
a. Acute gastric dilatation.
b. Acute intestinal obstruction.
c. Clostridium difficile infection.
d. Faecal impaction.
e. Systemic sepsis.

68.A 60-year-old man had undergone exploratory laparotomy for perforated


gastric ulcer with severe peritoneal contamination. Six hours after surgery, he
is tachycardic, hypertensive and has shallow respirations. Intubation and
institution of ventilatory support is indicated in the presence of which of the
following?
a. Respiratory rate of 23 breaths/min.
b. PaCO2 of 45 mmHg.
c. PaO2 of 55 mmHg on room air.
d. Heart rate of 140 bpm.
e. BP of 100/70 mmHg.
P a g e 17 | 26
SurgiNotes 2nd Ed. (2022)

69.Which of the following types of shock is associated with high pulmonary


wedge pressure?
a. Hypovolemic shock.
b. Cardiogenic shock.
c. Early septic shock.
d. Late septic shock.
e. Neurogenic shock.

70.Which of the following techniques does NOT provide a definitive airway?


a. Cricothyroidotomy.
b. Tracheostomy.
c. Nasotracheal tube.
d. Laryngeal mask airway.
e. Endotracheal tube.

71.Dopamine is a frequently used drug in critically ill patients because:


a. At high doses it increases splanchnic flow.
b. At high doses it increases coronary flow.
c. At low doses it decreases heart rate.
d. At low doses it lowers peripheral resistance.
e. It inhibits catecholamine release.

72.A 28-year-old motorcyclist is admitted following a road traffic accident, having


sustained bilateral femoral fractures and a ruptured spleen. Three days
postoperatively he is noted to be agitated, hypoxaemic and difficult to
ventilate. His observations show a blood pressure of 120/80 mmHg, regular
pulse of 88 beats/minute and he is apyrexial. A chest X-ray shows bilateral
diffuse lung infiltrates. What is the MOST likely underlying diagnosis?
a. Adult respiratory distress syndrome (ARDS).
b. Atelectasis.
c. Bronchopneumonia.
d. Pulmonary oedema.
e. Pulmonary thrombo-embolism.
P a g e 18 | 26
SurgiNotes 2nd Ed. (2022)

73.A 56-year-old motorcyclist presents to the emergency department after being


involved in a road traffic accident. He is conscious and is maintaining his own
airway and breathing. He is found to have an open right femoral fracture with
palpable distal pulses and normal sensation. Resuscitation is started. Urethral
catheterization drained 250 ml immediately but over the following 60 minutes
he drained only 10 ml. Which of the following is the MOST likely cause of his
low urine output?
a. Blocked catheter.
b. Cardiogenic shock.
c. Hypovolemic shock.
d. Ruptured bladder.
e. Septic shock.

74.A 55-year-old male patient is receiving 150 mg of aspirin daily because he has
coronary artery disease. Which of the following tests will be affected in this
patient?
a. Bleeding time.
b. Coagulation time.
c. INR.
d. PTT.
e. Thrombin time.

75.Which of the following is a contraindication to heparin therapy?


a. A Closed head injury two weeks ago.
b. Heparin-induced thrombocytopenia.
c. Subclavian vein thrombosis.
d. Superior mesenteric artery embolism.
e. Third trimester of pregnancy.

76.A 30-year-old male patient was admitted to the casualty department due to a
car accident. The patient had fracture of the pelvis and the right femur and he
received 5 liters of blood following which he started to have bleeding from his
nose and mouth. What is the main cause of this bleeding tendency?
a. Decrease in fibrinogen.
b. Decrease in prothrombin.
c. Decrease of calcium.
d. Increased fibrinolytic activity.
e. Platelet depletion.
P a g e 19 | 26
SurgiNotes 2nd Ed. (2022)

77.Which of the following is associated with a defect of the platelet function?


a. Epistaxis.
b. Hemarthrosis.
c. Normal bleeding time.
d. Prolonged prothrombin time.
e. Soft tissue hemorrhages.

78.What is the main mode of action of heparin?


a. It increases the level of protein C.
b. It is a cofactor of antithrombin III.
c. It prevents clot retraction.
d. It prevents platelets aggregation.
e. It prevents the synthesis of fibrinogen.

79.You were obliged to operate for a strangulated inguinal hernia in a patient


who was receiving 300 mg aspirin daily. During surgery there was excessive
bleeding. What would you advise?
a. Desmopressin.
b. Fresh blood.
c. Fresh frozen plasma.
d. Fresh platelets.
e. IV vitamin K.

80.You are planning to perform cholecystectomy for a female patient who is


receiving warfarin 5 mg daily because she had mitral valve replacement 4
years ago. What is your pre-operative management?
a. Continue warfarin and give fresh frozen plasma intraoperatively.
b. Continue warfarin and give fresh platelets intraoperatively.
c. Continue warfarin and give IV vitamin K intraoperatively.
d. Stop warfarin and substitute it by heparin for 10 days preoperatively
e. Stop warfarin and substitute it by heparin for 5 days preoperatively.
P a g e 20 | 26
SurgiNotes 2nd Ed. (2022)

81.A victim of blunt abdominal trauma requires a partial hepatectomy. He is


rapidly transfused with 8 units of appropriately cross-matched banked blood.
He is noted in the recovery room to be bleeding from intravenous puncture
sites and the surgical incision. Which of the following is responsible for the
patients coagulopathy?
a. II and VII.
b. II only.
c. IX and X.
d. V and VIII.
e. XI and XII.

82.A cirrhotic patient with abnormal haemostatic studies requires an urgent


cholecystectomy. A transfusion of fresh frozen plasma is planned to minimize
the risk of bleeding. What is the optimal timing of this transfusion?
a. In the recovery room.
b. Intraoperatively.
c. On call to surgery.
d. The night before surgery.
e. Two days before surgery.

83.A 5-year-old boy slipped and hurt his right knee while walking. He presents
with a tender, swollen, warm knee with significant hemarthrosis. His PT is 12
(normal 13 seconds), PTT is over 100 (normal 25 seconds), platelet count is
300,000/mm3 and bleeding time is normal. Initial management should consist
of which of the following?
a. Fresh-frozen plasma.
b. Aspiration of knee.
c. Factor VIII concentrate.
d. Passive exercise.
e. Long-leg cast.

84.Which of the following denotes a hemolytic transfusion reaction during


anesthesia?
a. Shaking chills and muscle spasms.
b. Fever and oliguria.
c. Hyperpyrexia and hypotension.
d. Tachycardia and cyanosis.
e. Bleeding and hypotension.
P a g e 21 | 26
SurgiNotes 2nd Ed. (2022)

85.A 50-year-old female patient has chronic renal failure and has been
maintained on chronic dialysis. The patient underwent cholecystectomy. Post-
operatively she had severe bleeding. What is the MOST likely cause for this
bleeding?
a. Elevated PT.
b. Elevated PTT.
c. Low platelet count.
d. Decreased platelet aggregation.
e. Sepsis.

86.A 70-year-old female patient is receiving warfarin because she has AF and had
previous thrombosis. Which of the following statements regarding warfarin is
correct?
a. The dose of warfarin is adjusted according to the partial thromboplastin
time.
b. Warfarin takes about 8 hours to exert its effects.
c. Warfarin acts by inhibiting factor XII.
d. If the patient is also taking aspirin, the dose of warfarin should be
reduced.
e. Protamine sulphate is the antidote to warfarin

87.In the awake, non-anesthetized patient suspected of having a hemolytic post-


transfusion reaction, the MOST characteristic signs are:
a. Nausea and vomiting.
b. Fever and chills.
c. Oliguria and hemoglobinuria.
d. Cyanosis and dyspnea.
e. Tenderness of the renal angle.

88.A 40-year-old-male-patient is complaining of persistent headache, recurrent


attacks of palpitation, sweating and visual disturbances. Blood pressure of the
patient is 180/110 mmHg. Which of the following is MOST accurate in the
diagnosis of the problem?
a. 24 hour urinary catecholamines.
b. 24 hour urinary VMA.
c. CT scan of the abdomen.
d. Plasma aldosterone level.
e. Plasma metanephrine level.
P a g e 22 | 26
SurgiNotes 2nd Ed. (2022)

89.A 30-year-old pregnant female patient (first trimester) developed palpitation


tremors, excessive sweating, nervousness and failure to gain weight. Which of
the following investigations is the MOST accurate to diagnose her condition?
a. Free thyroxine level.
b. Level of thyroid peroxidase enzyme.
c. Technetium scan.
d. Thyroglobulin level.
e. Total thyroxine level.

90.What is the hormone that acts on the intestines to increase calcium


absorption?
a. Calcitonin.
b. Corticotrophin releasing factor (CRF).
c. Pancreatic polypeptide.
d. Parathormone.
e. Thyroxine.

91.A 30-year-old female patient complains of fatigue, generalized bony aches and
depression. The patient gives a history of 2 previous operations for urinary
stones. Which of the following laboratory tests is MOST accurate in the
diagnosis?
a. 24 hours urinary calcium.
b. Ionizable serum calcium.
c. Total serum calcium .
d. Plasma chloride.
e. Serum phosphate.

92.What is the mode of action of carbimazole?


a. It diminishes the blood supply of the thyroid gland.
b. It prevents the release of thyroid hormone from the gland.
c. It prevents the union between iodine and tyrosine.
d. It prevents transformation of iodides to iodine.
e. It prevents uptake of iodides by the thyroid gland.
P a g e 23 | 26
SurgiNotes 2nd Ed. (2022)

93.What is the commonest cause of thyrotoxicosis?


a. Autonomous nodule.
b. Graves' disease.
c. Iodine intake.
d. Subacute thyroiditis.
e. Toxic nodular goiter.

94.What is the recommended treatment of acute adrenal insufficiency?


a. Normal saline, potassium and glucose.
b. Hypertonic saline and potassium.
c. Normal saline and potassium.
d. Intravenous mineralocorticoids.
e. Normal saline, glucose and intravenous glucocorticoids.

95.A 30-year-old primigravida complains of headaches, restlessness, sweating


and tachycardia. She is 18 week pregnant and her blood pressure is 200/120
mmHg. Appropriate workup might include:
a. Exploratory laparotomy.
b. Mesenteric angiography.
c. Head CT scan.
d. Abdominal CT scan.
e. Abdominal ultrasonogram.

96.A 53-year-old man with renal failure presents with nausea, headache and
pruritus. Blood tests show a serum calcium of 13 mg/dl (normal 9-10.5 mg/dl).
Which of the following is the MOST likely diagnosis?
a. Hypoparathyroidism.
b. Primary hyperparathyroidism.
c. Pseudohypoparathyroidism.
d. Secondary hyperparathyroidism.
e. Tertiary hyperparathyroidism.
P a g e 24 | 26
SurgiNotes 2nd Ed. (2022)

97. A patient who had adrenalectomy to remove a pheochromocytoma


demonstrates signs of confusion and complains of sweating and headache
several hours following his operation. His blood pressure is 130/65 mmHg, his
heart rate is 100 beats/min and his respiratory rate is 12 breaths/min. What is
the MOST likely cause of his symptoms?
a. Dehydration.
b. Postoperative bleeding.
c. Hypoglycemia.
d. Narcotic overdose.
e. Incomplete removal of the pheochromocytoma.

98. A 42-year-old woman is in the intensive care unit immediately following removal
of a left adrenal pheochromocytoma. Her blood pressure is 80/40 mmHg. The
MOST appropriate treatment of the patient's hypotension is:
a. Epinephrine.
b. IV bolus of lactated Ringer solution.
c. Methylprednisolone.
d. Phenoxybenzamine.
e. Phenylephrine.

99. On postoperative day 3, after undergoing an exploratory laparotomy, distal


pancreatectomy, splenectomy and fixation of a left femur fracture after a motor
vehicle collision, a 46-year-old man is found to be lethargic, confused and
vomiting. His blood pressure is 90/40 mmHg, serum glucose is 45 mg/dl, serum
sodium is 121 mEq/L, serum potassium is 5.3 mEq/L and hemoglobin 11.2 mg/dl.
Which of the following is the MOST likely cause accounting for his condition?
a. Acute adrenal insufficiency.
b. Internal bleeding.
c. Pituitary infarction.
d. Pulmonary embolism.
e. Volume overload with lactated Ringer solution.

100. A 75-year-old woman presents with acute back pain after a minor fall.
Radiological examination reveals an osteoporotic crush fracture. What is the
expected laboratory finding?
a. Hypercalcaemia.
b. Hyperphosphataemia.
c. Hypokalaemia.
d. Hyponatraemia.
e. Normocalcaemia.
P a g e 25 | 26
SurgiNotes 2nd Ed. (2022)

Egyptian Board Of General Surgery


1st Part Exam – Paper (1) MCQ "ANSWER"
March 2015

1 a 26 c 51 d 76 e
2 d 27 b 52 c 77 a
3 d 28 e 53 e 78 b
4 a 29 d 54 c 79 a
5 c 30 c 55 e 80 e
6 b 31 a 56 a 81 d
7 d 32 a 57 a 82 c
8 c 33 c 58 d 83 c
9 e 34 a 59 e 84 e
10 b 35 b 60 b 85 d
11 e 36 b 61 d 86 d
12 c 37 c 62 c 87 b
13 c 38 e 63 d 88 e
14 b 39 a 64 a 89 a
15 d 40 e 65 c 90 d
16 c 41 d 66 a 91 b
17 c 42 a 67 a 92 c
18 a 43 e 68 c 93 b
19 d 44 b 69 b 94 e
20 e 45 c 70 d 95 e
21 d 46 d 71 b 96 e
22 b 47 d 72 a 97 c
23 b 48 e 73 c 98 b
24 b 49 c 74 a 99 a
25 c 50 d 75 b 100 e

P a g e 26 | 26
SurgiNotes 2nd Ed. (2022)

Egyptian Board Of General Surgery


1st Part Exam – Paper (1) MCQ
February 2017
1. Which of the following nerves passes beneath the inguinal ligament?
a. Saphenous nerve.
b. Genital branch of the genitofemoral nerve.
c. Iliohypogastric nerve.
d. Ilioinguinal nerve.
e. Lateral cutaneous nerve of the thigh.

2. A 30-year-old male received a blow to the front of his arm. Following that he
suffered from weakness in flexion and supination of the forearm. Which of the
following nerves is MOST likely to be injured?
a. Axillary.
b. Median.
c. Musculocutaneous.
d. Radial.
e. Ulnar.

3. Which of the following overlies the right suprarenal gland?


a. Aorta.
b. Inferior vena cava.
c. Right crus of the diaphragm.
d. Right hepatic vein.
e. Right renal artery.

4. A 40-year-old-male patient had a superficial parotidectomy operation. Post-


operatively he complained of numbness at the lobule of the ear. What was the
injured nerve?
a. Buccal branch of the facial nerve.
b. Cervical branch of the facial nerve.
c. Great auricular nerve.
d. Lesser occipital nerve.
e. Marginal mandibular nerve.
P a g e 1 | 26
SurgiNotes 2nd Ed. (2022)

5. A 74-year-old woman present to the outpatient clinic with pelvic pain. A CT


scan reveals enlarged para-aortic lymph nodes. These are MOST likely to be
involved in secondary spread from a tumour in which of the following organs?
a. Cervical.
b. Ovary.
c. Uterus.
d. Vagina.
e. Vulva.

6. A 64-year-old woman is reviewed in the emergency department with an acute


right groin swelling, below and lateral to the pubic tubercle. Which of the
following forms the medial boundary of the ring through which this hernia is
protruding?
a. Conjoint tendon.
b. Inferior epigastric artery.
c. Inguinal Ligament.
d. Lacunar Ligament.
e. Pectineal Ligament.

7. A 30-year-old motorcyclist suffers a closed fracture to the mid-shaft of the


tibia. Anterior compartment syndrome could cause loss of sensation:
a. In the first web space.
b. Over the dorsum of the foot.
c. Over the lateral edge of the foot.
d. Over the medial aspect of the hallux.
e. Over the medial malleolus.

8. A 32-year-old man is brought to the emergency department following a fall.


An X-ray reveals that he has a fracture of the surgical neck of his humerus.
Neurological examination reveals paraesthesia over the upper lateral arm
(overlying the deltoid muscle). Which nerve is MOST likely to have been
damaged?
a. Axillary.
b. Median.
c. Musculocutaneous.
d. Radial.
e. Ulnar.

P a g e 2 | 26
SurgiNotes 2nd Ed. (2022)

9. A CT scan of the a 63-year-old man reveals that the left renal vein is occluded
as it crosses the aorta. The occlusion is due to compression by an arterial
aneurysm anterior to the vein. What is the MOST likely location of aneurysm?
a. Coeliac artery.
b. Inferior mesenteric artery.
c. Left colic artery.
d. Middle colic artery.
e. Superior mesenteric artery.

10.A 70-year-old woman undergoes a left mastectomy and axillary clearance. At


a follow-up appointment she was noted to have winging of left scapula. Which
muscle has been paralysed as a result of the dissection of the axilla?
a. Latissimus dorsi.
b. Pectoralis major.
c. Serratus anterior.
d. Teres major.
e. Trapezius.

11.A 35-year-old woman is seen at the surgical clinic with a history of faecal
incontinence since the birth of her second child 18 months previously. The first
stage of labour had been prolonged and difficult. Physical examination reveals
a relatively lax anal sphincter. Which nerve is likely to have been damaged in
labour?
a. Genitofemoral nerve.
b. Lumbosacral trunk.
c. Obturator nerve.
d. Pelvic splanchnic nerve.
e. Pudendal nerve.

12.A 65-year-old man presents with hematuria and right loin pain. Computerized
tomography demonstrates a right renal tumour and he undergoes a right
radical nephrectomy. The right testicular vein drains into which of the
following?
a. Inferior mesenteric vein.
b. Inferior vena cava.
c. Right adrenal vein.
d. Right lumber vein.
e. Right renal vein.
P a g e 3 | 26
SurgiNotes 2nd Ed. (2022)

13. A 19-year-old man is brought to the emergency department after dislocation his
shoulder while playing football. Following treatment of the dislocation, he
cannot initiate abduction of his arm. An MRI of the shoulder shows a torn muscle.
Which muscle was MOST likely damaged by the injury?
a. Coracobrachialis.
b. Long head of the triceps.
c. Pectoralis minor.
d. Supraspinatus.
e. Teres major.
14. A 32-years-old male patient is admitted to the emergency department with groin
pain. Examination reveals that the patient has an indirect inguinal hernia. Which
of the following nerves is compressed by the herniated structure in the inguinal
canal to give the patient pain?
a. Iliohypogastric.
b. Lateral femoral cutaneous.
c. Ilioinguinal.
d. Subcostal.
e. Pudendal.
15. A 44-year-old man is admitted to the emergency department with excessive
vomiting and dehydration. Radiographic images demonstrate that part of bowel
is being compressed between the abdominal aorta and the superior mesenteric
artery. Which of the following intestinal structures is MOST likely being
compressed?
a. second part of duodenum.
b. Transverse colon.
c. Third part of duodenum.
d. First part of duodenum.
e. Jejunum.
16. A 34-year-old is undergoing an emergency appendectomy has been performed
successfully. The patient undergoes an exploratory laparoscopy. Which of the
following anatomic feature are the MOST useful to distinguish the jejunum form
the ileum?
a. Jejunum has thinner walls compared with the ileum.
b. Jejunum has less mesenteric fat compared with ileum.
c. Jejunum has more numerous vascular arcades compared with the ileum.
d. Jejunum has more numerous lymphatic follicles beneath the mucosa
compared with ileum.
e. Jejunum has fewer villi compared with the ileum.
P a g e 4 | 26
SurgiNotes 2nd Ed. (2022)

17.After a modified radical mastectomy, a 45-year-old woman reports weakness


in her arm. On examination, she has mild weakness when she internally
rotates and adducts her arm. This is MOST likely due to injury to which of the
following nerves?
a. Intercostal brachial.
b. Long thoracic.
c. Thoracodorsal.
d. Supraclavicular.
e. Medial pectoral.

18.Which of the following cell types is essential for normal wound healing?
a. Erythrocytes.
b. Leukocytes.
c. Lymphocytes.
d. Monocytes.
e. Platelets.

19.Wound healing is relatively slowest in which of the following ?


a. Eyelid.
b. Lips.
c. Sternum.
d. Stomach.
e. Urinary bladder.

20.Which of the following is TRUE about the catabolic response to trauma?


a. Intravenous hyperalimentation can prevent the catabolic response to
trauma.
b. Liver glycogen is the source of dextrose in the first week.
c. the catabolic response is initiated by the thyroid hormones.
d. the catabolic response is the same regardless of the severity of trauma.
e. There is inevitable loss of muscle mass.
P a g e 5 | 26
SurgiNotes 2nd Ed. (2022)

21.Which of the following can lower the serum potassium level?


a. Calcium gluconate.
b. Dextrose 25%.
c. Ringer's lactate.
d. Saline.
e. Sodium bicarbonate.

22.What is the site of maximum absorption of short chain fatty acids produced
by bacteria?
a. The caecum.
b. The colon.
c. The duodenum.
d. The ileum.
e. The jejunum.

23.Which of the following is the MOST potent stimulus for contraction of the gall
bladder?
a. Acetylcholine.
b. Cholecystokinin.
c. Intravenous hyperalimentation.
d. Secretin.
e. Vagal stimulation.

24.The following acid base data: pH 7.2, PCO2 20 mmHg, HCO3- 8 mmol/L and
base excess 19 mmol/L, would be MOST consistent with:
a. Anxiety.
b. Lobar collapse of the lung.
c. Pyloric obstruction.
d. Septic shock.
e. Starvation.
P a g e 6 | 26
SurgiNotes 2nd Ed. (2022)

25.What is the main brain's fuel during prolonged starvation?


a. Amino acid.
b. Glucose.
c. Ketones.
d. Lactose.
e. Short-chain fatty acid.

26.A 45-year-old female patent complains of reflux oesophagitis secondary to a


sliding hiatus hernia. Which of following decreases the tone of the lower
esophageal sphincter?
a. Fatty meal.
b. Gastric alkalization.
c. Gastric distension.
d. Gastrin.
e. Proteins.

27.A 21-year-old graduate student has a large hypertrophic scar on the lower part
of face. The patient has sustained a laceration on her face 2 years previously
after injuring her face. Which of the following statement regarding scar
revision is TRUE?
a. Scar maturation refers to the change in size of the wound in the first 1 to
2 months.
b. Scar revision should have been performed in the first 3 months after
injury to minimize fibrosis.
c. Revision should be performed earlier in children than in adults.
d. It corrects undesirable pigmentation.
e. Scar revision should be delayed approximately 1 year to allow
maturation.

28.All of the following activate the sympathoadrenal and hypothalamic-pituitary


axes during stress or injury EXCEPT:
a. Pain.
b. Hypovolemia.
c. Acidosis.
d. Hypercapnia.
e. Acetylcholine.
P a g e 7 | 26
SurgiNotes 2nd Ed. (2022)

29.With regard to potassium which of the following statement is NOT true?


a. Normal dietary intake of potassium is 50 to 100 mEq/day.
b. In patients with normal renal function, most ingested potassium is
excreted in urine.
c. More than 90% of potassium in the body is located in the extracellular
compartment.
d. Critical hyperkalemia (> 6 mEq/L) is rarely encountered if renal function
is normal.
e. Administration of sodium bicarbonate shifts potassium from the
extracellular space (ECF) to the intracellular space (ICS).

30.Which of the following is NOT a trigger of visceral pain?


a. Ischemia.
b. Traction.
c. Distention.
d. Heat.
e. Inflammation.

31.Which pancreatic islet cell type produces a hormonal peptide to stimulate


glycogenolysis and gluconeogenesis?
a. Alpha cell.
b. Beta cell.
c. Delta cell.
d. F cell.
e. PP cell.

32.Norepinephrine (noradrenaline) causes an increase in the systemic vascular


resistance. In which condition, its use is MOST appropriate:
a. Cardiogenic shock.
b. Hypovolemic shock.
c. Neurogenic shock.
d. Obstructive shock (e.g. pulmonary embolism).
e. Septic shock.
P a g e 8 | 26
SurgiNotes 2nd Ed. (2022)

33.A 40-year-old man is brought to the emergency department following a road


traffic accident. The primary survey detects grade 3 shock as a result of pelvic
fracture. Which of the following pathophysiological mechanism is the first to
be activated to maintain circulating blood volume?
a. Antidiuretic hormone secretion.
b. Atrial naturetic peptide.
c. Parasympathetic nervous system.
d. Renin-angiotensin system.
e. Sympathetic nervous system.

34.Which of the following is often associated with early sepsis?


a. Normal acid-base balance.
b. Metabolic alkalosis.
c. Metabolic acidosis.
d. Respiratory alkalosis.
e. Respiratory acidosis.

35.Which of the following is TRUE with regard to wound contraction?


a. It is the primary process affecting closure of a sutured or stapled surgical
wound.
b. Bacterial colonization significantly slows the process of contraction.
c. It may account for a maximum of 50% decrease in the size of a wound.
d. It is based on specialized fibroblasts that contain actin myofilaments.
e. The percentage reduction of wound size is increased with adherence of
skin to underlying tissue.

36.Histopathological examination of a cavitating apical lung mass from a 45-year-


old man show necrotic material surrounded by epithelioid cells and occasional
multinucleated giant cells. Acid fast bacilli are identified. Which cell type gives
rise to giant cells?
a. Basophil.
b. Eosinophil.
c. Lymphocyte.
d. Macrophage.
e. Neutrophil.
P a g e 9 | 26
SurgiNotes 2nd Ed. (2022)

37.A 45-year-old man with a long history of ulcerative colitis undergoes


colonoscopy. Biopsy shows that the mucosal architecture is abnormal. The
epithelial cells have enlarged hyperchromatic nuclei and there is failure of
maturation towards the surface. There is no evidence of invasion. Which of
the following pathological processes best fits this histological description?
a. Carcinoma.
b. Dysplasia.
c. Hyperplasia.
d. Hypertrophy.
e. Metaplasia.

38.With regard to the healing process which of the following statements is


correct?
a. Collagen content reaches a maximum at approximately one week after
injury.
b. Monocytes are essential for normal wound healing.
c. Fibroblasts appear in the wound within 24 to 36 h after the injury.
d. The function of monocytes in wound healing is limited to phagocytosis of
bacteria and debris.
e. Early in wound healing type 1 collagen is predominant.

39.A 45-year-old man had a 22-0year history of gastro-oesophageal reflux


disease. The lower third of his oesophagus was reddened and biopsies
revealed columnar epithelium. What was the process affecting the tissue?
a. Anaplasia.
b. Dysplasia.
c. Hyperplasia.
d. Hypertrophy.
e. Metaplasia.

40.Which of the following is an early manifestation of sepsis?


a. Cutaneous vasodilatation.
b. Decreased cardiac output.
c. Hypoglycaemia.
d. Increased arteriovenous O2 difference.
e. Respiratory acidosis.
P a g e 10 | 26
SurgiNotes 2nd Ed. (2022)

41.Bacteroides fragilis responds best to adequate doses of which of the following


antibiotics?
a. Cefoxitin.
b. Clindamycin.
c. Erythromycin.
d. Tazocin.
e. Tobramycin.
42.A 30-year-old male patient was involved in a car accident. He had severe
lacerated wound involving the gluteal and thigh regions with laceration of
bulky muscles. What is the MOST important measure in the management of
this patient?
a. Hyperbaric O2.
b. Insertion of a drain.
c. Local antibiotics powder.
d. Removal of foreign and dead tissues.
e. Skin graft.
43.Which of the following cells will be involved in the chronic inflammatory
reaction which follows the introduction of foreign body?
a. Eosinophils.
b. Giant cells.
c. Mast cells.
d. Neutrophils.
e. Plasma cells.
44.A patient with a non-obstructing carcinoma of the colon is being prepared for
elective resection. Which of the following reduces the risk of postoperative
infectious complication?
a. Avoidance of oral antibiotics to prevent emergence of clostridium
difficile.
b. Postoperative administration for 48 h of parenteral antibiotics effective
against aerobes and anaerobes.
c. Postoperative administration of parenteral antibiotics effective against
aerobes and anaerobes until the patient's intravenous lines and all other
drains are removed.
d. Redosing of antibiotics in the operating room if the case lasts for more
than 2 hours of operating time.
e. Single preoperative parenteral dose of antibiotic effective against
aerobes.

P a g e 11 | 26
SurgiNotes 2nd Ed. (2022)

45.A 40-year-old male patient had renal transplantation and is receiving


immunosuppression. Few days following the operation the patient developed
persistent fever. Which of the following infectious microorganisms is currently
the MOST likely cause of this fever?
a. Candidiasis.
b. Coli sepsis.
c. Cytomegalovirus sepsis.
d. Pneumococcal sepsis.
e. Streptococci.

46.What of the following is the MOST common serious infectious complication of


blood transfusion?
a. Acquired immunodeficiency syndrome.
b. Cytomegalovirus.
c. Malaria.
d. Virus A hepatitis.
e. Virus C hepatitis.

47.Which of the following has the greatest influence for impaired healing of a
wound?
a. Anaemia.
b. Diabetes mellitus.
c. Intake of corticosteroids.
d. Local wound infection.
e. Malnutrition.

48.What is the commonest organism to cause infections following splenectomy?


a. Haemophilus influenza.
b. Escherichia coli.
c. Neisseria meningitides.
d. Staphylococcus aureus.
e. Streptococcus pneumonia.
P a g e 12 | 26
SurgiNotes 2nd Ed. (2022)

49.Which of the following statement is TRUE regarding beta-hemolytic


streptococci?
a. They produce a characteristic beta-lactamase.
b. They are one of the causes of necrotizing fasciitis.
c. They are the most common cause of infective endocarditis.
d. They are resistant to ampicillins.
e. They are part of normal vaginal flora.

50.What is the oral antibiotic MOST likely to be effective against a pseudomonas


urinary tract infection?
a. Ciprofloxacin.
b. Trimethoprim-sulfamethoxazole.
c. Cephalexin.
d. Nitrofurantoin.
e. Amoxicillin.

51.Regarding necrotizing fasciitis:


a. Skin manifestations may be totally absent.
b. It is commonly a single microbial infection.
c. The muscles are usually involved.
d. Broad spectrum antibiotics are the most essential measure of treatment.
e. It commonly occurs in previously healthy people.

52.Which of the following statements about the treatment of necrotizing fasciitis


is TRUE?
a. The underlying tissue necrosis is reflected by the extent of skin necrosis.
b. Intravenous immune globulin (IVIG) is the first line of therapy.
c. Penicillin provides sufficient antibiotic coverage.
d. Hyperbaric oxygen has been shown to improve survival.
e. Exploratory incisions over normal appearing skin are effective in
determining the extent of the necrosis.
P a g e 13 | 26
SurgiNotes 2nd Ed. (2022)

53.Which of the following is the mechanism of action of aminoglycosides?


a. Impairment of bacterial DNA synthesis.
b. Inhibition of cell wall synthesis.
c. Disruption of ribosomal protein synthesis.
d. Disruption of cell wall cation homeostasis.
e. Disruption of the cytoplasmic membrane.

54.The clinical course of majority of patients with HCV infection is characterized


by which one of the following?
a. Acute constitutional symptoms and jaundice.
b. Acute fulminant hepatic failure.
c. Development of chronic hepatitis.
d. Progression to cirrhosis.
e. Development of hepatocellular carcinoma.

55.An 80-year-old woman who lives in a nursing home and who had just finished
a 10-days course of antibiotics has abdominal pain and profuse diarrhea. Her
stool is tested and comes back positive for clostridium difficile. What is the
MOST appropriate initial management?
a. Oral vancomycin.
b. Intravenous vancomycin.
c. Metronidazole.
d. Vancomycin enemas.
e. Supportive treatment only.

56.Which of the following statement about the usage of antibiotics for the
prevention of surgical site infection is NOT true?
a. Antibiotics should be administrated within 60 minutes of incision.
b. The therapeutic dose of the antibiotic should be administrated
intravenously.
c. Adequate tissue concentrations should be maintained during operation
by re-dosing as necessary.
d. Antibiotics should be continued for 48 hours after operation.
e. Vancomycin is the agent of chronic for patients from nursing homes
undergoing hip replacement.
P a g e 14 | 26
SurgiNotes 2nd Ed. (2022)

57.The earliest manifestations of serious gram-negative infection may consist of


a triad of signs that includes:
a. Tachypnea, hypotension and altered sensorium.
b. Tachypnea, hypotension and lactic acidosis.
c. Thrombocytopenia, Hypotension and lactic acidosis.
d. Mild hyperventilation, respiratory alkalosis and an altered sensorium.
e. Tachycardia, hypotension and metabolic acidosis.

58.After being struck by a moving truck, a 23-year-old woman undergoes


splenectomy during diagnostic laparotomy. She leaves the hospital against
medical advice on postoperative day 4, after refusing vaccination. Infection
with of the following organisms is MOST likely to result in her developing
sepsis?
a. Beta-hemolytic streptococcus.
b. Candida albicans.
c. Clostridium difficile.
d. Escherichia coli.
e. Pseudomonas aeruginosa.

59.Severe sepsis is differentiated from sepsis by:


a. A history of premorbid conditions such as diabetes.
b. Positive blood cultures for bacteria and fungus.
c. Acute organ failure such as renal insufficiency.
d. Prolonged arterial hypotension.
e. Temperature more than 38.5 °C.

60.Which of the following cells is responsible for cellular mediated immunity?


a. B-lymphocytes.
b. Dendritic cells.
c. Monocytes.
d. Natural killer cells.
e. T-lymphocytes.
P a g e 15 | 26
SurgiNotes 2nd Ed. (2022)

61. What is the MOST common cause of death 2-years after heart
transplantation?
a. Accelerated graft arteriosclerosis.
b. Arrhythmia.
c. Cardiomyopathy.
d. Infection.
e. Malignancy.

62.Following organ transplantation, the rejection reaction that is caused by the


presence of preformed antibodies in the recipient is referred as:
a. Accelerated.
b. Acute.
c. Chronic.
d. Hyperacute.
e. Intermediate.

63.Which of the following is a major disadvantage of living donor organ


transplantation?
a. Elective procedure.
b. Immediate graft function.
c. Improved graft survival.
d. Mortality and morbidity of donors.
e. Shorter hospital stay.

64.A 40-year-old male patient underwent a renal transplant from a cadaveric


donor. Immediately after finishing the vascular anastomosis, the kidney
became cyanotic and flaccid. Histological examination revealed deposition of
immunoglobulins and complement in vessels walls. What is the
immunological background of the problem?
a. Donor cytotoxic T-lymphocytes directed against the host antigens.
b. Donor natural killer cells directed against host antigens.
c. Host natural killer cells against donor antigens.
d. Preformed donor antibodies against the host antigens.
e. Preformed host antibodies against the donor antigens.
P a g e 16 | 26
SurgiNotes 2nd Ed. (2022)

65.The arterial blood gas of a patient was pH 7.5, PCO2 47 mmHg and HCO3- 35
mmol/L. This patient MOST likely is suffering from:
a. Chronic obstructive pulmonary disease.
b. Diabetic ketoacidosis.
c. Persistent diarrhea.
d. Profound vomiting.
e. Salicylate poisoning.

66.Five days after an uneventful cholecystectomy, an asymptomatic middle aged


woman is found to have a serum sodium level of 120 mEq/L. What is the
proper management?
a. Administration of hypertonic saline solution.
b. Administration of Ringer's lactate solution.
c. Aggressive diuresis with furosemide.
d. Plasma ultra-filtration.
e. Restriction of free water.

67.What is the MOST common fluid disorder in the surgical patient?


a. Extracellular fluid deficit.
b. Hyperkalemia.
c. Hyponatremia.
d. Metabolic acidosis.
e. Metabolic alkalosis.

68.What is the MOST common symptom after major pulmonary embolism?


a. Cough.
b. Dyspnea.
c. Hemoptysis.
d. Pleural pain.
e. Palpitation.
P a g e 17 | 26
SurgiNotes 2nd Ed. (2022)

69.A 24-year-old woman has acute renal failure following postpartum


hemorrhage. Laboratory studies showed serum glucose 150 mg/dl, sodium
135 mEq/L, potassium 6.5 mEq/L, chloride 105 mEq/L and bicarbonate 15
mEq/L. Which of the following is recommended?
a. Decrease potassium chloride to 10 mEq/L.
b. Intravenous 0.9% sodium chloride.
c. 100 ml of 50% glucose water with 10 U insulin.
d. Intravenous calcitonin.
e. Intravenous magnesium sulfate.

70.A 40-year-old male patient developed convulsions 24 hours after having an


operation. His serum sodium is 120 mEq/L. Which of the following is the
appropriate management?
a. Administration of normal saline (0.9%).
b. Administration of hypertonic saline (3%).
c. Emergency hemodialysis.
d. Administration of vasopressor.
e. Administration of lasix 40 mg intravenously (IV).

71.A 50-year-old man presented with sever repeated vomiting as a result of


gastric outlet obstruction is admitted to the hospital. There is marked
dehydration, with urine output 20 ml/h and the hematocrit is 48%. Initial
treatment for this patient should include which of the following?
a. Administration of 10% dextrose (D10W) in one-third saline solution IV.
b. Antiemetics.
c. Ringer's Lactate solutions.
d. Hemodialysis to correct azotemia.
e. Saline fluid replacement with appropriate potassium administration.

72.Which of the following types of shock is high pulmonary wedge pressure?


a. Hypovolemic shock.
b. Cardiogenic shock.
c. Early septic shock.
d. Late septic shock.
e. Neurogenic shock.
P a g e 18 | 26
SurgiNotes 2nd Ed. (2022)

73.Which of the following techniques does NOT provide a definitive airway?


a. Cricothyroidotomy.
b. Tracheostomy.
c. Nasotracheal tube.
d. Laryngeal mask airway.
e. Endotracheal tube.

74.What is the correct management of commonest acid-base imbalance seen in


long standing or sever hemorrhagic shock?
a. Intravenous sodium bicarbonate.
b. Component blood therapy.
c. Increased fluid administration.
d. Vasopressors.
e. Hyperventilation.

75.The anion gap will increase with an increase in the plasma concentration of:
a. Sodium.
b. Potassium.
c. Chloride.
d. Bicarbonate.
e. Lactate.

76.A 65-year-old man undergoes trans-sphenoidal surgery for a pituitary macro-


adenoma. On the first postoperative day he is noted to be confused. The
MOST likely cause is:
a. Hyperkalemia.
b. Hyperuricemia.
c. Hypoglycemia.
d. Hyponatremia.
e. Hypoxia.
P a g e 19 | 26
SurgiNotes 2nd Ed. (2022)

77.A man suffers a brachial injury. On examination, he has a Horner's syndrome


in association with upper limb paralysis. The Horner's syndrome suggests
involvement of which nerve roots?
a. C2 & C3.
b. C4 & C5.
c. C6 & C7.
d. C8 & T1.
e. T2 & T3.

78.A patient with a life threatening pulmonary embolus is receiving heparin. She
developed serious vaginal bleeding and a major retroperitoneal haematoma
after 5 days of heparin therapy. What is the recommended treatment?
a. Reverse heparin and evacuate the hematoma.
b. Reverse heparin by protamine sulphate and insert a vena caval filter.
c. stop heparin and closely observe the patient.
d. stop heparin, give fresh frozen plasma and start warfarin therapy.
e. Switch to low-dose heparin.

79.What is the main mode of action of heparin?


a. It Increase the level of protein C.
b. It is a cofactor of antithrombin III.
c. It prevents clot retraction.
d. It prevents platelets aggregation.
e. It prevents the synthesis of fibrinogen.

80.You were obliged to perform cholecystectomy for a cirrhotic patient. During


surgery there was excessive bleeding. What would you order?
a. Fresh blood.
b. Fresh frozen plasma.
c. Fresh platelets.
d. IV vitamin K.
e. IV factor VIII.
P a g e 20 | 26
SurgiNotes 2nd Ed. (2022)

81.What does increased level of fibrin degradation products (FDP) in the blood
denote?
a. Diminished fibrin synthesis.
b. Diminished platelet count.
c. Diminished platelet function.
d. Disseminated intravascular coagulation (DIC).
e. Heparin overdose.

82.Low molecular weight heparin (LMWH) compared with unfractionated


intravenous heparin (UFH) for the treatment of pulmonary embolism is
associated with a:
a. Higher incidence of all-cause mortality.
b. Higher incidence of recurrent symptomatic venous thromboembolism at
the end of treatment.
c. Higher incidence of recurrent symptomatic venous thromboembolism at
3 months.
d. Lower incidence of major bleeding complications.
e. Lower risk of heparin-induced thrombocytopenia.

83.A cirrhotic patient with abnormal haemostatic studies requires an urgent


cholecystectomy. A transfusion of fresh frozen plasma is planned to minimize
the risk of bleeding. What is the optimal timing of this transfusion?
a. In the recovery room.
b. Intraoperatively.
c. On call to surgery.
d. The night before surgery.
e. Two days before surgery.

84.Which of the following mechanisms initiates the normal hemostatic


mechanism?
a. Fibrin formation.
b. Fibrin stabilization.
c. Fibrinolysis.
d. Platelet plug formation.
e. Vascular constriction.
P a g e 21 | 26
SurgiNotes 2nd Ed. (2022)

85.A patient who received 8 units of blood during an operation developed diffuse
oozing of blood at the site of the wound. Which of the following tests will be
helpful for the diagnosis of the etiology of this ooze?
a. Platelet count.
b. Bone marrow biopsy.
c. Liver-spleen scan.
d. Factor VIII assay.
e. Smear for Howell-Jolly bodies.
86.A 5-year-old boy slipped and hurted his right knee while walking. He presents
with a tender, swollen, warm knee with significant hemarthrosis. His PT is 12
(normal 13 second), PTT is over 100 (normal 25 second), platelet count is
300,000/ul and bleeding time is normal. Initial management should consist of
which of the following?
a. Fresh frozen plasma.
b. Aspiration of knee.
c. Factor VIII concentrate.
d. Passive exercise.
e. Long-leg cast.
87.Which of the following denotes a hemolytic transfusion reaction during
anesthesia?
a. Shaking, chills and muscle spasms.
b. Fever and oliguria.
c. Hyperpyrexia and hypotension.
d. Tachycardia.
e. Bleeding and hypotension.
88.A 22-year-old man is brought into the emergency department in profound
shock after a full from the fourth floor of a building. After resuscitation, small
bowel resection and hepatic segmentectomy are performed at laparotomy.
He receives 15 units of packed RBCs, 4 units of fresh-frozen plasma and 8 L of
Ringer's lactate. On closure, diffuse oozing of blood is noted. What is the MOST
likely cause?
a. Hepatic failure.
b. Hypersplenism.
c. Platelet deficiency.
d. Factor IX (Christmas factor) deficiency.
e. Congenital hypoprothrombinemia.
P a g e 22 | 26
SurgiNotes 2nd Ed. (2022)

89.Which of the following statements is TRUE about Cushing's disease?


a. ACTH is high.
b. Blood sugar is low.
c. It is due to a pituitary adenoma in 10% of cases.
d. Serum Na is low.
e. Surgery is not successful in the treatment.

90.A 40-year-old male patient is complaining of persistent headache, recurrent


attacks of palpitation, sweating and visual disturbances. Blood pressure of the
patient is 180/110 mmHg. Which of the following is MOST accurate in the
diagnosis of the problem?
a. 24 hour urinary catecholamines.
b. 24 hour urinary VMA.
c. T scan of abdomen.
d. Plasma aldosterone level.
e. Plasma metanephrine level.

91.What is the MOST common cause of the thyrotoxicosis?


a. Autonomous nodules.
b. Grave's disease.
c. Iodine intake.
d. Subacute thyroiditis.
e. Toxic nodular goiter.

92.Which of the following is the MOST potent stimulus for aldosterone secretion?
a. ACTH.
b. Antidiuretic hormone.
c. Hyperkalemia.
d. Hypernatremia.
e. Renin angiotensin system.
P a g e 23 | 26
SurgiNotes 2nd Ed. (2022)

93.Which of the following is the common sign of Cushing's syndrome?


a. Acne.
b. Hirsutism.
c. Hypertension.
d. Purple striae.
e. Truncal obesity.

94.You have a patient who has pheochromocytoma and his blood pressure is
200/120 mmHg. You are preparing this patient for surgery. What is the
following drugs should be first for the control of blood pressure?
a. Phenoxybenzamine.
b. Propranolol.
c. Nifedipine.
d. Atenolol.
e. Captopril.

95.What is the recommended treatment of acute adrenal insufficiency?


a. Normal saline, potassium and glucose.
b. Hypertonic saline and potassium.
c. Normal saline and potassium.
d. Intravenous mineralocorticoids.
e. Normal saline, glucose and intravenous mineralocorticoids.

96.Which of the following is NOT present in primary hyperparathyroidism?


a. Hypercalcemia.
b. Hyperchloremia.
c. Hypophosphatemia.
d. Low bicarbonate levels.
e. Increased pH.
P a g e 24 | 26
SurgiNotes 2nd Ed. (2022)

97.A 40-year-old women had the anterior lobe of the pituitary removed because
of a tumour. Without postoperative supplements, which of the following
could occur?
a. Failure to produce adequate amounts of thyroxine.
b. Failure to produce parathyroid hormone in response to hypocalcemia.
c. Failure to secrete catecholamines in response to stress.
d. Failure to secrete insulin in hyperglycaemia.
e. Inability to concentrate urine in response to water deprivation.

98.A 30-year-old primigravida complains of headaches, restlessness, sweeting


and tachycardia. She is 18 week pregnant and her blood pressure is 200/120
mmHg. Appropriate workup might include:
a. Exploratory laparotomy.
b. Mesenteric angiography.
c. Head CT scan.
d. Abdominal CT scan.
e. Abdominal ultrasound.

99.A 50-year-old woman present with lethargy, weight gain, cold intolerance and
loss of interest for the past six months. Which is the MOST appropriate initial
investigation?
a. Erythrocyte sedimentation rate estimation.
b. Radioactive isotope scan of thyroid.
c. Thyroid antibodies screen.
d. Thyroid stimulating hormone estimation.
e. Ultrasound scan of thyroid gland.

100. A 48-year-old woman with breast cancer diagnosed five years ago is
admitted with a two week history of back pain, nausea, fatigue and
constipation. Her serum calcium is 14 mg/dl. Which of the following is the
correct first line treatment for this patient?
a. Bisphosphonate infusion.
b. Calcitonin infusion.
c. Dextrose/insulin infusion.
d. Forced diuresis with furosemide.
e. Intravenous fluids.
P a g e 25 | 26
SurgiNotes 2nd Ed. (2022)

Egyptian Board Of General Surgery


1st Part Exam – Paper (1) MCQ "ANSWER"
February 2017

1 e 26 a 51 a 76 d
2 b 27 e 52 e 77 d
3 b 28 e 53 c 78 b
4 c 29 c 54 c 79 b
5 b 30 d 55 c 80 b
6 d 31 a 56 d 81 d
7 a 32 e 57 d 82 e
8 a 33 e 58 a 83 c
9 e 34 d 59 c 84 e
10 c 35 d 60 e 85 a
11 e 36 d 61 a 86 c
12 b 37 b 62 d 87 e
13 d 38 b 63 d 88 c
14 c 39 e 64 e 89 a
15 c 40 a 65 d 90 e
16 b 41 d 66 e 91 b
17 c 42 d 67 a 92 e
18 d 43 b 68 b 93 e
19 c 44 d 69 c 94 a
20 e 45 c 70 b 95 e
21 e 46 e 71 e 96 e
22 b 47 d 72 b 97 a
23 b 48 e 73 d 98 e
24 d 49 b 74 c 99 d
25 c 50 a 75 a 100 e

P a g e 26 | 26
SurgiNotes 2nd Ed. (2022)

Egyptian Board Of General Surgery


1st Part Exam – Paper (1) MCQ
February 2018
1. A 34-year-old man is seen in the emergency department with a fibular fracture
following a football match. On examination he is noted to have loss of foot eversion.
Which area of skin should be examined to confirm loss of the cutaneous distribution
of the affected nerve?
a. Along the lateral aspect of the foot.
b. Along the medial aspect of the foot.
c. Between the hallux and the second digit.
d. On the dorsal surface of the foot.
e. On the plantar surface of the foot.
2. Which of the following statement is TRUE regarding surgical excision of the right
submandibular salivary gland?
a. Injury to the lingual nerve would result in loss of sensations to the posterior
one third of the right side of the tongue.
b. Injury to the hypoglossal nerve would result in deviation of the tongue to the
right side on protrusion of the tongue.
c. The great auricular nerve is at a risk of injury.
d. Injury of the marginal mandibular branch of the facial nerve would result in
sensory loss at the angle of the mandible.
e. Injury to the cervical branch of the facial nerve would result in drooping of the
right angle of the mouth.
3. Which of the following is the sensory supply to the skin of the tip of the index finger?
a. Radial nerve only.
b. Radial and median nerves.
c. Median and ulnar nerves.
d. Ulnar nerve only.
e. Median nerve only.
4. Which of the following statements is TRUE regarding the marginal mandibular
nerve?
a. Is a branch of the 5th cranial nerve.
b. Supplies motor function to the strap muscles.
c. May be associated with the submandibular gland.
d. Lies deep to the facial vein.
e. Is most often injured during dissection of the posterior triangle (level 5) of the
neck.
P a g e 1 | 26
SurgiNotes 2nd Ed. (2022)

5. A 74-year-old woman presents to the outpatient clinic with pelvic pain. A CT


scan reveals enlarged para-aortic lymph nodes. These are MOST likely to be
involved in secondary spread from a tumor in which of the following organs?
a. Cervix.
b. Ovary.
c. Uterus.
d. Vagina.
e. Vulva.

6. A 64-year-old woman is reviewed in the emergency department with an acute


right groin swelling, below and lateral to the pubic tubercle. Which of the
following forms the medial boundary of the ring through which this hernia is
protruding?
a. Conjoint tendon.
b. Inferior epigastric artery.
c. Inguinal ligament.
d. Lacunar ligament.
e. Pectineal ligament.

7. A 30-year-old motorcyclist suffers a closed fracture to the mid-shaft of the


tibia. Anterior compartment syndrome could cause loss of sensation:
a. In the first web space.
b. Over the dorsum of the foot.
c. Over the lateral edge of the foot.
d. Over the medial aspect of the hallux.
e. Over the medial malleolus.

8. An 82-year-old man has complete occlusion of his inferior mesenteric artery


on angiography but no symptoms or signs of colonic ischaemia. Which of the
following arteries is the MOST likely additional source of blood supply to the
territory of the inferior mesenteric artery?
a. Left colic.
b. Left gastroepiploic.
c. Middle colic.
d. Splenic.
e. Superior rectal.
P a g e 2 | 26
SurgiNotes 2nd Ed. (2022)

9. A 62-year-old man develops abdominal pain after eating. An arteriogram


reveals absence of blood flow in the celiac artery. Collateral branches supply
the stomach through which of the following?
a. Intercostal arteries.
b. Right renal artery.
c. Superior mesenteric artery.
d. Inferior epigastric artery.
e. Left colic artery.

10.A 21-year-old man has been stabbed in the back of the knee, dividing the
popliteal artery and is undergoing repair via a posterior approach. Which of
the following structures is MOST likely to be encountered first when dissecting
deep from the skin incision?
a. Popliteal artery.
b. Popliteus muscle.
c. Popliteal vein.
d. Tibial nerve.
e. Soleus muscle.

11.An 80-year-old man presents with a prostatic malignancy. Imaging reveals a


sclerotic lesion in a thoracic vertebra. Which is the MOST likely vascular route
of metastatic spread?
a. Cremasteric vein.
b. Inferior mesenteric vein.
c. Internal iliac vein.
d. Testicular vein.
e. Vertebral veins.

12.A 42-year-old woman presents to her doctor with shoulder pain. On


examination, shoulder abduction is weak and painful. Which muscle is likely
to be affected?
a. Infraspinatus.
b. Subscapularis.
c. Supraspinatus.
d. Teres major.
e. Teres minor.
P a g e 3 | 26
SurgiNotes 2nd Ed. (2022)

13.A 32-year-old man is brought to the emergency department following a fall.


An X-ray reveals that he has a fracture of the surgical neck of his humerus.
Neurological examination reveals paraesthesia over the upper lateral arm
(overlying the deltoid muscle). Which nerve is MOST likely to have been
damaged?
a. Axillary.
b. Median.
c. Musculocutaneous.
d. Radial.
e. Ulnar.

14.While assessing an elderly woman with severe neck arthritis you note she has
weakness of finger abduction and adduction. This is MOST likely to be due to
compression of which spinal nerve root of the brachial plexus?
a. Fifth cervical.
b. Sixth cervical.
c. Seventh cervical.
d. Eighth cervical.
e. First thoracic.

15.Wound healing is relatively slowest in which of the following?


a. Eyelid.
b. Lips.
c. Sternum.
d. Stomach.
e. Urinary bladder.

16.Which of the following is TRUE about the catabolic response to trauma?


a. Intravenous hyperalimentation can prevent the catabolic response to
trauma.
b. Liver glycogen is the source of dextrose in the first week.
c. The catabolic response is initiated by the thyroid hormones.
d. The catabolic response is the same regardless of the severity of trauma.
e. There is inevitable loss of muscle mass.
P a g e 4 | 26
SurgiNotes 2nd Ed. (2022)

17.Which of the following can lower the serum potassium level?


a. Calcium gluconate.
b. Dextrose 25%.
c. Ringer's lactate.
d. Saline.
e. Sodium bicarbonate.

18.Which of the following hormones exhibits a diurnal rhythm in its secretion?


a. ACTH.
b. Aldosterone.
c. Noradrenaline.
d. Parathyroid hormone.
e. Prolactin.

19.Which of the following cells is responsible for the secretion of pepsinogen?


a. Chief cells.
b. Enterochromaffin cells.
c. Mucous cells.
d. Paneth cells.
e. Parietal cells.

20.A 60-year-old male patient is suffering from diabetic ketoacidosis. Which of


the following is the MOST important buffer base in the extracellular fluid?
a. Bicarbonate.
b. Haemoglobin.
c. Lactate.
d. Phosphate.
e. Plasma proteins.
P a g e 5 | 26
SurgiNotes 2nd Ed. (2022)

21.The following acid-base data: pH 7.2, PCO2 20 mmHg, HCO3- 8 mmol/L and base
excess -19 mmol/L, would be MOST consistent with:
a. Anxiety.
b. Lobar collapse of the lung.
c. Pyloric obstruction.
d. Septic shock.
e. Starvation.

22.What is the main brain's fuel during prolonged starvation?


a. Amino acid.
b. Glucose.
c. Ketones.
d. Lactose.
e. Short-chain fatty acid.

23.A 45-year-old female patient complains of reflux oesophagitis secondary to a


sliding hiatus hernia. Which of the following decreases the tone of the lower
esophageal sphincter?
a. Fatty meal.
b. Gastric alkalization.
c. Gastric distension.
d. Gastrin.
e. Proteins.

24.A 65-year-old man undergoes trans-sphenoidal surgery for a pituitary macro-


adenoma. On the first postoperative day he is noted to be confused. The
MOST likely cause is:
a. Hyperkalaemia.
b. Hyperuricaemia.
c. Hypoglycaemia.
d. Hyponatraemia.
e. Hypoxia.
P a g e 6 | 26
SurgiNotes 2nd Ed. (2022)

25. Which of the following is TRUE regarding the release of tumor necrosis factor-α?
a. Can be effectively blocked by anti-TNF-α antibodies to halt systemic
inflammatory response syndrome (SIRS).
b. Does not have any beneficial effects in the early phases of the inflammatory
response.
c. Is primarily from leukocytes.
d. Promotes polymorphonuclear (PMN) cell adherence and further cytokines
release.
e. Is always deleterious.

26. A 21-year-old graduate student has a large hypertrophic scar on the lower part
of her face. The patient had sustained a laceration on her face 2 years previously
after injuring her face. Which of the following statements regarding scar revision
is TRUE?
a. Scar maturation refers to the change in size of the wound in the first 1 to 2
months.
b. Scar revision should have been performed in the first 3 months after injury
to minimize fibrosis.
c. Revision should be performed earlier in children than in adults.
d. It corrects undesirable pigmentation.
e. Scar revision should be delayed approximately 1 year to allow maturation.

27. A 25-year-old man presents to the emergency department after being involved
in a road traffic accident. He is conscious, talking, has a blood pressure of 90/70
mmHg and a heart rate of 100 beats/minute. He has sustained a fractured pelvis
and femur. Which of the following is TRUE?
a. His total peripheral resistance is decreased.
b. The discharge rate of his carotid sinus nerves is increased.
c. There is a decrease in renin production.
d. There is an increase in angiotensin II.
e. There is an increase in renal sodium excretion.

28. Which one of the following findings is MOST likely to be present in a patient with
severe magnesium deficiency?
a. Respiratory depression.
b. Bradycardia.
c. Tetany.
d. Hypotension.
e. Loss of patellar reflex Hyperactive reflexes.
P a g e 7 | 26
SurgiNotes 2nd Ed. (2022)

29.Which of the following is often associated with early sepsis?


a. Normal acid-base balance.
b. Metabolic alkalosis.
c. Metabolic acidosis.
d. Respiratory alkalosis.
e. Respiratory acidosis.

30.A 50-year-old male patient was punctured in his foot by a very tiny piece of
glass which could not be removed. Three months later the patient presented
by a firm small nodule at the site of the puncture and this was excised. Which
of the following cells will be predominant on pathological examination of the
removed nodule?
a. Epithelioid cells.
b. Fibroblasts.
c. Lymphocytes.
d. Plasma cells.
e. Polymorphonuclear leucocytes.

31.What does the presence of cytokeratin on immunohistochemical staining


points to?
a. Carcinoma.
b. Ewing's sarcoma.
c. Hodgkin's lymphoma.
d. Melanoma.
e. Soft tissue sarcoma.

32.Which of the following cells is an example of a permanent cell NOT capable of


division?
a. Acinar cells of the pancreas.
b. Colonic mucosal cells.
c. Erythrocytes.
d. Hepatocytes.
e. Osteocytes.
P a g e 8 | 26
SurgiNotes 2nd Ed. (2022)

33.In patients with reflux esophagitis. What does the presence of columnar cells
in the esophageal mucosa represent?
a. Carcinoma in situ.
b. Carcinoma.
c. Dysplasia.
d. Hyperplasia.
e. Metaplasia.

34.Which of the following microscopic features MOST likely indicates that a


neoplasm is malignant?
a. Atypia.
b. Increased nuclear : cytoplasmic ratio.
c. Invasion.
d. Necrosis.
e. Pleomorphism.

35.A 45-year-old man with a long history of ulcerative colitis undergoes


colonoscopy. Biopsy shows that the mucosal architecture is abnormal. The
epithelial cells have enlarged hyperchromatic nuclei and there is failure of
maturation towards the surface. There is no evidence of invasion. Which of
the following pathological processes best fits this histological description?
a. Carcinoma.
b. Dysplasia.
c. Hyperplasia.
d. Hypertrophy.
e. Metaplasia.

36.In the presence of acute inflammation, when an abscess has formed, which of
the following cell types is MOST likely to be primarily involved in phagocytic
activity?
a. Erythrocytes.
b. Lymphocytes.
c. Macrophages.
d. Mast cells.
e. Plasma cells.
P a g e 9 | 26
SurgiNotes 2nd Ed. (2022)

37.Which of the following is the MOST likely cause of pyrexia occurring 48 hours
after an abdominal operation?
a. Chest infection.
b. DVT.
c. Leaking intestinal anastomosis.
d. Pulmonary embolism.
e. Wound infection.

38.Which of the following is TRUE regarding antibiotics?


a. Aminoglycosides may cause hepatotoxicity.
b. Ampicillin in effective against pseudomonas infections.
c. Cephalosporins are rarely used for prophylaxis.
d. Penicillins act by disrupting the peptidoglycan of the bacterial cell wall.
e. Vancomycin is the first choice for treatment of infections with Staph.
aureus.

39.A 45-year-old diabetic male patient complains of severe pain in the thigh.
Examination reveals spreading oedema, swelling and tenderness of the thigh.
A provisional diagnosis of necrotizing fasciitis is made. Which of the following
is MOST important in the treatment?
a. Antifungal agents.
b. Antitoxin.
c. Hyperbaric O2.
d. Immunoglobulins.
e. Wide surgical debridement.

40.A 35-year-old female patient had an operation for perforated appendicitis.


Postoperatively the patient complained of fever and wound pain. There was
tenderness, redness and swelling at the site of the wound. Which of the
following organisms MOST likely to be responsible for this problem?
a. Clostridium botulinum.
b. Clostridium tetani.
c. Escherichia coli.
d. Staphylococcus aureus.
e. Streptococcus pyogenes.
P a g e 10 | 26
SurgiNotes 2nd Ed. (2022)

41.A 40-year-old male patient had renal transplantation and is receiving


immunosuppression. Few days following the operation the patient develop
persistent fever. Which of the following infectious microorganisms is currently
the MOST likely cause of this fever?
a. Candidiasis.
b. Coli sepsis.
c. Cytomegalovirus sepsis.
d. Pneumococcal sepsis.
e. Streptococci.

42.A 30-year-old female patient developed postoperative wound infection after


a thyroidectomy operation. What is the MOST likely causative organism?
a. Bacteroides.
b. E. coli.
c. Proteus.
d. Staph. aureus.
e. Streptococci.

43.Which of the following is the mode of action of amikacin?


a. Blockage of bacterial DNA replication.
b. Competitive inhibition of the enzyme dihydrofolate reductase.
c. Inhibition of bacterial wall synthesis.
d. Inhibition of protein synthesis by ribosomes.
e. Inhibition of translocation of Peptides.

44.The use of prophylactic antibiotics has become commonplace. Which of the


following statement(s) is TRUE concerning the prophylactic use of antibiotics?
a. Prophylactic antibiotics should be administered for all surgical
procedures.
b. Continuing thee antibiotic into the postoperative period has led to
improved results in antibiotic prophylaxis.
c. The appropriate use of prophylactic antibiotics must include the initiation
of the agent prior to the surgical procedure.
d. The prophylactic administration of broad-spectrum agents (Third-
generation cephalosporins) has been shown to be particularly
advantageous.
e. The topical use of antimicrobial agents is of no advantage in the
prophylactic setting.
P a g e 11 | 26
SurgiNotes 2nd Ed. (2022)

45.Which of the following measures is MOST likely to reduce the risk of


postoperative wound infection with MRSA?
a. 5 days of broad spectrum prophylactic antibiotics.
b. Ensure that the patient showers with chlorhexidine wash prior to surgery.
c. A policy of staff handwashing between patients.
d. Screening patients for MRSA carriage prior to surgery.
e. Preoperative shaving the area of incision.

46.What is the oral antibiotic MOST likely to be effective against a pseudomonas


urinary tract infection?
a. Ciprofloxacin.
b. Trimethoprim-sulfamethoxazole.
c. Cephalexin.
d. Nitrofurantoin.
e. Amoxicillin.

47.An obese 50-year-old man who has had a renal transplant undergoes left
direct hernia repair with mesh under local anesthesia. Medications include
tacrolimus (Prograf) and prednisone. A nasal swab is positive for methicillin-
resistant staphylococcus aureus (MRSA). Which of the following statements is
TRUE?
a. His wound classification is clean.
b. He should receive oral antibiotics for 48 hours postoperatively.
c. His risk of surgical site infection is less than 1%.
d. Mesh is contraindicated in this patient.
e. The wound should be irrigated with vancomycin.

48.Compared with wet to dry dressing changes, negative pressure wound therapy
is associated with all of the following EXCEPT:
a. Increased local blood flow.
b. Stimulation of granulation tissue.
? Wrong Q.
c. Decreased tissue edema.
d. Decreased bacterial counts.
e. Increased rate of wound healing.
P a g e 12 | 26
SurgiNotes 2nd Ed. (2022)

49.A 70-year-old woman is admitted to hospital with a severe respiratory tract


infection. Five days later she develops diarrhoea. On flexible sigmoidoscopy
patchy raised yellow plaques are seen in the sigmoid colon. Which of the
following is the MOST likely causative organism?
a. Clostridium difficile.
b. Escherichia coli.
c. Proteus mirabilis.
d. Pseudomonas aeruginosa.
e. Streptococcus faecalis.

50.Which of the following is NOT characteristic of aminoglycosides?


a. Active against a broad spectrum of gram-negative aerobes and useful for
synergy against some gram-positive cocci.
b. Emergence of resistant bacterial strains.
c. Narrow margin between therapeutic and toxic blood levels.
d. Nephrotoxicity, ototoxicity and neuromuscular paralysis.
e. Excellent activity in abscesses in which gram-negative organisms are
involved.

51.Which of the following cells is responsible for cellular mediated immunity?


a. B-lymphocytes.
b. Dendritic cells.
c. Monocytes.
d. Natural killer cells.
e. T-lymphocytes.

52.Which of the following cancers is MOST common in organ transplant


recipients?
a. Bronchogenic carcinoma.
b. Lung cancer.
c. Pancreatic cancer.
d. Prostatic cancer.
e. Skin cancer.
P a g e 13 | 26
SurgiNotes 2nd Ed. (2022)

53.A 40-year-old male patient underwent a renal transplant from a cadaveric


donor. Immediately after finishing the vascular anastomosis, the kidney
became cyanotic and flaccid. Histological examination revealed deposition of
immunoglobulins and complement in the vessels walls. What is the
immunological background of the problem?
a. Donor cytotoxic T-lymphocytes directed against the host antigens.
b. Donor natural killer cells directed against host antigens.
c. Host natural killer cells against donor antigens.
d. Preformed donor antibodies against the host antigens.
e. Preformed host antibodies against the donor antigens.

54.Three months after orthotopic liver transplantation for cirrhosis, a 42-year-old


man develops fever, diarrhea and crampy abdominal pain. Clostridium difficile
assay is negative. The diagnosis is best obtained by:
a. Stool for ova and parasites.
b. Peripheral blood smear.
c. Urinary antigen assay.
d. Liver biopsy.
e. Colonoscopy with mucosal biopsies.

55.Which of the following is commonest vascular complication which may follow


hepatic living related donor transplantation?
a. Portal vein stenosis.
b. Hepatic artery thrombosis.
c. Hepatic artery stenosis.
d. Inferior vena cava thrombosis.
e. Hepatic vein stenosis.

56.Which of the following statements regarding rejection of solid organ


transplants is TRUE?
a. Hyperacute rejection begins in the operating room with reperfusion of
the transplanted organ.
b. Liver transplants are especially susceptible to hyperacute rejection.
c. Most immunosuppressive medications are used to prevent chronic
rejection.
d. The major cause of graft failure is acute rejection.
e. Chronic rejection is characterized histologically by lymphocyte
infiltration.
P a g e 14 | 26
SurgiNotes 2nd Ed. (2022)

57.Five days after an uneventful cholecystectomy, an asymptomatic middle aged


woman is found to have a serum sodium level of 120 mEq/L. What is the
proper management?
a. Administration of hypertonic saline solution.
b. Administration of Ringer's lactate solution.
c. Aggressive diuresis with furosemide.
d. Plasma ultra-filtration.
e. Restriction of free water.

58.A 65-year-old diabetic woman with chronic steroid-dependent bronchospasm


had hemicolectomy for a perforated cecum. The patient was intubated and
was maintained on broad-spectrum antibiotics, renal-dose dopamine and a
rapid steroid taper. On postoperative day 2 she develop a fever of 39.2 °C
(102.5 °F), hypotension and lethargy. Laboratory investigations revealed
hypoglycemia and hyperkalemia. What is the MOST likely diagnosis of this
acute event?
a. Adrenal insufficiency.
b. Diabetic ketoacidosis.
c. Hypovolemia.
d. Insulin overdose.
e. Sepsis.

59.Which of the following is a clinical feature of a major pulmonary embolism?


a. Bradycardia.
b. Collapsed neck veins.
c. Haemoptysis.
d. High fever.
e. Tachypnea.

60.A 24-year-old woman has acute renal failure following postpartum


hemorrhage. Laboratory studies showed serum glucose 150 mg/dl, sodium
135 mEq/L, potassium 6.5 mEq/L, chloride 105 mEq/L and bicarbonate 15
mEq/L. Which of the following is recommended?
a. Decrease potassium chloride to 10 mEq/L.
b. Intravenous 0.9% sodium chloride.
c. 100 ml of 50% glucose water with 10 U insulin.
d. Intravenous calcitonin.
e. Intravenous magnesium sulfate.
P a g e 15 | 26
SurgiNotes 2nd Ed. (2022)

61.Following an operation and extubation a patient was restless. His arterial


blood gases revealed: pH 7.36, PaO2 65 mmHg, PaCO2 55 mmHg and PaCO3- 30
mmol/L. The physiologic status can best be described as which of the
following?
a. Respiratory alkalosis.
b. Respiratory acidosis.
c. Metabolic acidosis.
d. Metabolic alkalosis.
e. Combined respiratory and metabolic acidosis.

62.A 68-year-old man has a history of myocardial infarction. He undergoes


uneventful left hemicolectomy for carcinoma of the colon. In the recovery
room, he is hypotensive and is given a fluid bolus of 500 ml Ringer's lactate
over 30 minutes. He is intubated and his neck veins are distended. His HR is
130 bpm, his BP is 80/60 mmHg and his urine output is 20 ml over the last
hour. What should be the next step in his management?
a. Administration of Ringer's lactate 500 ml over 1 hour.
b. Administration of dopamine.
c. Insertion of a Swan-Ganz catheter.
d. Administration of lasix.
e. Extubation of the patient.

63.Which of the following metabolic effects is present in shock?


a. Increase in sodium and water excretion.
b. Increase in renal perfusion.
c. Decrease in cortisol levels.
d. Hyperkalemia.
e. Hypoglycemia.

64.A 40-year-old man is confused and restless the second day after upper
abdominal surgery and repair of a hiatus hernia. What is the MOST probable
cause of his condition?
a. Pulmonary embolism.
b. Narcotic overdose.
c. Pulmonary atelectasis.
d. Electrolyte imbalance.
e. Starvation ketosis.
P a g e 16 | 26
SurgiNotes 2nd Ed. (2022)

65.A 70-year-old man with chronic obstructive pulmonary disease is admitted for
elective hemicolectomy. What is a preoperative arterial blood gas analysis
likely to show?
Arterial PCO2 Bicarbonate.
a. Decreased Decreased.
b. Decreased Increased.
c. Decreased Normal.
d. Increased Decreased.
e. Increased Increased.
66.A 60-year-old man had undergone exploratory laparotomy for perforated
gastric ulcer with severe peritoneal contamination. Six hours after surgery, he
is tachycardic, hypertensive and has shallow respirations. Intubation and
institution of ventilatory support is indicated in the presence of which of the
following?
a. Respiratory rate of 23 breaths/min.
b. PaCO2 of 45 mmHg.
c. PaO2 of 55 mmHg on room air.
d. Heart rate of 140 bpm.
e. BP of 100/70 mmHg.
67.A seventy year-old man presents to the ER with several bone fractures and a
ruptured spleen after failing from a height. Which of the following best
represents his response to injury?
a. Decreased liver gluconeogenesis.
b. Inhibition of skeletal muscle breakdown by interleukin-1 and tumor
necrosis factor (TNF, cachectin).
c. Decreased urinary nitrogen loss.
d. Decreased glutamine consumption by fibroblasts, lymphocytes and
intestinal epithelial cells.
e. Hepatic synthesis of acute-phase reactants.
68.The arterial blood gases of a patient who had an operation for ruptured aortic
aneurysm are: pH 7.54, PO2 100 mmHg, PCO2 30 mmHg and HCO3- 30 mmol/L.
What is the acid base disturbance?
a. Respiratory acidosis.
b. Respiratory alkalosis.
c. Metabolic alkalosis.
d. Combined respiratory and metabolic alkalosis.
e. Compensated respiratory acidosis.

P a g e 17 | 26
SurgiNotes 2nd Ed. (2022)

69.Which of the following types of shock is associated with high pulmonary


wedge pressure?
a. Hypovolemic shock.
b. Cardiogenic shock.
c. Early septic shock.
d. Late septic shock.
e. Neurogenic shock.

70.A common drug that can trigger an episode of malignant hyperthermia:


a. Propofol.
b. Dantrolene.
c. Ketamine.
d. Succinylcholine.
e. Neostigmine.

71.Choose the TRUE statement about septic shock:


a. Gram-positive organisms do not cause septic shock because they lack the
cell-wall endotoxin.
b. Leucopenia is not a feature of septic shock.
c. The second most common source of gram-negative bacteremia is the
urinary tract.
d. Escherichia coli is the second most common organism identified in gram-
negative bacteremia.
e. The toxic effects of gram-negative endotoxin appear to be the result of
the abnormal activation of normal physiologic pathways.

72.What is the correct management of the commonest acid-base imbalance seen


in long standing or severe hemorrhagic shock?
a. Intravenous sodium bicarbonate.
b. Component blood therapy.
c. Increased fluid administration.
d. Vasopressors.
e. Hyperventilation.
P a g e 18 | 26
SurgiNotes 2nd Ed. (2022)

73. A 38-year-old man in end-stage renal failure resulting from polycystic kidney
disease receives a cadaveric renal transplant. Good renal function is established
but four weeks later deteriorates, the serum creatinine rising by 25%. Which of
the following processes is MOST likely to be responsible for this deterioration?
a. B-cell mediated rejection.
b. Circulating immune complex disease.
c. IgG antibody mediated rejection.
d. Post-transplant lymphoproliferative disorder.
e. T-cell mediated rejection.
74. A 78-year-old woman with emphysema receiving 28% oxygen by mask has the
following blood gas results:
PH PO2 PaCO2 Bicarbonate Base excess
Finding: 7.28 70 mmHg 48 mmHg 36 mmol/L +5
Normal: 7.35-7.45 90-110 35-45 22-26 -2 to +2
The MOST likely interpretation is:
a. Mixed respiratory and metabolic acidosis.
b. Partially compensated metabolic acidosis.
c. Partially compensated respiratory acidosis.
d. Uncompensated metabolic acidosis.
e. Uncompensated respiratory acidosis.
75. An elderly diabetic patient who has acute cholecystitis is found to have a serum
sodium level of 122 mEq/L and blood glucose of 600 mg/dl. After correction of
the glucose concentration to 120 mg/dl with insulin, the serum sodium
concentration would:
a. Decrease significantly unless patient receive normal saline.
b. Decrease transiently but return to 122 mEq/L.
c. Remain essentially unchanged.
d. Increase to normal range without specific therapy.
e. Increase to hypernatremic level.
76. A 60-year-old man with a past history of angina undergoes an uncomplicated
operation for an inguinal hernia. Postoperatively he is found to be hypotensive,
tachycardic and has a raised jugular venous pressure. What is the MOST likely
underlying cause of his hypotension?
a. Reduced afterload.
b. Reduced parasympathetic tone.
c. Reduced preload.
d. Reduced stroke volume.
e. Reduced sympathetic tone.
P a g e 19 | 26
SurgiNotes 2nd Ed. (2022)

77.Which of the following are effects of epinephrine in response to injury?


a. It enhances the adherence of leukocytes to vascular endothelial
membranes.
b. It stimulates the release of aldosterone.
c. It inhibits the secretion of thyroid hormones.
d. It increases glucagon secretion.
e. It decreases lipolysis in adipose tissue.

78.A 55-year-old male patient is receiving 150 mg of aspirin daily because he has
coronary artery disease. Which of the following tests will be affected this
patient?
a. Bleeding time.
b. Coagulation time.
c. INR.
d. PTT.
e. Thrombin time.

79.Which of the following is a contraindication to heparin therapy?


a. Closed head injury two weeks ago.
b. Heparin-induced thrombocytopenia.
c. Subclavian vein thrombosis.
d. Superior mesenteric artery embolism.
e. Third trimester of pregnancy.

80.What is the main mode of action of heparin?


a. It increases the level of protein C.
b. It is a cofactor of antithrombin III.
c. It prevents clot retraction.
d. It prevents platelets aggregation.
e. It prevents the synthesis of fibrinogen.
P a g e 20 | 26
SurgiNotes 2nd Ed. (2022)

81.What is the principal mechanism of the antithrombotic action of administering


aspirin in low dose?
a. Induction of capillary vasodilatation.
b. Induction of endothelial cell prostacyclin production.
c. Induction of endothelial heparin production.
d. Inhibition of factor V production by the liver.
e. Inhibition of platelet production of thromboxane A2.
82.You were obliged to perform cholecystectomy for a cirrhotic patient. During
surgery there was excessive bleeding. What would you order?
a. Fresh blood.
b. Fresh frozen plasma.
c. Fresh platelets.
d. IV vitamin K.
e. IV factor VIII.
83.What does increased level of fibrin degradation products (FDP) in the blood
denote?
a. Diminished fibrinogen synthesis.
b. Diminished platelet count.
c. Diminished platelet function.
d. Disseminated intravascular coagulation (DIC).
e. Heparin overdose.
84.A 53-year-old woman has acute onset of right calf pain, swelling and
tenderness. Duplex scanning demonstrates acute deep venous thrombosis of
the right femoral-popliteal veins. Intravenous unfractionated heparin therapy
is begun. On postoperatively day 9, she has the acute onset of shortness of
breath, hypoxemia and chest pain. Computed tomographic (CT) angiography
demonstrates multiple, bilateral pulmonary emboli. In addition, her platelet
count has fallen to 50,000. What is the definitive management of this
problem?
a. Administration of low molecular heparin and placing an inferior vena cava
filter.
b. Continuing unfractionated heparin and beginning of a direct thrombin
inhibitor (argatroban).
c. Continuing unfractionated heparin and immediate platelet transfusion.
d. Discontinuing unfractionated heparin.
e. Discontinuing unfractionated heparin and beginning low molecular
weight heparin.
P a g e 21 | 26
SurgiNotes 2nd Ed. (2022)

85.A cirrhotic patient with abnormal haemostatic studies requires an urgent


cholecystectomy. A transfusion of fresh frozen plasma is planned to minimize
the risk of bleeding. What is the optimal timing of this transfusion?
a. In the recovery room.
b. Intraoperatively.
c. On call to surgery.
d. The night before surgery.
e. Two days before surgery.

86.Which of the following mechanisms initiates the normal haemostatic


mechanism?
a. Fibrin formation.
b. Fibrin stabilization.
c. Fibrinolysis.
d. Platelet plug formation.
e. Vascular constriction.

87.After undergoing a transurethral resection of the prostate, a 65-year-old man


experiences excessive bleeding attributed to fibrinolysis. It is appropriate to
administer which of the following?
a. Heparin.
b. Warfarin (coumadin).
c. Volume expanders and cryoprecipitate.
d. Aminocaproic acid (amicar).
e. Fresh-frozen plasma and vitamin K.

88.A 50-year-old female patient has chronic renal failure and has been
maintained an chronic dialysis. The patient underwent cholecystectomy. Post-
operatively she had severe bleeding. What is the MOST likely cause for this
bleeding?
a. Elevated PT.
b. Elevated PTT.
c. Low platelet count.
d. Decreased platelet aggregation.
e. Sepsis.
P a g e 22 | 26
SurgiNotes 2nd Ed. (2022)

89. A 22-year-old man is brought into the emergency department in profound shock
after a fall from the fourth floor of a building. After resuscitation, small bowel
resection and hepatic segmentectomy are performed at laparotomy. He receives
15 units of packed RBCs, 4 units of fresh-frozen plasma and 8 L of Ringer's lactate.
On closure, diffuse oozing of blood is noted. What is the MOST likely cause?
a. Hepatic failure.
b. Hypersplenism.
c. Platelet deficiency.
d. Factor IX (Christmas factor) deficiency.
e. Congenital hypoprothrombinemia.
90. A mother mentions that her 8-year-old boy gets recurrent attacks of
hemarthrosis following a minimal trauma. Investigations revealed normal
platelet count and prothrombin time, but the partial thromboplastin time is
prolonged. Which of the following statements regarding this clinical condition is
TRUE?
a. There is an underlying liver problem.
b. The sisters of this boy are usually having the same problem.
c. The boy has had repeated episodes of epistaxis.
d. There is no family history in this condition.
e. Transfusion of factor VIII concentrate is helpful.
91. A 70-year-old female patient is receiving warfarin because she has AF and had
previous thrombosis. Which of the following statements regarding warfarin is
correct?
a. The dose of warfarin is adjusted according to the partial thromboplastin
time.
b. Warfarin takes about 8 hours to exert its effects.
c. Warfarin acts by inhibiting factor XII.
d. If the patient is also taking aspirin, the dose of warfarin should be reduced.
e. Protamine sulphate is the antidote to warfarin.
92. A 12-year-old boy with-known factor VIII deficiency has a painful, swollen,
immobile right knee. The clinician suspects hemarthrosis. Therapeutic options
include which of the following?
a. Immediate aspiration and compression dressings to prevent cartilage
necrosis.
b. Compression dressings and immobilization to prevent further bleeding.
c. Immediate aspiration after appropriate factor VIII replacement therapy.
d. Initial trial of factor VIII therapy, compression dressings, cold packs and rest
followed by active range-of-motion exercises.
e. Factor VIII replacement followed by arthrotomy.
P a g e 23 | 26
SurgiNotes 2nd Ed. (2022)

93.The MOST useful laboratory test to assess both risk of bleeding and response
to therapy in patients with uremia is:
a. Bleeding time.
b. Platelet count.
c. INR.
d. APTT.
e. Thrombin time.

94.Which of the following statements is TRUE about Cushing's disease?


a. ACTH is high.
b. Blood sugar is low.
c. It is due to a pituitary adenoma in 10% of cases.
d. Serum Na is low.
e. Surgery is not successful in the treatment.

95.A 30-year-old female patient complains of fatigue, generalized bony aches and
depression. The patient gives a history of 2 previous operations for urinary
stones. Which of the following laboratory tests is MOST accurate in the
diagnosis?
a. 24 hours urinary calcium.
b. Ionizable serum calcium.
c. Parathormone hormone.
d. Plasma chloride.
e. Serum phosphate.

96.A 33-year-old woman is scheduled for elective cholecystectomy. Preoperative


evaluation shows the presence of mild to moderate hypothyroidism. What is
the MOST appropriate policy for this patient?
a. Postpone surgery until an euthyroid state is achieved.
b. Proceed with surgery if severe clinical symptoms are not present.
c. Proceed with surgery while beginning treatment with L-thyroxin.
d. Proceed with surgery while beginning treatment with thion-amides.
e. Proceed with surgery with the knowledge that minor perioperative
complications could develop.
P a g e 24 | 26
SurgiNotes 2nd Ed. (2022)

97.A 23-year-old woman undergoes total thyroidectomy for carcinoma of the


thyroid gland. On the second postoperative day, she begins to complain of
tingling sensation in her hands. She appears quite anxious and later complains
of muscle cramps. What is the initial therapy?
a. 10 ml of 10% magnesium sulfate intravenously.
b. 22-dihydrotachysterol orally.
c. Continuous infusion of calcium gluconate.
d. Oral calcium gluconate.
e. Oral vitamin D.

98.Twelve hours after having undergone a subtotal thyroidectomy, a 30-year-old


woman develops agitation and difficulty in breathing. Examination reveals
tachycardia and anterior cervical swelling. The surgical dressing is dry. What is
the MOST appropriate treatment?
a. Insertion of an endotracheal tube.
b. Re-opening of the cervical wound at bed.
c. Determination of the serum calcium concentration.
d. Administration of morphine.
e. Administration of O2 by nasal catheters.

99.You have a patient who has pheochromocytoma and his blood pressure is
200/120 mmHg. You are preparing this patient for surgery. Which of the
following drugs should be used first for the control of blood pressure?
a. Phenoxybenzamine.
b. Propranolol.
c. Nifedipine.
d. Atenolol.
e. Captopril.

100. What is the recommended treatment of acute adrenal insufficiency?


a. Normal saline, potassium and glucose.
b. Hypertonic saline and potassium.
c. Normal saline and potassium.
d. Intravenous mineralocorticoids.
e. Normal saline, glucose and intravenous glucocorticoids.
P a g e 25 | 26
SurgiNotes 2nd Ed. (2022)

Egyptian Board Of General Surgery


1st Part Exam – Paper (1) MCQ "ANSWER"
February 2018

1 d 26 e 51 e 76 d
2 b 27 d 52 e 77 d
3 e 28 c 53 e 78 a
4 c 29 d 54 e 79 b
5 b 30 a 55 b 80 b
6 d 31 a 56 a 81 e
7 a 32 c 57 e 82 b
8 c 33 e 58 a 83 d
9 c 34 c 59 e 84 a
10 d 35 b 60 c 85 c
11 e 36 c 61 b 86 e
12 c 37 a 62 c 87 e
13 a 38 d 63 d 88 d
14 e 39 e 64 c 89 c
15 c 40 c 65 e 90 e
16 e 41 c 66 c 91 d
17 e 42 d 67 e 92 d
18 a 43 d 68 d 93 a
19 a 44 b 69 b 94 a
20 a 45 c 70 d 95 b
21 d 46 a 71 e 96 a
22 c 47 a 72 c 97 c
23 a 48 ? Wrong Q. 73 e 98 b
24 d 49 a 74 c 99 a
25 d 50 e 75 d 100 e

P a g e 26 | 26
SurgiNotes 2nd Ed. (2022)

Egyptian Board Of General Surgery


1st Part Exam – Paper (1) MCQ
July 2018

P a g e 1 | 26
SurgiNotes 2nd Ed. (2022)

5. A 27-year-old man is admitted to the emergency department after a car crash.


Physical examination reveals weakness in medial rotation and adduction of
the humerus. Which of the following nerve was MOST probably injured?
a. Thoracodorsal.
b. Axillary.
c. Dorsal scapular.
d. Spinal accessory.
e. Radial.
6. A 32-year-old male is admitted to the emergency department with groin pain.
Examination reveals that the patient has an indirect inguinal hernia. Which of
the following nerves is compressed by the herniating structure in the inguinal
canal to give the patient pain?
a. Iliohypogastric.
b. Lateral femoral cutaneous.
c. Ilioinguinal.
d. Subcostal.
e. Pudendal.
7. A 44-year-old man is admitted to the emergency department with excessive
vomiting and dehydration. Radiographic images demonstrate that part of the
bowel is being compressed between the abdominal aorta and the superior
mesenteric artery. Which of the following intestinal structures is MOST likely
being compressed?
a. Second part of duodenum.
b. Transverse colon.
c. Third part of duodenum.
d. First part of duodenum.
e. Jejunum.
8. A 34-year-old man is undergoing an emergency appendectomy. After the
appendectomy has been performed successfully, the patient undergoes an
exploratory laparoscopy. Which of the following anatomic features are the
MOST useful to distinguish the jejunum from the ileum?
a. Jejunum has thinner walls compared with the ileum.
b. Jejunum has less mesenteric fat compared with the ileum.
c. Jejunum has more numerous vascular arcades compared with the ileum.
d. Jejunum has more numerous lymphatic follicles beneath the mucosa
compared with the ileum.
e. Jejunum has fewer villi compared with the ileum.

P a g e 2 | 26
SurgiNotes 2nd Ed. (2022)

P a g e 3 | 26
SurgiNotes 2nd Ed. (2022)

14.The femoral canal to a point of weakness in the abdominal wall. Which of the
following statements is the MOST accurate regarding the anatomy of the
femoral canal?
a. The femoral artery forms the anterior border of the femoral canal.
b. The formal vein forms the lateral border.
c. The inguinal ligament lies posterior to the canal.
d. The pectineal pant of the inguinal ligament is anterior to the canal.
e. The pectineal ligament forms the medial border.

15.Which of the following is consistently the largest artery to the stomach?


a. Left gastric artery.
b. Right gastric artery.
c. Left gastroepiploic artery.
d. Right gastroepiploic artery.
e. Short gastric vessels.

16.Wound healing is relatively slowest in which of the following?


a. Eyelid.
b. Lips.
c. Sternum.
d. Stomach.
e. Urinary bladder.

P a g e 4 | 26
SurgiNotes 2nd Ed. (2022)

17.Which of the following stimulates glycogenesis?


a. Aldosterone.
b. Cortisol.
c. Glucagon.
d. Insulin.
e. Thyroxine.

18.Which of the following can lower the serum potassium level?


a. Calcium gluconate.
b. Dextrose 25%.
c. Ringer's lactate.
d. Saline.
e. Sodium bicarbonate.

19.Which of the following is the MOST potent stimulus for ADH hormone
secretion?
a. Decreased plasma osmolality.
b. Decreased plasma volume.
c. Hypothalamic releasing factor.
d. Increased plasma osmolality.
e. Increased plasma volume.

20.Which of the following is the MOST potent stimulus for contraction of the gall
bladder?
a. Acetylcholine.
b. Cholecystokinin.
c. Intravenous hyperalimentation.
d. Secretin.
e. Vagal stimulus.
P a g e 5 | 26
SurgiNotes 2nd Ed. (2022)

21.A 60-year old male patient is suffering from diabetic ketoacidosis. Which of
the following is the MOST powerful buffer base in the extracellular fluid?
a. Bicarbonate.
b. Haemoglobin.
c. Lactate.
d. Phosphate.
e. Plasma proteins.

22.The following acid-base data: pH 7.2, PCO2 20 mmHg, HCO3- 8 mmol/L and base
excess -19 mmol/L, would be MOST consistent with:
a. Anxiety.
b. Lobar collapse of the lung.
c. Pyloric obstruction.
d. Septic shock.
e. Starvation.

23.Which of the following is the MOST potent stimulus for aldosterone synthesis?
a. ACTH.
b. Antidiuretic hormone (ADH).
c. Hyperkalemia.
d. Hypernatraemia.
e. Renin angiotensin system.

24.The patient whose blood pH is 7.47, whose PCO2 is 31 mmHg in the arterial
blood and whose levels of bicarbonate ion in arterial blood is 23 mEq/L is in:
a. Compensated metabolic alkalosis.
b. Uncompensated respiratory acidosis.
c. Uncompensated respiratory alkalosis.
d. Uncompensated metabolic acidosis.
e. Uncompensated metabolic alkalosis.
P a g e 6 | 26
SurgiNotes 2nd Ed. (2022)

25.A 25-year old man presents to the emergency department after being involved
in a road traffic accident. He is conscious, talking, has a blood pressure of
90/70 mmHg and heart rate of 100 beats/minute. He has sustained a fracture
pelvis and femur. Which of the following is TRUE?
a. His total peripheral resistance is decreased.
b. The discharge rate of his carotid sinus nerves is increased.
c. There is a decrease in renin production.
d. There is an increase in angiotensin II.
e. There is an increase in renal sodium excretion.

26.Which of the following is the MOST important stimulus for triggering the
endocrine response after injury?
a. Afferent nerve stimuli from the injured area.
b. Hypovolemia.
c. Tissue acidosis.
d. Local wound factors.
e. Temperature changes.

27.A 65-year-old man undergoes trans-sphenoidal surgery for a pituitary macro-


adenoma. On the first postoperative day he is noted to be confused. The
MOST likely cause is:
a. Hyperkalemia.
b. Hyperuricemia.
c. Hypoglycemia.
d. Hyponatremia.
e. Hypoxia.

28.All of the following activate the sympathoadrenal and hypothalamic-pituitary


axis during stress or injury EXCEPT:
a. Pain.
b. Hypovolemia.
c. Acidosis.
d. Hypercapnia.
e. Acetylcholine.
P a g e 7 | 26
SurgiNotes 2nd Ed. (2022)

29.Which pancreatic islet cell type produces a hormonal peptide to stimulate


glycogenolysis and gluconeogenesis?
a. Alpha cell.
b. Beta cell.
c. Delta cell.
d. F cell.
e. PP cell.

30.Parathyroid hormone is important for regulation of calcium homeostasis,


which of the following best describes the hormone?
a. Accelerates renal hydroxylation of vitamin D.
b. Directly accelerates calcium absorption from the intestine.
c. Is a steroid hormone.
d. Is secreted under the control of the anterior pituitary.
e. Increases plasma phosphate concentration in a normal adult.

31.A patient who presented with severe diarrhea as a result of clostridium


difficile develops a metabolic acidosis. This is MOST likely due to loss of which
one of the following ions in the diarrhea?
a. Cl-.
b. HCO3-.
c. K+.
d. NH4+.
e. OH-.

32.A 15-year-old female patient presented by enlargement of the upper deep


cervical lymph nodes which had been present for 4 months. The patient did
not respond to multiple courses of antibiotics. The temperature was 37.8 oC.
The lymph nodes were matted together and they were slightly tender. Which
of the following cells will be predominant on histopathological examination?
a. Epithelioid cells.
b. Fibroblasts.
c. Lymphocytes.
d. Plasma cells.
e. Polymorphonuclear leucocytes.
P a g e 8 | 26
SurgiNotes 2nd Ed. (2022)

33.In patients with reflux esophagitis. What does the presence of columnar cells
in the esophageal mucosa represent?
a. Carcinoma in situ.
b. Carcinoma.
c. Dysplasia.
d. Hyperplasia.
e. Metaplasia.

34.A patient who has hydatid cyst of the liver will have increased percentage of
which cells in his blood count:
a. Basophils.
b. Eosinophils.
c. Lymphocytes.
d. Monocytes.
e. Neutrophils.

35.With regard to protein loss after injury, which of the following statements is
TRUE?
a. It can be prevented by total parenteral nutrition.
b. It occurs primarily from skeletal muscle.
c. It occurs primarily from the site of injury.
d. It results from impaired synthesis.
e. It results from significant decrease intake.

36.A 45-year-old man with a long history of ulcerative colitis undergoes


colonoscopy. Biopsy shows that the mucosal architecture is abnormal. The
epithelial cells have enlarged hyperchromatic nuclei and there is failure of
maturation towards the surface. There is no evidence of invasion. Which of
the following pathological processes best fits this histological description?
a. Carcinoma.
b. Dysplasia.
c. Hyperplasia.
d. Hypertrophy.
e. Metaplasia.
P a g e 9 | 26
SurgiNotes 2nd Ed. (2022)

37.Which of the following is an anaerobic organism?


a. Bacteroides.
b. E. coli.
c. Klebsiella.
d. Pseudomonas aeruginosa.
e. Staphylococcus aureus.

38.Which of the following operations is a clean contaminated operation?


a. Colectomy for a prepared colon.
b. Drainage of a peri-colic abscess.
c. Inguinal herniectomy.
d. Perforated appendicitis.
e. Thyroidectomy.

39.A 45-year-old diabetic male patient complains of severe pain in the thigh.
Examination reveals spreading oedema, swelling and tenderness of the thigh.
A provisional diagnosis of necrotizing fasciitis is made. Which of the following
is MOST important in the treatment?
a. Antifungal agents.
b. Antitoxin.
c. Hyperbaric O2.
d. Immunoglobulins.
e. Wide surgical debridement.

P a g e 10 | 26
SurgiNotes 2nd Ed. (2022)

P a g e 11 | 26
SurgiNotes 2nd Ed. (2022)

45.An 80-year-old woman who lives in a nursing home and who had just finished
a 10-days course of antibiotics, has abdominal pain and profuse diarrhea. Her
stool is tested and comes back positive for clostridium difficile. What is the
MOST appropriate initial management?
a. Oral vancomycin.
b. Intravenous vancomycin.
c. Metronidazole.
d. Vancomycin enema.
e. Supportive treatment only.

46.Which of the following statement about the usage of antibiotics for the
prevention of surgical site infection is NOT true?
a. Antibiotics should be administrated within 60 minutes of incision.
b. The therapeutic dose of the antibiotic should be administrated
intravenously.
c. Adequate tissue concentrations should be maintained during operation
by re-dosing as necessary.
d. Antibiotics should be continued for 48 hours after operation.
e. Vancomycin is the agent of chronic for patients from nursing homes
undergoing hip replacement.

47.What is the MOST common bacterial organism present in the colon?


a. Bacteroides.
b. Clostridium difficile.
c. Escherichia coli.
d. Salmonella.
e. Streptococci.

48.After being struck by a moving truck, a 23-year-old woman undergoes


splenectomy during diagnostic laparotomy. She leaves the hospital against
medical advice on postoperative day 4, after refusing vaccination. Infection
with which of the following organisms is MOST likely to result in her
developing sepsis?
a. Beta-hemolytic streptococcus.
b. Candida albicans.
c. Clostridium difficile.
d. Escherichia coli.
e. Pseudomonas aeruginosa.
P a g e 12 | 26
SurgiNotes 2nd Ed. (2022)

49.Which of the following organisms is the MOST common pathogenic organism


causing transfusion sepsis?
a. Pseudomonas aeruginosa.
b. Staph. aureus.
c. Strept. pyogenes.
d. E. coli.
e. CI. welchii.

50.Following urinary tract infection associated with extraction of a stone, a 64-


year-old woman developed gram-negative septicemia. Which statement is
TRUE for gram-negative bacterial septicemia?
a. Pseudomonas is the most common organism isolated.
b. Many of the adverse changes can be accounted for the endotoxin release.
c. The cardiac index is low.
d. Central venous pressure (CVP) is high.
e. Endotoxin is mainly a long-chain peptide.

51.A non-immune surgical resident is stuck by a contaminated needle from an


HBsAg-positive source. Which of the following is the correct initial
management?
a. None because the patient doesn’t have active HBV and is immune to HBV.
b. Interferon.
c. Vaccination against HBV.
d. Hepatitis B immunoglobulin (HBIG).
e. Vaccination against HBV and administration of HBIG.

52.A 40-year-old male patient a renal transplant from a cadaveric donor.


Immediately after finishing the vascular anastomosis, the kidney became
cyanotic and flaccid. Histological examination revealed deposition of
immunoglobulins and complement in the vessels walls. What is the
immunological background of the problem?
a. Donor cytotoxic T-lymphocytes directed against the host antigens.
b. Donor natural killer cells directed against host antigens.
c. Host natural killer cells against donor antigens.
d. Preformed donor antibodies against the host antigens.
e. Preformed host antibodies against the donor antigens.

P a g e 13 | 26
SurgiNotes 2nd Ed. (2022)

53.Three months after orthotopic liver transplantation for cirrhosis, a 42-year-old


man develops fever, diarrhea and crampy abdominal pain. Clostridium difficile
assay is negative. The diagnosis is best obtained by:
a. Stool for ova and parasites.
b. Peripheral blood smear.
c. Urinary antigen assay.
d. Liver biopsy.
e. Colonoscopy with mucosal biopsies.

54.Which of the following statements regarding TNF is TRUE?


a. TNF-α is produce only by monocytes and macrophages.
b. Release of TNF is stimulated by exotoxins.
c. TNF is an anabolic stimulant to the host that results in the deposition of
fat.
d. TNF is responsible for the cachexia associated with metastatic disease.
e. TNF expression increase in response to glucocorticoid exposure.

55.The strongest independent risk factor for stress-related mucosal damage and
gastrointestinal bleeding in critically ill patients is:
a. Glucocorticoid administration.
b. Hypotension.
c. Mechanical ventilator support for 48 or more hours.
d. Organ transplantation.
e. Renal failure.

56.A 65-year-old diabetic woman with chronic steroid-dependent bronchospasm


had hemicolectomy for a perforated cecum. The patient was intubated and
was maintained on broad-spectrum antibiotics, renal-dose dopamine and a
rapid steroid taper. On postoperative day 2 she develop a fever of 39.2 °C
(102.5 °F), hypotension and lethargy. Laboratory investigations revealed
hypoglycemia and hyperkalemia. What is the MOST likely diagnosis of this
acute event?
a. Adrenal insufficiency.
b. Diabetic ketoacidosis.
c. Hypovolemia.
d. Insulin overdose.
e. Sepsis.
P a g e 14 | 26
SurgiNotes 2nd Ed. (2022)

57.A 40-year-old male patient developed convulsions 24 hours after having an


operation. His serum sodium is 120 mEq/L. Which of the following is the
appropriate management?
a. Administration of normal saline (0.9%).
b. Administration of hypertonic saline (3%).
c. Emergency hemodialysis.
d. Administration of vasopressin.
e. Administration of lasix 40 mg intravenously (IV).

58.A 68-year-old man has a history of myocardial infarction. He undergoes


uneventful left hemicolectomy for carcinoma of the colon. In the recovery
room, he is hypotensive and is given a fluid bolus of 500 ml Ringer's lactate
over 30 minutes. He is intubated and his neck veins are distended. His HR is
130 bpm, his BP is 80/60 mmHg and his urine output is 20 ml over the last
hour. What should be the next step in his management?
a. Administration of Ringer's lactate 500 ml over 1 hour.
b. Administration of dopamine.
c. Insertion of a Swan-Ganz catheter.
d. Administration of lasix.
e. Extubation of the patient.

59.Which of the following metabolic effects is present in shock?


a. Increase in sodium and water excretion.
b. Increase in renal perfusion.
c. Decrease in cortisol levels.
d. Hyperkalemia.
e. Hypoglycemia.

60. A patient who had gastrectomy 10 days ago developed massive pulmonary
embolism proved by CT angiography. What is the recommended treatment?
a. IV heparin.
b. Fibrinolytic agent.
c. Warfarin.
d. IV acetylsalicylic acid.
e. Corticosteroids.
P a g e 15 | 26
SurgiNotes 2nd Ed. (2022)

61.Which of the following is the commonest cause of ARDS?


a. Sepsis syndrome.
b. Aspiration.
c. Acute pancreatitis.
d. DIC.
e. Fat embolism.

62.A 70-year-old man with chronic obstructive pulmonary disease is admitted for
elective hemicolectomy. What is a preoperative arterial blood gas analysis
likely to show?
Arterial PCO2 Bicarbonate.
a. Decreased Decreased.
b. Decreased Increased.
c. Decreased Normal.
d. Increased Decreased.
e. Increased Increased.

63.A 39-year-old woman is making a slow but adequate recovery after sustaining
a 40% surface area burn injury. On the sixth postoperative day she becomes
unwell. She vomits intermittently, has painless abdominal distention and
starts to hiccup. What is the MOST likely cause of these symptoms?
a. Acute gastric dilatation.
b. Acute intestinal obstruction.
c. Clostridium difficile infection.
d. Faecal impaction.
e. Systemic sepsis.

64.A 78-year-old man with a history of coronary artery disease requests an


elective epigastric hernia repair. Which of the following is valid reason for
delaying the proposed surgery?
a. Coronary artery bypass surgery 3 months earlier.
b. A history of cigarette smoking.
c. Jugular venous distension.
d. Hypertension.
e. Hyperlipidemia.
P a g e 16 | 26
SurgiNotes 2nd Ed. (2022)

65.A 50-year-old man presented with severe repeated vomiting as a result of


gastric outlet obstruction is admitted to the hospital. There is marked
dehydration, with urine output 20 ml/h and the hematocrit is 48%. Initial
treatment for this patient should include which of the following?
a. Administration of 10% dextrose (D10W) in one-third saline solution IV.
b. Antiemetics.
c. Ringer's lactate solution.
d. Hemodialysis to correct azotemia.
e. Saline fluid replacement with appropriate potassium administration.

66.Which of the following types of shock is associated with high pulmonary


wedge pressure?
a. Hypovolemic shock.
b. Cardiogenic shock.
c. Early septic shock.
d. Late septic shock.
e. Neurogenic shock.

67.A common drug that can trigger an episode of malignant hyperthermia:


a. Propofol.
b. Dantrolene.
c. Ketamine.
d. Succinylcholine.
e. Neostigmine.

68.Which of the following techniques does NOT provide a definitive airway?


a. Cricothyroidotomy.
b. Tracheostomy.
c. Nasotracheal tube.
d. Laryngeal mask airway.
e. Endotracheal tube.
P a g e 17 | 26
SurgiNotes 2nd Ed. (2022)

69.The anion gap will increase with an increase in the plasma concentration of:
a. Sodium.
b. Potassium.
c. Chloride.
d. Bicarbonate.
e. Lactate.

70.A 55-year-old male patient is receiving 150 mg of aspirin daily because he has
coronary artery disease. Which of the following tests will be affected in this
patient?
a. Bleeding time.
b. Coagulation time.
c. INR.
d. PTT.
e. Thrombin time.

71.A 30-year-old male patient was admitted to the casualty department due to a
car accident. The patient had fracture of the pelvis and the right femur and he
received 5 liters of blood following which he started to have bleeding from his
nose and mouth. What is the main cause of this bleeding tendency?
a. Decrease in fibrinogen.
b. Decrease in prothrombin.
c. Decrease of calcium.
d. Increased fibrinolytic activity.
e. Platelet depletion.

72.A patient with a life threatening pulmonary embolus is receiving heparin. She
developed serious vaginal bleeding and a major retroperitoneal haematoma
after 5 days of heparin therapy. What is the recommended treatment?
a. Reverse heparin and evacuate the haematoma.
b. Reverse heparin by protamine sulphate and insert a vena caval filter.
c. Stop heparin and closely observe the patient.
d. Stop heparin, give fresh frozen plasma and start warfarin therapy.
e. Switch to low-dose heparin.
P a g e 18 | 26
SurgiNotes 2nd Ed. (2022)

73.What is the main mode of action of heparin?


a. It increases the level of protein C.
b. It is a cofactor of antithrombin III.
c. It prevents clot retraction.
d. It prevents platelets aggregation.
e. It prevents the synthesis of fibrinogen.

74.You were obliged to operate for a strangulated inguinal hernia in a patient


who was receiving 300 mg aspirin daily. During surgery there was excessive
bleeding. What would you advise?
a. Desmopressin.
b. Fresh blood.
c. Fresh frozen plasma.
d. Fresh platelets.
e. IV vitamin K.

75.What is the cause of the prolonged prothrombin time in patients with liver
cirrhosis?
a. Abnormal fibrinolysis.
b. Deficiency of protein S.
c. Factor VII deficiency.
d. Thrombocytopenia.
e. Von Willebrand's disease.

76.Low molecular weight heparin (LMWH) compared with unfractionated


intravenous heparin (UIH) for the treatment of pulmonary embolism is
associated with a:
a. Higher incidence of all-cause mortality
b. Higher incidence of recurrent symptomatic venous thromboembolism at
the end of treatment.
c. Higher incidence of recurrent symptomatic venous thromboembolism at
3 months.
d. Lower incidence of major bleeding complications.
e. Lower risk of heparin-induced thrombocytopenia.
P a g e 19 | 26
SurgiNotes 2nd Ed. (2022)

77.A cirrhotic patient with abnormal hemostatic studies requires an urgent


cholecystectomy. A transfusion of fresh frozen plasma is planned to minimize
the risk of bleeding. What is the optimal timing of this transfusion?
a. In the recovery room.
b. Intraoperatively.
c. On call to surgery.
d. The night before surgery.
e. Two days before surgery.

78.Which of the following mechanisms initiates the normal hemostatic


mechanism?
a. Fibrin formation.
b. Fibrin stabilization.
c. Fibrinolysis.
d. Platelet plug formation.
e. Vascular constriction.

79.What is the treatment of a hemophilic patient who had massive hemarthrosis


of the knee following a fall on his knee?
a. Penicillamine.
b. Fresh frozen plasma.
c. Transfusion of factor VIII to 10% of normal factor levels.
d. Platelet transfusion.
e. Exploration of joint.

80.After undergoing a transurethral resection of the prostate, a 65-year-old man


experiences excessive bleeding attributed to fibrinolysis. It is appropriate to
administer which of the following?
a. Heparin.
b. Warfarin (coumadin).
c. Volume expanders and cryoprecipitate.
d. Aminocaproic acid (amicar).
e. Fresh-frozen plasma and vitamin K.
P a g e 20 | 26
SurgiNotes 2nd Ed. (2022)

81.Which of the following denotes hemolytic transfusion reaction during


anesthesia?
a. Shaking chills and muscle spasm
b. Fever and oliguria.
c. Hyperpyrexia and hypotension.
d. Tachycardia and cyanosis.
e. Bleeding and hypotension.

82.A 50-year-old female patent has chronic renal failure and has been maintained
on chronic dialysis. The patient underwent cholecystectomy. Post-operatively
she had severe bleeding. What is the MOST likely cause for this bleeding?
a. Elevated PT.
b. Elevated PTT.
c. Low platelet count.
d. Decreased platelet aggregation.
e. Sepsis.

83.A mother mentions that her 8-year-old boy gets recurrent attacks of
hemarthrosis following a minimal trauma. Investigations revealed normal
platelet count and prothrombin time, but the partial thromboplastin time is
prolonged. Which of the following statements regarding this clinical condition
is TRUE?
a. There is an underlying liver problem.
b. The sisters of this boy are usually having the same problem.
c. The boy has had repeated episodes of epistaxis.
d. There is no family history in this condition.
e. Transfusion of factor VIII concentrate is helpful.

84.A 70-year-old female patient is receiving warfarin because she has AF and had
previous thrombosis. Which of the following statements regarding warfarin is
correct?
a. The dose of warfarin is adjusted according to the partial thromboplastin
time.
b. Warfarin takes about 8 hours to exert its effects.
c. Warfarin acts by inhibiting factor XII.
d. If the patient is also taking aspirin, the dose of warfarin should be
reduced.
e. Protamine sulphate is the antidote to warfarin.
P a g e 21 | 26
SurgiNotes 2nd Ed. (2022)

85.Which of the following is MOST likely to be a risk factor for the development
of carpal tunnel syndrome?
a. Addison's syndrome.
b. Diabetes insipidus.
c. Fibromyalgia.
d. Graves' disease.
e. Hypothyroidism.

86.Regarding the anatomy of the thyroid gland, which of the following is TRUE?
a. In about 80% of persons, the recurrent laryngeal nerve traverses anterior
to the inferior thyroid artery.
b. The recurrent laryngeal nerve has an oblique course around the
subclavian artery on the left side.
c. The superior laryngeal nerve provides both sensory and motor function
to the larynx.
d. The thyroid gland is innervated only by parasympathetic fibers from the
vagus nerve.
e. Unilateral recurrent laryngeal nerve injury usually results in airway
compromise that necessitates tracheotomy.

87.What is the mode of action of carbimazole?


a. It diminishes the blood supply of the thyroid gland.
b. It prevents the release of thyroid hormone from the gland.
c. It prevents the union between iodine and tyrosine.
d. It prevents transformation of iodides to iodine.
e. It prevents uptake of iodides by the thyroid gland.

88.What is the commonest cause of thyrotoxicosis?


a. Autonomous nodule.
b. Graves' disease.
c. Iodine intake.
d. Subacute thyroiditis.
e. Toxic nodular goiter.
P a g e 22 | 26
SurgiNotes 2nd Ed. (2022)

89.What of the following is the MOST common sign of Cushing’s syndrome?


a. Acne.
b. Hirsutism.
c. Hypertension.
d. Purple striae.
e. Truncal obesity.

91.Twelve hours after having undergone a subtotal thyroidectomy, a 30-year-old


woman develops agitation and difficulty in breathing. Examination reveals
tachycardia and anterior cervical swelling. The surgical dressing is dry. What is
the MOST appropriate treatment?
a. Insertion of an endotracheal tube.
b. Re-opening of the cervical wound at bed.
c. Determination of the serum calcium concentration.
d. Administration of morphine.
e. Administration of O2 by nasal catheters.

92.A 55-year-old female patient who had metastatic breast cancer presents with
weakness, anorexia, malaise, constipation and back pain and lethargy.
Laboratory studies include a normal chest X-Ray, serum albumin 3.2 mg/dl,
serum calcium 14 mg/dl, serum phosphorus 2.6 mg/dl, serum chloride 108
mg/dl, BUN 32 mg/dl and creatinine 2.0 mg/dl. What is the appropriate initial
management?
a. Intravenous normal saline infusion.
b. Administration of thiazide diuretics.
c. Administration of intravenous phosphorus.
d. Use of mithramycin.
e. Neck exploration and parathyroidectomy.

P a g e 23 | 26
SurgiNotes 2nd Ed. (2022)

P a g e 24 | 26
SurgiNotes 2nd Ed. (2022)

98.A patient who had adrenalectomy to remove a pheochromocytoma


demonstrates signs of confusion and complains of sweating and headache
several hours following his operation. His blood pressure is 130/65 mmHg, his
heart rate is 100 beats/min and his respiratory rate is 12 breaths/min. What is
the MOST likely cause of his symptoms?
a. Dehydration.
b. Postoperative bleeding.
c. Hypoglycemia.
d. Narcotic overdose.
e. Incomplete removal of the pheochromocytoma.

99.An unconscious accident victim is hypotensive from intra-abdominal


hemorrhage and needs an emergency laparotomy, his identity is unknown and
therefore, no family is available. Which of the following should be done?
a. Nothing, it is illegal to operate on a patient without consent.
b. The surgeon should document the need for the surgery in the chart and
proceeds.
c. Three doctors should document the need for the surgery in the chart and
the surgeon should then proceed.
d. A court order for surgery should be obtained prior to proceeding.
e. Inform local health authority and then proceed.

100. World health organization established the surgical safety checklist in order
to maintain patient safety in the operating theatre. One of the measures that
eliminate wrong site surgery is to perform a time-out check. The timing of this
check is:
a. Before referral to the operating theatre.
b. Before induction of anesthesia.
c. Before skin incision.
d. Before removal of important organ.
e. Before closure of the wound.
P a g e 25 | 26
SurgiNotes 2nd Ed. (2022)

Egyptian Board Of General Surgery


1st Part Exam – Paper (1) MCQ "ANSWER"
July 2018

1 26 b 51 e 76 e
2 27 d 52 e 77 c
3 28 e 53 e 78 e
4 29 a 54 d 79 c
5 a 30 a 55 c 80 e
6 c 31 b 56 a 81 e
7 c 32 a 57 b 82 d
8 b 33 e 58 c 83 e
9 34 b 59 d 84 d
10 35 b 60 b 85 e
11 36 b 61 a 86 c
12 37 a 62 e 87 c
13 38 a 63 a 88 b
14 b 39 e 64 c 89 e
15 a 40 65 e 90
16 c 41 66 b 91 b
17 d 42 67 d 92 a
18 e 43 68 d 93
19 d 44 69 a 94
20 b 45 c 70 a 95
21 a 46 d 71 e 96
22 d 47 a 72 b 97
23 e 48 a 73 b 98 c
24 c 49 a 74 a 99 b
25 d 50 b 75 c 100 c

P a g e 26 | 26
SurgiNotes 2nd Ed. (2022)

Egyptian Board Of General Surgery


1st Part Exam – Paper (1) MCQ
July 2019
1. A 25-year-old man is playing football when he complains of posterior thigh pain
whilst sprinting with the ball. On examination he has tenderness in the lower
lateral posterior thigh. He is unable to fully extend the knee due to pain. What is
the MOST likely injury?
a. Biceps femoris tear.
b. Gracilis tear.
c. Gastrocnemius tear.
d. Semimembranosus tear.
e. Semitendinosus tear.
2. After undergoing a surgical procedure on the small (short) saphenous vein, a
patient complains of pain and numbness on the lateral aspect of the foot. The
nerve MOST likely to be affected is the:
a. Deep peroneal.
b. Lateral plantar.
c. Saphenous.
d. Superficial peroneal.
e. Sural.
3. A 70-year-old woman undergoes a left mastectomy and axillary clearance. At a
follow-up appointment she was noted to have winging of the left scapula. Which
muscle has been paralyzed as a result of the dissection of the axilla?
a. Latissimus dorsi.
b. Pectoralis major.
c. Serratus anterior.
d. Teres major.
e. Trapezius.
4. A 27-year-old man is admitted to the emergency department after a car crash.
Physical examination reveals weakness in medial rotation and adduction of the
humerus. Which of the following nerves was MOST probably injured?
a. Thoracodorsal.
b. Axillary.
c. Dorsal scapular.
d. Spinal accessory.
e. Radial.
P a g e 1 | 26
SurgiNotes 2nd Ed. (2022)

5. A 34-year-old man is undergoing an emergency appendectomy. After the


appendectomy has been performed successfully, the patient undergoes an
exploratory laparoscopy. Which of the following anatomic features are the
MOST useful to distinguish the jejunum from the ileum?
a. Jejunum has thinner walls compared with the ileum.
b. Jejunum has less mesenteric fat compared with the ileum.
c. Jejunum has more numerous vascular arcades compared with the ileum.
d. Jejunum has more numerous lymphatic follicles beneath the mucosa
compared with the ileum.
e. Jejunum has fewer villi compared with the ileum.
6. A 45-year-old woman is admitted to the emergency department with a
complaint of severe abdominal pain. CT scan and MRI examinations reveal a
tumor of the head of the pancreas involving the uncinate process. Which of
the following vessels is MOST likely to be occluded?
a. Common hepatic artery.
b. Cystic artery and vein.
c. Superior mesenteric artery.
d. Inferior mesenteric artery.
e. Portal vein.
7. Upon removal of a leg cast, a 15-year-old boy complains of numbness of the
dorsum of his right foot and inability to dorsiflex and evert his foot. Which is
the MOST probable site of the nerve compression that resulted in these
symptoms?
a. Popliteal fossa.
b. Neck of the fibula.
c. Lateral compartment of the leg.
d. Anterior compartment of the leg.
e. Medial malleolus.
8. Which of the following statements is TRUE regarding the marginal mandibular
nerve?
a. Is a branch of the 5th cranial nerve.
b. Supplies motor function to the strap muscles.
c. May be associated with the submandibular gland.
d. Lies deep to the facial vein.
e. Is most often injured during dissection of the posterior triangle (level 5)
of the neck.
P a g e 2 | 26
SurgiNotes 2nd Ed. (2022)

9. A 74-year-old woman presents to the outpatient clinic with pelvic pain. A CT


scan reveals enlarged para-aortic lymph nodes. These are MOST likely to be
involved in secondary spread from a tumor in which of the following organs?
a. Cervix.
b. Ovary.
c. Uterus.
d. Vagina.
e. Vulva.

10.A 64-year-old woman is reviewed in the emergency department with an acute


right groin swelling, below and lateral to the pubic tubercle. Which of the
following forms the medial boundary of the ring through which this hernia is
protruding?
a. Conjoint tendon.
b. Inferior epigastric artery.
c. Inguinal ligament.
d. Lacunar ligament.
e. Pectineal ligament.

11.A 30-year-old motorcyclist suffers a closed fracture to the mid-shaft of the


tibia. Anterior compartment syndrome could cause loss of sensation:
a. In the first web space.
b. Over the dorsum of the foot.
c. Over the lateral edge of the foot.
d. Over the medial aspect of the hallux.
e. Over the medial malleolus.

12.An 82-year-old man has complete occlusion of his inferior mesenteric artery
on angiography but no symptoms or signs of colonic ischemia. Which of the
following arteries is the MOST likely additional source of blood supply to the
territory of the inferior mesenteric artery?
a. Left colic.
b. Left gastroepiploic.
c. Middle colic.
d. Splenic.
e. Superior rectal.
P a g e 3 | 26
SurgiNotes 2nd Ed. (2022)

13.In L5 nerve root compression from a herniated intervertebral disc, altered


cutaneous sensation is predominantly in the:
a. Dorsum of foot.
b. Heel of foot.
c. Medial border of foot.
d. Outer border of foot.
e. Sole of foot.

14.A 22-year-old man is admitted to the emergency department after falling from
his bicycle. Radiograph examination reveals a fracture of the tibia above ankle.
MRI and physical examination reveal the tibial nerve is severed on the
posterior aspect of tibia. Which of the following signs will MOST likely present
during physical examination?
a. Sensory loss of the dorsum of the foot.
b. Sensory loss on the sole of the foot.
c. Foot drop.
d. Paralysis of the extensor digitorum brevis.
e. Sensory loss of the entire foot.

15.Wound healing is relatively slowest in which of the following?


a. Eyelid.
b. Lips.
c. Sternum.
d. Stomach.
a. c. Urinary bladder.

16.Which of the following is TRUE regarding the physiology of the thyroid gland?
a. Absorption of iodides occurs by passive diffusion.
b. Potassium perchlorate prevents the union between iodine and tyrosine.
c. T3 is four times more active than T4.
d. The half-life of thyroxine is 4 days.
e. The necessary intake of iodide is 50 ug/day.
P a g e 4 | 26
SurgiNotes 2nd Ed. (2022)

17.Which of the following can lower the serum potassium level?


a. Calcium gluconate.
b. Dextrose 25%.
c. Ringer's lactate.
d. Saline.
e. Sodium bicarbonate.

18.Which of the following is the MOST potent stimulus for contraction of the gall
bladder?
a. Acetylcholine.
b. Cholecystokinin.
c. Intravenous hyperalimentation.
d. Secretin.
e. Vagal stimulation.

19.A 60-year-old male patient is suffering from diabetic ketoacidosis. Which of


the following is the MOST important buffer base in the extracellular fluid?
a. Bicarbonate.
b. Hemoglobin.
c. Lactate.
d. Phosphate.
e. Plasma proteins.

20.Which of the following statements is correct regarding potassium


homeostasis?
a. About 60% of the total body potassium is in the intracellular
compartment.
b. Aldosterone stimulates potassium reabsorption in the distal convoluted
tubules.
c. Hypokalaemia causes depression of the ST segment
d. In alkalosis there is usually hyperkalaemia.
e. Insulin causes potassium to leave the cell.
P a g e 5 | 26
SurgiNotes 2nd Ed. (2022)

21.Which of the following is a function of the parasympathetic system?


a. Increase the blood supply of the heart.
b. Increase the blood supply of skeletal muscles.
c. Increase in the heart rate.
d. Stimulation of glycogenolysis.
e. Increased motility of the gastrointestinal tract.

22.During the second (proliferative) phase of wound healing the predominant


cells in the wound site are fibroblasts. Cross linkage of collagen requires
hydroxy-proline and hydroxy-lysine residues, which requires a specific vitamin
to be available in sufficient quantities. Deficiency of which vitamin results in
collagen that is unstable?
a. Vitamin B2 (riboflavin).
b. Vitamin B6 (pyridoxine).
c. Vitamin C (ascorbic acid).
d. Vitamin D (cholecalciferol).
e. Vitamin E (tocopherol).

23.A 70-year-old patient with diabetes and paraplegia is undergoing an elective


laparoscopic cholecystectomy after an episode of biliary pancreatitis. Shortly
after induction, blood pressure is normal, but ECG shows peaked P waves and
a widened QRS complex. The Most likely diagnosis is:
a. Ketoacidosis.
b. Hyperkalemia.
c. Hypoglycemia.
d. Hypocalcemia.
e. Acute myocardial infarction.

24.A 70-year-old man with chronic obstructive pulmonary disease is admitted for
elective hemicolectomy. What is a preoperative arterial blood gas analysis
likely to show?
Bicarbonate Arterial PCO2
a Decreased Decreased
b Increased Decreased
c Normal Decreased
d Decreased Increased
e Increased Increased

P a g e 6 | 26
SurgiNotes 2nd Ed. (2022)

25.Norepinephrine (noradrenaline) causes an increase in the systemic vascular


resistance. In which condition is its use MOST appropriate?
a. Cardiogenic shock.
b. Hypovolemic shock.
c. Neurogenic shock.
d. Obstructive shock (e.g. pulmonary embolism).
e. Septic shock.

26.Parathyroid hormone is important for regulation of calcium homeostasis.


Which of the following best describes the hormone?
a. Accelerates renal hydroxylation of vitamin D.
b. Directly accelerates calcium absorption from the intestine.
c. Is a steroid hormone.
d. Is secreted under the control of the anterior pituitary.
e. Increases plasma phosphate concentration in a normal adult.

27.Which of the following clinical situations can be associated with hypovolemic


hyponatremia?
a. Congestive heart failure.
b. SIADHI (syndrome of inappropriate ADH).
c. Cirrhosis.
d. Hyperglycemia.
e. Gastrointestinal losses.

28.An 80-year-old man with a history of hypertension, controlled with thiazide


diuretics, undergoes transurethral resection of prostate under general
anesthetic. The surgery is prolonged and in the recovery room he complains
of nausea and a headache. He later becomes agitated and confused. Which of
the following is the MOST likely diagnosis?
a. Hypercalcaemia.
b. Hyperuricaemia.
c. Hypoglycaemia.
d. Hypokalaemia.
e. Hyponatraemia.
P a g e 7 | 26
SurgiNotes 2nd Ed. (2022)

29.A 50-year-old male patient was punctured in his foot by a very tiny piece of
glass which could not be removed. Three months later the patient presented
by a firm small nodule at the site of the puncture and this was excised. Which
of the following cells will be predominant on pathological examination of the
removed nodule?
a. Epithelioid cells.
b. Fibroblasts.
c. Lymphocytes.
d. Plasma cells.
e. Polymorphonuclear leucocytes.

30.In patients with reflux esophagitis. What does the presence of columnar cells
in the esophageal mucosa represent?
a. Carcinoma in situ.
b. Carcinoma.
c. Dysplasia.
d. Hyperplasia.
e. Metaplasia.

31.With regard to protein loss after injury, which of the following statements is
TRUE?
a. It can be prevented by total parenteral nutrition.
b. It occurs primarily from skeletal muscle.
c. It occurs primarily from the site of injury.
d. It results from significant decrease intake.
e. It results from impaired synthesis.

32.A 54-year-old man with a history of depression is brought to the emergency


department. He has taken an overdose of benzodiazepines and is found
unconscious, lying on his right upper limb. His right arm is swollen and mottled
in color, with an insensate and stiff hand. Analysis of his urine is MOST likely
to reveal increased levels of:
a. Haemoglobin.
b. Myoglobin.
c. Protein.
d. Red blood cells.
e. White blood cells.

P a g e 8 | 26
SurgiNotes 2nd Ed. (2022)

33.Which of the following statements regarding TNF is TRUE?


a. TNF-α is produced only by monocytes and macrophages.
b. Release of TNF is stimulated by exotoxins.
c. TNF enhances the anticoagulant activity of endothelial surfaces.
d. TNF is responsible for the cachexia associated with metastatic disease.
e. Glucocorticoids have essentially no effect on TNF-α expression.

34.What is the MOST common bacterial organism present in the colon?


a. Bacteroides.
b. Clostridium difficile.
c. Escherichia coli.
d. Salmonella.
e. Streptococci.

35.Severe sepsis can be defined as:


a. Sepsis in association with refractory hypotension despite adequate fluid
resuscitation.
b. The systemic host response to invasive infection.
c. Sepsis in association with organ dysfunction.
d. The presence of micro-organisms invading normally sterile host tissues.
e. A clinical syndrome characterized by two or more of tachycardia,
tachypnea, hyper- or hypothermia and leukocytosis or leucopenia.

36.Which of the following operations is a clean contaminated operation?


a. Colectomy for a prepared colon.
b. Drainage of a pericolic abscess.
c. Inguinal hemiectomy.
d. Perforated appendicitis.
b. Thyroidectomy.
P a g e 9 | 26
SurgiNotes 2nd Ed. (2022)

37.Which of the following is MOST important in the treatment of necrotizing


fasciitis?
a. Antifungal agents.
b. Antitoxin.
c. Hyperbaric O2.
d. Immunoglobulins.
e. Wide surgical debridement.
38.A 30-year-old woman sustained a puncture wound to the foot. The patient has
been on a therapeutic dose of steroids for the past 5 years for ulcerative colitis.
Her last tetanus toxoid booster was 8 years ago. What should the patient
receive?
a. Tetanus toxoid booster.
b. Human immunoglobulin, antibiotics with anaerobic coverage.
c. Tetanus toxoid plus human immunoglobulin.
d. Tetanus toxoid plus human immunoglobulin and antibiotics with aerobic
and anaerobic coverage.
e. Wide debridement of the wound.
39.Treatment of wounds with negative-pressure wound dressings (VAC
dressings) may result in all of following EXCEPT:
a. Increased apoptosis.
b. Increased granulation tissue.
c. Improved microvascular blood flow to wound edges.
d. Removal of excess fluid and debris.
e. Lower bacterial counts.
40.A 45-year-old woman undergoes an uneventful laparoscopic cholecystectomy
for which she receives one dose of cephalosporin. One week later, she returns
to the emergency room with fever, nausea and copious diarrhea and is
subsequently diagnosed with pseudomembranous colitis. With respect to this
disease, which one of the following statements is correct?
a. Surgical intervention is frequently required.
b. After appropriate antibiotic therapy, the relapse rate is less than 5%.
c. Tissue culture assay for clostridium difficile toxin B is neither sensitive nor
specific; therefore diagnosis should be based on clinical findings.
d. If surgery is performed a left hemicolectomy is usually adequate to treat
pseudo- membranous colitis.
e. Indications for surgical treatment include intractable disease failure of
medical therapy, toxic megacolon and colonic perforation.

P a g e 10 | 26
SurgiNotes 2nd Ed. (2022)

41.A non-immune surgical resident is stuck by a contaminated needle from a


hepatitis B surface antigen (HBsAg)-positive source. Which of the following is
the correct initial treatment?
a. None because the patient does not have active hepatitis B virus (HBV)
infection and is immune to HBV.
b. Interferon.
c. Vaccination against HBV.
d. Hepatitis B immune globulin (HBIG).
e. Vaccination against HBV and administration of HBIG.
42.Which of the following statements about the treatment of necrotizing fasciitis
is TRUE?
a. The underlying tissue necrosis is reflected by the extent of skin necrosis.
b. Intravenous immunoglobulin is the first line of therapy.
c. Penicillin provides sufficient antibiotic coverage.
d. Hyperbaric oxygen has been shown to improve survival.
e. Exploratory incisions over normal appearing skin are effective in
determining the extent of the necrosis.
43.Which of the following statements regarding the risk for postsplenectomy
sepsis is TRUE?
a. The indication for splenectomy has no bearing on a patient's risk for
developing postsplenectomy sepsis.
b. Adult splenectomy patients have a greater likelihood of developing
postsplenectomy sepsis than do children or newborns who require a
splenectomy.
c. The risk for postsplenectomy sepsis is highest in the first year after
splenectomy, but asplenic patients' increased risk for developing sepsis
persists for approximately 10 years following splenectomy.
d. The risk of sepsis is increased in splenectomy patients due to impaired
cellular immunity.
e. The most common organism implicated in postsplenectomy sepsis is
Haemophilus influenzae.
44.Which of the following is the best parameter for monitoring septic shock?
a. Central venous pressure (CVP).
b. Vasopressor requirement.
c. Urine output.
d. Serum lactate.
e. Mental status changes.

P a g e 11 | 26
SurgiNotes 2nd Ed. (2022)

45.Which of the following which is the MOST effective way in preventing surgical
infection?
a. Antibiotic prophylaxis.
b. Bowel preparation.
c. Drains and irrigation.
d. Skin preparation.
e. Surgical technique.

46.Which of the following is an early manifestation of sepsis?


a. Cutaneous vasodilatation.
b. Decreased cardiac output.
c. Hypoglycaemia.
d. Increased arteriovenous O2 difference.
e. Respiratory acidosis.

47.A 37-year-old woman with end-stage renal disease had a cadaveric renal
transplant 18 months previously. She tolerated her transplantation well and
has been receiving a stable regiment of tacrolimus and corticosteroids with a
creatinine of 1.0 mg/dl. She now presents with fatigue, bone pain and
osteoporosis. Her serum calcium is 11.5 mg/dl. The next step in management
should be:
a. Subtotal parathyroidectomy.
b. An increase in corticosteroid dose.
c. Hydration and diuresis.
d. Calcitonin injections.
e. Phosphate binders.

48.Which of the following statements about acute rejection is NOT true?


a. It is T-cell mediated.
b. It is related to organ-host human leukocyte antigen disparity.
c. Treatment can save the grafted organ in 90% to 95% of cases.
d. It does not occur with living related donors.
e. It is associated with an increased risk of chronic rejection.
P a g e 12 | 26
SurgiNotes 2nd Ed. (2022)

49.Which of the following statements regarding rejection of solid organ


transplants is TRUE?
a. Hyper-acute rejection begins in the operating room with reperfusion of
the transplanted organ.
b. Liver transplants are especially susceptible to hyper-acute rejection.
c. Most immunosuppressive medications are used to prevent chronic
rejection.
d. The major cause of graft failure is acute rejection.
e. Chronic rejection is characterized histologically by lymphocyte
infiltration.

50.Which of the following is NOT an indication for liver transplant?


a. Cirrhosis complicated by ascites and esophageal varices.
b. Acute liver failure secondary to acetaminophen overdose.
c. NASH (nonalcoholic steatohepatitis) complicated by hepatic
encephalopathy.
d. Hepatocellular carcinoma with underlying cirrhosis.
e. Hepatocellular carcinoma with biopsy-proven lymph node metastasis.

51.The arterial blood gas analysis of a patient was pH 7.5, PCO2 47 mmHg and
HCO3- 35 mmol/L. This patient MOST likely is suffering from:
a. Chronic obstructive pulmonary disease.
b. Diabetic ketoacidosis.
c. Persistent diarrhea.
d. Profound vomiting.
e. Salicylate poisoning.

52.Five days after an uneventful cholecystectomy, an asymptomatic middle aged


woman is found to have a serum sodium level of 120 mEq/L. What is the
proper management?
a. Administration of hypertonic saline solution.
b. Administration of Ringer's lactate solution.
c. Aggressive diuresis with furosemide.
d. Plasma ultra-filtration.
e. Restriction of free water.
P a g e 13 | 26
SurgiNotes 2nd Ed. (2022)

53.In the presence of acute blood loss, what is the initial mechanism to maintain
an adequate preload to the heart?
a. Development of tachycardia.
b. Hormonal effects of angiotensin.
c. Hormonal effects of rennin.
d. Increase in systemic vascular resistance.
e. Increased cortisol secretion.

54.Which of the following is a clinical feature of a major pulmonary embolism?


a. Bradycardia.
b. Collapsed neck veins.
c. Hemoptysis.
d. High fever.
e. Tachypnea.

55.Following an operation and extubation a patient was restless. His arterial


blood gases revealed: pH 7.36, PaO2 65 mmHg, PaCO2 55 mmHg and PaCO3- 30
mmol/L. The physiologic status can best be described as which of the
following?
a. Respiratory alkalosis.
b. Respiratory acidosis.
c. Metabolic acidosis.
d. Metabolic alkalosis.
e. Combined respiratory and metabolic acidosis.

56.A 75-year-old woman who is in the ICU after undergoing colectomy is


hypotensive and tachycardic. Pulmonary capillary wedge pressure (PCWP) is
elevated to 18 mmHg and cardiac output is 3 L/min. She is in shock best
described as which of the following?
a. Hypovolemic shock.
b. Septic shock.
c. Cardiogenic shock.
d. Anaphylactic shock.
e. Neurogenic shock.
P a g e 14 | 26
SurgiNotes 2nd Ed. (2022)

57.Which of the following metabolic effects is present in shock?


a. Increase in sodium and water excretion.
b. Increase in renal perfusion.
c. Decrease in cortisol levels.
d. Hyperkalemia.
e. Hypoglycemia.

58.A 21-year-old man undergoes major abdominal surgery after a motor vehicle
collision. He has a cardiac arrest in the intensive care unit shortly after
returning from surgery. Select the MOST appropriate pharmacologic agent for
the patient:
a. Epinephrine.
b. Norepinephrine.
c. Phenylephrine.
d. Dopamine.
e. Dobutamine.

59.Advantages of epidural analgesia include the following EXCEPT:


a. Earlier mobilization after surgery.
b. Earlier return of bowel function.
c. Shorter hospitalizations.
d. Increased stress response to surgery.
e. Excellent pain relief.

60.Which of the following is an important goal of therapy at 24 hours after the


onset of hypovolemic shock that may improve mortality?
a. Normalization of blood pressure and heart rate.
b. Normalization of urine output and base deficit.
c. Discontinuation of vasopressors and inotropes.
d. Ensuring the hematocrit is at least 30% (0.3).
e. Normalization of body and skin temperature.

P a g e 15 | 26
SurgiNotes 2nd Ed. (2022)

61.Which of the following is TRUE about the catabolic response to trauma?


a. Intravenous hyperalimentation can prevent the catabolic response to
trauma.
b. Liver glycogen is the source of dextrose in the first week.
c. The catabolic response is initiated by the thyroid hormones.
d. The catabolic response is the same regardless of the severity of trauma.
e. There is inevitable loss of muscle mass.

62.A 55-year-old male patient is receiving 150 mg of aspirin daily because he has
coronary artery disease. Which of the following tests will be affected in this
patient?
a. Bleeding time.
b. Coagulation time.
c. INR.
d. PTT.
e. Thrombin time.

63.Which of the following is a contraindication to heparin therapy?


a. Closed head injury two weeks ago.
b. Heparin-induced thrombocytopenia.
c. Subclavian vein thrombosis.
d. Superior mesenteric artery embolism.
e. Third trimester of pregnancy.

64.Which of the following is the MOST likely cause of increased prothrombin


time?
a. Christmas disease.
b. Hemophilia.
c. Heparin therapy.
d. Von Willebrand’s disease.
e. Warfarin overdose.
P a g e 16 | 26
SurgiNotes 2nd Ed. (2022)

65.What is the main mode of action of heparin?


a. It increases the level of protein C.
b. It is a co-factor of antithrombin III.
c. It prevents clot retraction.
d. It prevents platelets aggregation.
e. It prevents the synthesis of fibrinogen.

66.What is the principal mechanism of the antithrombotic action of administering


aspirin in low dose?
a. Induction of capillary vasodilatation.
b. Induction of endothelial cell prostacyclin production.
c. Induction of endothelial heparin production.
d. Inhibition of factor V production by the liver.
e. Inhibition of platelet production of thromboxane A2.

67.Which of the following statement is TRUE concerning hemophilia A?


a. A positive family history for bleeding disorders is present in all patients.
b. Hemophilia A is inherited as a sex-linked recessive deficiency of factor VIII.
c. Laboratory tests reveal a prolongation of aPTT, prothrombin time (PT),
thrombin clotting time and platelet aggregation.
d. Minor bleeding cannot be controlled locally without the need for
replacement therapy.
e. Spontaneous bleeding is usual with factor VIII levels greater than 10% of
normal.

68.You were obliged to perform cholecystectomy for a cirrhotic patient. During


surgery there was excessive bleeding. What would you order?
a. Fresh blood.
b. Fresh frozen plasma.
c. Fresh platelets.
d. IV vitamin K.
e. IV factor VIII.
P a g e 17 | 26
SurgiNotes 2nd Ed. (2022)

69.What does increased level of fibrin degradation products (FDP) in the blood
denote?
a. Diminished fibrinogen synthesis.
b. Diminished platelet count.
c. Diminished platelet function.
d. Disseminated intravascular coagulation (DIC).
e. Heparin overdose.

70.A fully heparinized patient develops a condition requiring emergency surgery.


After stopping the heparin, what else should be done to prepare the patient?
a. 2 units of cryoprecipitates.
b. Administration of protamine sulphate 1 mg for every 100 units of heparin
most recently administered.
c. Immediate fresh frozen plasma.
d. Transfusion of 10 units of platelets.
e. Vitamin K intravenously.

71.Which of the following is the best method of identifying the liability to


bleeding during a surgical procedure?
a. Platelet count.
b. A complete history and physical examination.
c. Bleeding time.
d. Lee-White clotting time.
e. Prothrombin time (PT).

72.Which of the following denotes a hemolytic transfusion reaction during


anesthesia?
a. Shaking chills and muscle spasms.
b. Fever and oliguria.
c. Hyperpyrexia and hypotension.
d. Tachycardia and cyanosis.
e. Bleeding and hypotension.
P a g e 18 | 26
SurgiNotes 2nd Ed. (2022)

73.A victim of blunt abdominal trauma requires a partial hepatectomy. He is


rapidly transfused with 8 units of appropriately cross-matched packed red
blood cells from the blood bank. He is noted in the recovery room to be
bleeding from intravenous puncture sites and the surgical incision. His
coagulopathy is likely due to thrombocytopenia and deficiencies of which
clotting factors?
a. II only.
b. II and VII.
c. V and VIII.
d. IX and X.
e. XI and XII.

74.Regarding low molecular weight heparin, which of the following is TRUE?


a. Activity is effectively measured by activated partial thromboplastin time
(APTT).
b. Strongly binds to plasma proteins.
c. Has a longer plasma half-life than a standard heparin.
d. Has its effect reversed by an equivalent dose of protamine.
e. Has a normal plasma clearance in patients with renal failure.

75.A 45-year-old woman with deep vein thrombosis is taking warfarin (coumadin)
5 mg/d. Seven days after initiation of therapy, she has warfarin-induced skin
necrosis. Which of the following statements regarding this condition is TRUE?
a. It commonly occurs after warfarin therapy.
b. It usually involves the upper extremities.
c. It improves with an increase in the dose of coumadin.
d. It improves with a decrease in the dose of coumadin.
e. It requires cessation of coumadin and infusion of heparin.

76.Which of the following is MOST likely to be a risk factor for the development
of carpal tunnel syndrome?
a. Addison's syndrome.
b. Diabetes insipidus.
c. Fibromyalgia.
d. Graves' disease.
e. Hypothyroidism.
P a g e 19 | 26
SurgiNotes 2nd Ed. (2022)

77.A 30-year-old female patient complains of fatigue, generalized bony aches and
depression. The patient gives a history of 2 previous operations for urinary
stones. Which of the following laboratory tests is MOST accurate in the
diagnosis?
a. 24 hours urinary calcium.
b. Ionizable serum calcium.
c. Parathormone hormone.
d. Plasma chloride.
e. Serum phosphate.

78.What is the mode of action of carbimazole?


a. It diminishes the blood supply of the thyroid gland.
b. It prevents the release of thyroid hormone from the gland.
c. It prevents the union between iodine and tyrosine.
d. It prevents transformation of iodides to iodine.
e. It prevents uptake of iodides by the thyroid gland.

79.Which of the following is the MOST potent stimulus for aldosterone secretion?
a. ACTH.
b. Antidiuretic hormone.
c. Hyperkalaemia.
d. Hypernatraemia.
e. Rennin angiotensin system.

80.Which of the following abnormalities indicates the possibility of


hyperaldosteronism?
a. Hyperkalemia, hyponatremia, hypochloremia.
b. Hyperkalemia, hypernatremia, low pH.
c. Hyperkalemia, hyponatremia, hyperglycemia.
d. Hypokalemia, hypernatremia, high pH.
e. Hypokalemia, hypochloremia, high pH.
P a g e 20 | 26
SurgiNotes 2nd Ed. (2022)

81.Twelve hours after having undergone a subtotal thyroidectomy, a 30-year-old


woman develops agitation and difficulty in breathing. Examination reveals
tachycardia and anterior cervical swelling. The surgical dressing is dry. What is
the MOST appropriate treatment?
a. Insertion of an endotracheal tube.
b. Re-opening of the cervical wound at bed.
c. Determination of the serum calcium concentration.
d. Administration of morphine.
e. Administration of O2 by nasal catheters.

82.A 40-year-old woman had the anterior lobe of the pituitary removed because
of a tumor. Without postoperative supplements, which of the following could
occur?
a. Failure to produce adequate amounts of thyroxine.
b. Failure to produce parathyroid hormone in response to hypocalcaemia.
c. Failure to secrete catecholamines in response to stress.
d. Failure to secrete insulin in hyperglycaemia.
e. Inability to concentrate urine in response to water deprivation.

83.What is the MOST common cause of primary adrenal insufficiency (Addison’s


disease)?
a. Tuberculosis.
b. Kaposi sarcoma.
c. Cytomegalovirus.
d. Lymphoma.
e. Autoimmune disorder.

84.Which of the following is the MOST common cause of goitrous hypothyroidism


in adults?
a. Graves' disease.
b. Riedel's thyroiditis.
c. Hashimoto's disease.
d. De Quervain's thyroiditis.
e. Lymphoma of the thyroid gland.
P a g e 21 | 26
SurgiNotes 2nd Ed. (2022)

85. Which of the following biochemical tests for the work-up of pheochromocytomas
has the highest sensitivity?
a. Urinary norepinephrine.
b. Urinary vanillylmandelic acid.
c. Urinary total metanephrine.
d. Plasma free metanephrines and normetanephrine.
e. Urinary epinephrine.
86. Regarding the anatomy of the thyroid gland, which of the following is TRUE?
a. In about 80% of persons, the recurrent laryngeal nerve traverses anterior to
the inferior thyroid artery.
b. The recurrent laryngeal nerve has an oblique course around the subclavian
artery on the left side.
c. The superior laryngeal nerve provides both sensory and motor function to
the larynx.
d. The thyroid gland is innervated only by parasympathetic fibers from the
vagus nerve.
e. Unilateral recurrent laryngeal nerve injury usually results in airway
compromise that necessitates tracheotomy.
87. A 40-year-old male patient who had excision of pheochromocytoma
demonstrates signs of confusion and complains of sweating and headache
several hours following his operation. His blood pressure is 130/65 mmHg, heart
rate is 85 beats/min and respiratory rate is 12 breaths/min. What is the MOST
likely cause of his symptoms?
a. Dehydration.
b. Postoperative bleeding.
c. Hypoglycemia.
d. Narcotic overdose.
e. Incomplete removal of the pheochromocytoma.
88. A 47-year-old male with Crohn's colitis maintained on 40 mg prednisone daily for
the past year presents for elective colectomy. The procedure was uncomplicated
and he was adequately resuscitated. In the post-anesthesia care unit (PACU), the
patient is noted to be febrile and hypotensive with mean arterial blood pleasure
of 80/60 mmHg in the 50s. What is your next step in management?
a. IV dobutamine.
b. Hydrocortisone.
c. 1 unit of packed red blood cells.
d. Antibiotics.
e. Epinephrine.
P a g e 22 | 26
SurgiNotes 2nd Ed. (2022)

89.Which one of the following clinical scenarios is associated with


hypercalcemia?
a. Fluid resuscitation from shock.
b. Rapid infusion of blood products.
c. Improper administration of phosphates.
d. Malignancy.
e. Acute pancreatitis.

90.What is the MOST deleterious effect of excessive glucose in the formula of


longstanding TPN?
a. Hyperglycemia.
b. Excessive diuresis.
c. Hepatic steatosis.
d. Hyponatremia.
e. Hypoglycemia.

91.An adult male develops acute necrotizing pancreatitis after an endoscopic


retrograde cholangiopancreaticogram. The patient requires ventilatory
support and is in need of nutritional support. Which is the best route of
providing nutrition?
a. Parenteral nutrition through peripheral access.
b. Parenteral nutrition through central access.
c. Enteral nutrition through jejunal feeding tube.
d. Oral elemental supplementation.
e. Intra-gastric tube feeding.

92.A 50-year-old man with small-bowel fistula has been receiving TPN for the
previous 3 weeks. He is scheduled for exploratory laparotomy and closure of
fistula. On the morning of the day of surgery, TPN is discontinued and
intravenous infusion with Ringer's lactate is started. An hour later, the patient
is found to be anxious, sweating and tachycardic. What is the MOST likely
cause?
a. Anxiety.
b. Hypoglycemia.
c. Hypovolemia.
d. Unexplained hemorrhage.
e. Hyperglycemia.
P a g e 23 | 26
SurgiNotes 2nd Ed. (2022)

93.A patient of serious car accident remains intubated and is without enteral
feeding for 3 days. Which of the following is the main energy source during
critical illness/injury?
a. Skeletal muscle.
b. Liver.
c. Adipose tissue.
d. Kidney.
e. Gut.

94.Four days after undergoing hysterectomy, a 30-year-old woman develops


phlegmasia cerulea dolens over the right lower extremity. What is the MOST
appropriate treatment?
a. Bed rest and elevation.
b. Systemic heparinization.
c. Venous thrombectomy.
d. Prophylactic vena caval filter.
e. Local urokinase infusion.

95.A patient with multiple small bowel resections for Crohn's disease presents
with an acute flare. He is suspected of having functionally a short gut
syndrome and started on TPN. Refeeding syndrome is characterized by which
of the following electrolyte abnormalities?
a. Hyponatremia, hypokalemia and hypercalcemia.
b. Hyperphosphatemia, hypokalemia and hypocalcaemia.
c. Hypokalemia, hypomagnesemia and hypophosphatemia.
d. Hypokalemia, hyponatremia and hypomagnesemia.
e. Hyperkalemia, hypernatremia and hypercalcemia.

96.After 4 days of TPN, a patient above develops blood glucose levels greater
than 300 mg/dl. Hyperglycemia in a surgical patient receiving TPN may best be
managed by:
a. Oral hypoglycemics.
b. Decreasing the dextrose load and doubling the amount of fat.
c. Adding regular insulin to the TPN.
d. Discontinuing TPN for 2 weeks and then trying to start TPN again.
e. Increasing the concentration of protein and carbohydrate calories and
decreasing that of lipids.

P a g e 24 | 26
SurgiNotes 2nd Ed. (2022)

97.An unconscious accident victim is hypotensive from intra-abdominal


hemorrhage and needs an emergency laparotomy. His identity is unknown
and therefore, no family is available. Which of the following should be done?
a. Nothing, it is illegal to operate on a patient without consent.
b. The surgeon should document the need for the surgery in the chart and
proceed.
c. Three doctors should document the need for the surgery in the chart and
the surgeon should then proceed.
d. A court order for surgery should be obtained prior to proceeding.
e. Inform local health authority and then proceed.
98.The nurse prepared a unit of blood to be transfused to a patient with lower
gastrointestinal bleeding. Just before transfusion she noticed that the blood
unit didn't belong to the patient and she sent it back to the blood bank and
received the right one. This situation is called:
a. Adverse event.
b. Near miss event.
c. Never event.
d. No harm event.
e. Sentinel event.
99.World health organization established the surgical safety checklist in order to
maintain patient safety in the operating theatre. One of the measures that
eliminate wrong site surgery is to perform a time-out check. The timing of this
check is:
a. Before referral to the operating theatre.
b. Before induction of anesthesia.
c. Before skin incision.
d. Before removal of important organ.
e. Before closure of the wound.
100. A 12-year-old child undergoes spinal fixation for severe scoliosis.
Postoperatively, the patient becomes hypotensive, tachycardic and oliguric
and hematocrit is 18%. The parents and child refuse all blood products. The
next step should be to:
a. Withhold all blood products.
b. Transfuse packed red blood cells.
c. Administer erythropoietin.
d. Transfuse human hemoglobin substitute (polyheme).
e. Obtain court decision for blood transfusion.

P a g e 25 | 26
SurgiNotes 2nd Ed. (2022)

Egyptian Board Of General Surgery


1st Part Exam – Paper (1) MCQ "ANSWER"
July 2019

1 a 26 a 51 d 76 e
2 e 27 e 52 e 77 b
3 c 28 e 53 d 78 c
4 a 29 a 54 e 79 e
5 b 30 e 55 b 80 d
6 c 31 b 56 c 81 b
7 b 32 b 57 d 82 a
8 c 33 d 58 a 83 e
9 b 34 a 59 d 84 c
10 d 35 c 60 b 85 d
11 a 36 a 61 e 86 c
12 c 37 e 62 a 87 c
13 a 38 d 63 b 88 b
14 b 39 a 64 e 89 d
15 c 40 e 65 b 90 a
16 c 41 e 66 e 91 c
17 e 42 e 67 b 92 b
18 b 43 c 68 b 93 c
19 a 44 d 69 d 94 c
20 c 45 a 70 b 95 c
21 e 46 a 71 b 96 c
22 c 47 a 72 e 97 b
23 b 48 d 73 c 98 b
24 e 49 a 74 c 99 c
25 e 50 e 75 e 100 e

P a g e 26 | 26
SurgiNotes 2nd Ed. (2022)

Egyptian Board Of General Surgery


1st Part Exam – Paper (1) MCQ
October 2020
1. Which of the following is the main blood supply to the breast?
a. Acromiothoracic artery.
b. Intercostal arteries.
c. Internal thoracic artery.
d. Lateral thoracic artery.
e. Superior thoracic artery.

2. A 40-year-old male patient had excision of a carotid body tumor. Which of the
following nerves is MOST liable to be injured?
a. Facial nerve.
b. Glossopharyngeal nerve.
c. Hypoglossal nerve.
d. Spinal accessory nerve.
e. Vagus nerve.

3. Intracapsular fractures of the neck of the femur are very liable to be followed
by avascular necrosis of the head of femur. From which arteries does the head
of femur receive its main blood supply?
a. Artery of the ligament of the head of femur.
b. Deep external pudendal artery.
c. First perforator artery of the deep femoral.
d. Medial and lateral circumflex femoral arteries.
e. The superficial femoral artery.

4. Taste from the posterior one third of tongue is provided by which of the
following nerves?
a. Facial.
b. Glossopharyngeal.
c. Hypoglossal.
d. Lingual.
e. Vagus.
P a g e 1 | 26
SurgiNotes 2nd Ed. (2022)

5. Pain referred to the right side of the neck and extending laterally from the
right clavicle to the tip of the right shoulder is MOST likely due to involvement
of:
a. Cervical cardiac nerves.
b. Posterior vagal trunk.
c. Right intercostal nerves.
d. Right phrenic nerve.
e. Right recurrent laryngeal nerve.

6. Which of the following nerves is MOST likely to be injured following fracture


shaft of the humerus?
a. Medial cord of the brachial plexus.
b. Median nerve.
c. Musculocutaneous nerve.
d. Radial nerve.
e. Ulnar nerve.

7. Which of the following will be present following injury of the long thoracic
nerve?
a. Depression of the shoulder.
b. Inability to abduct the shoulder.
c. Inability to extend the shoulder.
d. Loss of sensation at the medial side of the axilla.
e. Winging of the scapula.

8. During an operation for abdominal aortic aneurysm, the inferior mesenteric


artery was ligated, but still the vascularity of the descending colon was
preserved. From which of the following arteries came the blood supply of the
descending colon?
a. Ileocolic artery.
b. Left gastroepiploic artery.
c. Marginal artery.
d. Middle rectal.
e. Superior rectal.
P a g e 2 | 26
SurgiNotes 2nd Ed. (2022)

9. Which of the following is TRUE regarding the anatomy of the thyroid gland?
a. In about 80 % of persons, the recurrent laryngeal nerve traverses anterior
to the inferior thyroid artery.
b. The recurrent laryngeal nerve has an oblique course around the
subclavian artery on the left side.
c. The superior laryngeal nerve provides both sensory and motor function
to the larynx.
d. The thyroid gland is innervated only by parasympathetic fibers from the
vagus nerve.
e. Unilateral recurrent laryngeal nerve injury usually results in airway
compromise that necessitates tracheotomy.

10.A patient who had an injury to the right lateral side of the neck was unable to
raise his right shoulder. What is the injured nerve?
a. Accessory.
b. Dorsal scapular.
c. Greater occipital.
d. Thoracodorsal.
e. Transverse cutaneous nerve of the neck.

11.Which group of lymph nodes will be inflamed secondary to paronychia of the


big toe?
a. Deep inguinal lymph nodes.
b. External iliac lymph nodes.
c. Lateral group of superficial inguinal lymph nodes.
d. Medial group of superficial inguinal lymph nodes.
e. Vertical group of superficial inguinal lymph nodes.

12.A patient has severe hematemesis due to bleeding ulcer on the lesser
curvature of the stomach. During surgery which artery needs to be ligated to
stop the bleeding?
a. Gastroduodenal.
b. Left gastric.
c. Left gastroepiploic.
d. Right gastroepiploic.
e. Short gastric.
P a g e 3 | 26
SurgiNotes 2nd Ed. (2022)

13. A 40-year-old male patient had a superficial parotidectomy operation. Post


operatively he complained of numbness at the lobule of the ear. What was the
injured nerve?
a. Buccal branch of the facial nerve.
b. Cervical branch of the facial nerve.
c. Great auricular nerves.
d. Lesser occipital nerve.
e. Marginal mandibular nerve.
14. A 30-year-old male patient was subjected to a motor car accident and he had only
a fracture of the upper end of the fibula. At examination the patient had sensory
loss at the dorsum of the right foot. Which else may be found on clinical
examination of the patient?
a. Sensory loss at the middle side of the foot.
b. Sensory loss at the sole of the foot.
c. Weakness in dorsiflexion of the foot.
d. Weakness in flexion of the knee.
e. Weakness in planter flexion of the foot.
15. A 25-year-old male patient is involved in a serious car accident. Examination
reveals that his right leg is shortened and internally rotated. The patient is unable
to dorsiflex or planter flex his foot. All sensations below the knee are lost apart
from the medial side of the leg and foot and upper back of the calf. Which of the
following is the MOST likely injured nerve?
a. Common peroneal nerve.
b. Tibial nerve.
c. Obturator nerve.
d. Sciatic nerve.
e. Femoral nerve.
16. A 20-year-old man presented with a stab injury to the anterior aspect of the right
wrist. On examination he was found to have a deep 2 cm long transverse
laceration at the front of the right wrist with loss of sensation in the ring and little
fingers. The patient was unable to abduct and adduct his fingers on the affected
side. Radial pulse was palpable. What was the MOST likely structure to have been
injured?
a. Anterior interosseous nerve.
b. Median nerve.
c. Musculocutaneous nerve.
d. Radial nerve.
e. Ulnar nerve.
P a g e 4 | 26
SurgiNotes 2nd Ed. (2022)

17.A 19-year-old woman attends the emergency department after being shot in
the neck with an air-gun pellet. In the course of surgical exploration of the
posterior triangle, a nerve is injured. The patient is unable to shrug her left
shoulder or fully abduct her left arm. Which is the MOST likely structure to
have been injured?
a. Axillary nerve.
b. Dorsal scapular nerve.
c. Long thoracic nerve.
d. Spinal accessory nerve.
e. Suprascapular nerve.
18.Which of the following statement is TRUE regarding surgical excision of the
right submandibular salivary gland?
a. Injury to the lingual nerve would result in loss of sensations to the
posterior one third of the right side of the tongue.
b. Injury to the hypoglossal nerve would result in deviation of the tongue to
the right side on protrusion of the tongue.
c. The great auricular nerve is at a risk of injury.
d. Injury of the marginal mandibular branch of the facial nerve would result
in sensory loss at the angle of the mandible.
e. Injury to the cervical branch of the facial nerve would result in drooping
of the right angle of the mouth.
19.A 30-year-old male had a stab wound of the front of the forearm. Examination
revealed loss of flexion of the distal phalanges of the thumb and the lateral 3
fingers. Which of the following was the injured nerve?
a. Ulnar nerve.
b. Posterior interosseous nerve.
c. Anterior interosseous nerve.
d. Radial nerve.
e. Musculocutaneous nerve.
20.A 24-year-old woman presents to the outpatient clinic with pelvic pain. A CT
scan reveals enlarged para-aortic lymph nodes. These are MOST likely to be
involved in secondary spread from a tumor in which of the following organs?
a. Cervix.
b. Ovary.
c. Uterus.
d. Vagina.
e. Vulva.

P a g e 5 | 26
SurgiNotes 2nd Ed. (2022)

21.A 64-year-old woman is reviewed in the emergency department with an acute


right groin swelling, below and lateral to the pubic tubercle. Which of the
following forms the medial boundary of the ring through which this hernia is
protruding?
a. Conjoint tendon.
b. Inferior epigastric artery.
c. Inguinal ligament.
d. Lacunar ligament.
e. Pectineal ligament.

22.A 30-year-old motorcyclist suffers a closed fracture to the mid-shaft of the


tibia. Anterior compartment syndrome could cause loss of sensation:
a. In the first web space.
b. Over the dorsum of the foot.
c. Over the lateral edge of the foot.
d. Over the medial aspect of the hallux.
e. Over the medial malleolus.

23.A 42-year-old construction worker noted a swelling in the right submandibular


region. Biopsy reveals malignancy and surgical excision is advised. The patient
is informed that one of the risks of this operation is which of the following?
a. Horner syndrome.
b. Excessive sweating in the temporal region.
c. Deformity of the angle of the mouth.
d. Submandibular duct calculus.
e. Trismus.

24.A 32-year-old man is brought to the emergency department following a fall.


An X-ray reveals that he has a fracture of the surgical neck of his humerus.
Neurological examination reveals paraesthesia over the upper lateral arm
(overlying the deltoid muscle). Which nerve is MOST likely to have been
damaged?
a. Axillary.
b. Median.
c. Musculocutaneous.
d. Radial.
e. Ulnar.

P a g e 6 | 26
SurgiNotes 2nd Ed. (2022)

25.A CT scan of a 63-year-old man reveals that the left renal vein is occluded as it
crosses the aorta. The occlusion is due to compression by an arterial aneurysm
anterior to the vein. What is the MOST likely location of this aneurysm?
a. Coeliac artery.
b. Inferior mesenteric artery.
c. Left colic artery.
d. Middle colic artery.
e. Superior mesenteric artery.

26.A 27-year-old man is admitted to the emergency department after a car crash.
Physical examination reveals weakness in medial rotation and adduction of
the humerus. Which of the following nerves was MOST probably injured?
a. Thoracodorsal.
b. Axillary.
c. Dorsal scapular.
d. Spinal accessory.
e. Radial.

27.A 19-year-old man is brought to the emergency department after dislocating


his shoulder while playing football. Following treatment of the dislocation, he
cannot initiate abduction of his arm. An MRI of the shoulder shows a torn
muscle. Which muscle was MOST likely damaged by the injury?
a. Coracobrachialis.
b. Long head of the triceps.
c. Pectoralis minor.
d. Supraspinatus.
e. Teres major.

28.A 48-year-old female complains of symptoms of carpal tunnel syndrome for


almost a year. Which muscles MOST typically become weakened in this
condition?
a. Dorsal interossei.
b. Lumbricals III and IV.
c. Thenar.
d. Palmar interossei.
e. Hypothenar.
P a g e 7 | 26
SurgiNotes 2nd Ed. (2022)

29.A 60-year-old-male male accidentally injured his wrist with a knife, partially
dividing the ulnar nerve. Which of the following actions would MOST likely be
lost as a result of this injury?
a. Flexion of the proximal interphalangeal joint of the fifth digit (little finger).
b. Extension of the thumb.
c. Adduction of the fifth digit.
d. Abduction of the thumb.
e. Opposition of the thumb.

30.A 43-year-old woman cannot hold a piece of paper between her thumb and
the lateral side of her index finger without flexing the distal joint of her thumb.
Weakness of which specific muscle causes this sign to appear?
a. Flexor pollicis longus.
b. Adductor pollicis.
c. Flexor digiti minimi.
d. Flexor carpi radialis.
e. Extensor indicis.

31.A 34-year-old man is undergoing an emergency laparotomy. Which of the


following anatomic features are the MOST useful to distinguish the jejunum
from the ileum?
a. Jejunum has thinner walls compared with the ileum.
b. Jejunum has less mesenteric fat compared with the ileum.
c. Jejunum has more numerous vascular arcades compared with the ileum.
d. Jejunum has more numerous lymphatic follicles beneath the mucosa
compared with the ileum.
e. Jejunum has fewer villi compared with the ileum.

32.What is the major source of protein in multi-trauma patients during the


catabolic phase?
a. Fat.
b. Kidney.
c. Liver.
d. Plasma protein.
e. Skeletal muscle.
P a g e 8 | 26
SurgiNotes 2nd Ed. (2022)

33.Which of the following cell types is essential for normal wound healing?
a. Erythrocytes.
b. Leukocytes.
c. Lymphocytes.
d. Monocytes.
e. Platelets.

34.A 50-year-old male patient is suffering from septic shock due to leaking
intestinal anastomosis. Which of the following is the best index for the
severity of shock?
a. Creatinine level.
b. Hb %.
c. Lactate level.
d. O₂ saturation of arterial blood.
e. Plasma sodium concentration.

35.Which of the following is TRUE regarding the physiology of the thyroid gland?
a. Absorption of iodides occurs by passive diffusion.
b. Potassium perchlorate prevents the union between iodine and tyrosine.
c. T3 is four times more active than T4.
d. The half-life of thyroxine is 4 days.
e. The necessary intake of iodide is 50 ug/day.

36.Which of the following is the major stimulus for aldosterone secretion?


a. ACTH.
b. Angiotensin II.
c. Cortisol.
d. Hypernatraemia.
e. Hypokalaemia.
P a g e 9 | 26
SurgiNotes 2nd Ed. (2022)

37.Which of the following cells is responsible for the secretion of pepsinogen?


a. Chief cells.
b. Enterochromaffin cells.
c. Mucous cells.
d. Paneth cells.
e. Parietal cells.

38.A 60-year-old male patient is suffering from diabetic ketoacidosis. Which of


the following is the MOST important buffer base in the extracellular fluid?
a. Bicarbonate.
b. Hemoglobin.
c. Lactate.
d. Phosphate.
e. Plasma proteins.

39.The following acid-base data: pH 7.2, PCO₂ 20 mmHg, HCO3- 8 mmol/L and base
excess: -19 mmol/L, would be MOST consistent with:
a. Anxiety.
b. Lobar collapse of the lung.
c. Pyloric obstruction.
d. Septic shock.
e. Starvation.

40.What is the main brain's fuel during prolonged starvation?


a. Amino acid.
b. Glucose.
c. Ketones.
d. Lactose.
e. Short-chain fatty acid.
P a g e 10 | 26
SurgiNotes 2nd Ed. (2022)

41.A 63-year-old man undergoes a peripheral vascular procedure under general


anesthesia. A decrease in urine formation and excretion are noted. Decreased
urine flow under general anesthesia occurs because of which of the following?
a. Vasopressin.
b. Aldosterone suppression.
c. Depression of glucocorticoid.
d. Depression of thyroid function.
e. Specific effect of anesthesia on renal tubules.

42.Which of the following is a function of the parasympathetic system?


a. Increase the blood supply of the heart.
b. Increase the blood supply of skeletal muscles.
c. Increase in the heart rate.
d. Stimulation of glycogenolysis.
e. Increased motility of the gastrointestinal tract.

43.A 25-year-old man presents to the emergency department after being


involved in a road traffic accident. He is conscious, talking, has a blood
pressure of 90/70 mmHg and a heart rate of 100 beats/minute. He has
sustained a fractured pelvis and femur. Which of the following is TRUE?
a. His total peripheral resistance is decreased.
b. The discharge rate of his carotid sinus nerves is increased.
c. There is a decrease in renin production.
d. There is an increase in angiotensin II.
e. There is an increase in renal sodium excretion.

44.Which of the following is characteristic of protein metabolism after trauma?


a. Decreased liver gluconeogenesis.
b. Inhibition of skeletal muscle breakdown by interleukin-1 and tumor
necrosis factor (TNF, cachectin).
c. Decreased urinary nitrogen loss.
d. Hepatic synthesis of acute-phase reactants.
e. Decreased glutamine consumption by fibroblasts, lymphocytes and
intestinal epithelial cells .
P a g e 11 | 26
SurgiNotes 2nd Ed. (2022)

45.A 78-year-old woman with emphysema receiving 28% oxygen by mask has the
following blood gas results:
PH PO2 PaCO2 Bicarbonate Base excess
Finding: 7.28 70 mmHg 48 mmHg 36 mmol/L +5
What is the MOST likely interpretation?
a. Mixed respiratory and metabolic acidosis.
b. Partially compensated metabolic acidosis.
c. Partially compensated respiratory acidosis.
d. Uncompensated metabolic acidosis.
e. Uncompensated respiratory acidosis.

46.A 70-year-old patient with diabetes and paraplegia is undergoing an elective


laparoscopic cholecystectomy after an episode of biliary pancreatitis. Shortly
after induction, blood pressure is normal, but ECG shows peaked T waves and
a widened QRS complex. The MOST likely diagnosis is:
a. Ketoacidosis.
b. Hyperkalemia.
c. Hypoglycemia.
d. Hypocalcemia.
e. Acute myocardial infarction.

47.Which of the following statements regarding wound healing is TRUE?


a. Keloids contain an overabundance of fibroblasts.
b. A hypertrophic scar extends beyond the boundaries of the original
wound.
c. Improvement is usually seen with keloid excision followed by intralesional
steroid injection.
d. An incision placed perpendicular to the lines of natural skin tension will
result in the least obvious scar.
e. Hypertrophic scars occur most commonly on the lower extremities.

48.Which of the following clinical situations can be associated with hypovolemic


hyponatremia?
a. CHF.
b. Syndrome of inappropriate ADH secretion.
c. Cirrhosis.
d. Hyperglycemia.
e. Gastrointestinal losses.
P a g e 12 | 26
SurgiNotes 2nd Ed. (2022)

49.A 35-year-old man is admitted to the ICU following an emergency splenectomy


and nephrectomy for injuries sustained in a car accident. He received 12 units
of packed red blood cells intraoperatively. What electrolyte abnormality is
MOST likely to occur?
a. Hypokalemia.
b. Hyperkalemia.
c. Hypercalcemia.
d. Hypomagnesemia.
e. Hyperphosphatemia.

50.Which of the following is NOT an action of cortisol in a metabolically stressed


patient?
a. It stimulates release of insulin by the pancreas.
b. It induces the insulin resistance in muscles and adipose tissue.
c. It stimulates release of lactate from skeletal muscle.
d. It induces release of glycerol from adipose tissue.
e. It leads to immunosuppression.

51.Which one of the following is LEAST useful in the immediate treatment of


hyperkalemia?
a. Calcium salts.
b. Sodium bicarbonate.
c. Potassium-binding resins.
d. Glucose and insulin.
e. Hemodialysis.

52.A 40-year-old male patient was injured in the thigh and a bleeding point was
ligated by a silk suture. The patient got recurrent attacks of inflammation at
the site of the wound. The surgeon explored the wound and removed an
inflammatory mass around the silk suture. Which of the following cells will be
the predominant cell detected in this mass?
a. Basophils.
b. Eosinophils.
c. Lymphocyte.
d. Monocytes.
e. Neutrophils.

P a g e 13 | 26
SurgiNotes 2nd Ed. (2022)

53.Which of the following cells is an example of a permanent cell NOT capable of


division?
a. Acinar cells of the pancreas.
b. Colonic mucosal cells.
c. Erythrocytes.
d. Hepatocytes.
e. Osteocytes.

54.In patients with reflux esophagitis. What does the presence of columnar cells
in the esophageal mucosa represent?
a. Carcinoma in situ.
b. Carcinoma.
c. Dysplasia.
d. Hyperplasia.
e. Metaplasia.

55.Which of the following cytokines has anti-inflammatory properties?


a. Granulocyte macrophage colony-stimulating factor.
b. IL-10.
c. IL-2.
d. IL-3.
e. Tumor necrosis factor-α.

56.Which of the following microscopic features MOST likely indicates that a


neoplasm is malignant?
a. Atypia.
b. Increased nuclear : cytoplasmic ratio.
c. Invasion.
d. Necrosis.
e. Pleomorphism.
P a g e 14 | 26
SurgiNotes 2nd Ed. (2022)

57.A 55-year-old male patient is receiving 150 mg of aspirin daily because he has
coronary artery disease. Which of the following tests will be affected in this
patient?
a. Bleeding time.
b. Coagulation time.
c. INR.
d. PTT.
e. Thrombin time.

58.A 30-year-old male patient was admitted to the casualty department due to a
car accident. The patient had fracture of the pelvis and the right femur and he
received 5 liters of blood following which he started to have bleeding from his
nose and mouth. What is the main cause of this bleeding tendency?
a. Decrease in fibrinogen.
b. Decrease in prothrombin.
c. Decrease of calcium.
d. Increased fibrinolytic activity.
e. Platelet depletion.

59.A patient with a life threatening pulmonary embolus is receiving heparin. She
developed serious vaginal bleeding and a major retroperitoneal haematoma
after 5 days of heparin therapy. What is the recommended treatment?
a. Reverse heparin and evacuate the haematoma.
b. Reverse heparin by protamine sulphate and insert a vena caval filter.
c. Stop heparin and closely observe the patient.
d. Stop heparin, give fresh frozen plasma and start warfarin therapy.
e. Switch to low-dose heparin.

60.Which of the following is the MOST likely cause of increased prothrombin


time?
a. Christmas disease.
b. Hemophilia.
c. Heparin therapy.
d. Von Willebrand’s disease.
e. Warfarin overdose.
P a g e 15 | 26
SurgiNotes 2nd Ed. (2022)

61.What is the main mode of action of heparin?


a. It increases the level of protein C.
b. It is a cofactor of antithrombin III.
c. It prevents clot retraction.
d. It prevents platelets aggregation.
e. It prevents the synthesis of fibrinogen.

62.You were obliged to perform a cholecystectomy for a cirrhotic patient. During


surgery there was excessive bleeding. What would you order?
a. Fresh blood.
b. Fresh frozen plasma.
c. Fresh platelets.
d. IV vitamin K.
e. IV factor VIII.

63.You are planning to perform cholecystectomy for a female patient who is


receiving warfarin 5 mg daily because she had mitral valve replacement 4
years ago. What is your pre-operative management?
a. Continue warfarin and give fresh frozen plasma intraoperatively.
b. Continue warfarin and give fresh platelets intraoperatively.
c. Continue warfarin and give IV vitamin K intraoperatively.
d. Stop warfarin and substitute it by heparin for 10 days preoperatively.
e. Stop warfarin and substitute it by heparin for 5 days preoperatively.

64.What does increased level of fibrin degradation products (FDP) in the blood
denote?
a. Diminished fibrinogen synthesis.
b. Diminished platelet count.
c. Diminished platelet function.
d. Disseminated intravascular coagulation (DIC).
e. Heparin overdose.
P a g e 16 | 26
SurgiNotes 2nd Ed. (2022)

65.A cirrhotic patient with abnormal hemostatic studies requires an urgent


cholecystectomy. A transfusion of fresh frozen plasma is planned to minimize
the risk of bleeding. What is the optimal timing of this transfusion?
a. In the recovery room.
b. Intraoperatively.
c. On call to surgery.
d. The night before surgery.
e. Two days before surgery.

66.A fully heparinized patient develops a condition requiring emergency surgery.


After stopping the heparin, what else should be done to prepare the patient?
a. 2 units of cryoprecipitates.
b. Administration of protamine sulphate 1 mg for every 100 units of heparin
most recently administered.
c. Immediate fresh frozen plasma.
d. Transfusion of 10 units of platelets.
e. Vitamin K intravenously.

67.A 5-year-old boy slipped and hurt his right knee while walking. He presents
with a tender, swollen, warm knee with significant hemarthrosis. His PT is 12
(normal: 13 seconds), PTT is over 100 (normal: 25 seconds), platelet count is
300,000/mm³ and bleeding time is normal. Initial management should consist
of which of the following?
a. Fresh-frozen plasma.
b. Aspiration of knee.
c. Factor VIII concentrate.
d. Passive exercise.
e. Long-leg cast.

68.Which of the following denotes a hemolytic transfusion reaction during


anesthesia?
a. Shaking chills and muscle spasms.
b. Fever and oliguria.
c. Hyperpyrexia and hypotension.
d. Tachycardia and cyanosis.
e. Bleeding and hypotension.
P a g e 17 | 26
SurgiNotes 2nd Ed. (2022)

69.A 50-year-old female patient has chronic renal failure and has been
maintained an chronic dialysis. The patient underwent cholecystectomy. Post-
operatively she had severe bleeding. What is the MOST likely cause for this
bleeding?
a. Elevated PT.
b. Elevated PTT.
c. Low platelet count.
d. Decreased platelet aggregation.
e. Sepsis.
70.A 70-year-old female patient is receiving warfarin because she has AF and had
previous thrombosis. Which of the following statements regarding warfarin is
correct?
a. The dose of warfarin is adjusted according to the partial thromboplastin
time.
b. Warfarin takes about 8 hours to exert its effects.
c. Warfarin acts by inhibiting factor XII.
d. If the patient is also taking aspirin, the dose of warfarin should be
reduced.
e. Protamine sulphate is the antidote to warfarin.
71.In the awake, non-anesthetized patient suspected of having a hemolytic post-
transfusion reaction, the MOST characteristic signs are:
a. Nausea and vomiting.
b. Fever and chills.
c. Oliguria and hemoglobinuria.
d. Cyanosis and dyspnea.
e. Tenderness of the renal angle.
72.On postoperative day 4 after a Whipple procedure, a patient develops
symptoms consistent with massive pulmonary embolism. Computed
tomography confirms a saddle embolus. After emergent lytic therapy with
urokinase, the patient is bleeding from the surgical site. Which of the following
options is the best choice to treat the bleeding?
a. Aminocaproic acid.
b. Cryoprecipitate.
c. Dextran.
d. Fresh frozen plasma.
e. Packed red blood cells.
P a g e 18 | 26
SurgiNotes 2nd Ed. (2022)

73.Regarding low molecular weight heparin, which of the following is TRUE?


a. Activity is effectively measured by activated partial thromboplastin time
(APTT).
b. Strongly binds to plasma proteins.
c. Has a longer plasma half-life than a standard heparin.
d. Has its effect reversed by an equivalent dose of protamine.
e. Has a normal plasma clearance in patients with renal failure.

74.Trauma patients sustaining what type of injury are at highest risk of venous
thromboembolism?
a. Head trauma.
b. Femur fracture.
c. Pelvic fracture.
d. Splenectomy.
e. Spinal cord injury.

75.Which of the following is the MOST effective way in preventing surgical site
infection?
a. Antibiotic prophylaxis.
b. Bowel preparation.
c. Drains and irrigation.
d. Skin preparation.
e. Surgical technique.

76.Which of the following is TRUE regarding antibiotics?


a. Aminoglycosides may cause hepatotoxicity.
b. Ampicillin in effective against pseudomonas infections.
c. Cephalosporins are rarely used for prophylaxis.
d. Penicillins act by disrupting the peptidoglycan of the bacterial cell wall.
e. Vancomycin is the first choice for treatment of infections with staph.
aureus.
P a g e 19 | 26
SurgiNotes 2nd Ed. (2022)

77. A 45-year-old diabetic male patient complains of severe pain in the thigh.
Examination reveals spreading oedema, swelling and tenderness of the thigh. A
provisional diagnosis of necrotizing fasciitis is made. Which of the following is
MOST important in the treatment?
a. Antifungal agents.
b. Antitoxin.
c. Hyperbaric O₂.
d. Immunoglobulins.
e. Wide surgical debridement.

78. A patient with a non-obstructing carcinoma of the sigmoid colon being prepared
for elective resection. Which of the following reduces the risk of postoperative
infectious complications?
a. Avoidance of oral antibiotics to prevent emergence of Clostridium difficile.
b. Postoperative administration for 48 h of parenteral antibiotics effective
against aerobes and anaerobes.
c. Postoperative administration of parenteral antibiotics effective against
aerobes and anaerobes until the patient's intravenous lines and all other
drains are removed.
d. Redosing of antibiotics in the operating room if the case lasts for more than
2 hours of operating time.
e. Single preoperative parenteral dose of antibiotic effective against aerobes.

79. A 40-year-old male patient had renal transplantation and is receiving


immunosuppression. Few days following the operation the patient developed
persistent fever. Which of the following infectious microorganisms is currently
the MOST likely cause of this fever?
a. Candidiasis.
b. Coli sepsis.
c. Cytomegalovirus sepsis.
d. Pneumococcal sepsis.
e. Streptococci.

80. A 30-year-old female patient developed postoperative wound infection after a


thyroidectomy operation. What is the MOST likely causative organism?
a. Bacteroides
b. E. coli.
c. Proteus.
d. Staph. aureus.
e. Streptococci.
P a g e 20 | 26
SurgiNotes 2nd Ed. (2022)

81.Which of the following is the mode of action of ciprofloxacin?


a. Blockage of bacterial DNA replication.
b. Competitive inhibition of the enzyme dihydrofolate reductase.
c. Inhibition of bacterial wall synthesis.
d. Inhibition of protein synthesis by ribosomes.
e. Inhibition of translocation of peptides.

82.A 30-year-old woman sustained a puncture wound to the foot. The patient has
been on a therapeutic dose of steroids for the past 5 years for ulcerative colitis.
Her last tetanus toxoid booster was 8 years ago. What should the patient
receive?
a. Tetanus toxoid booster.
b. Human immunoglobulin, antibiotics with anaerobic coverage.
c. Tetanus toxoid plus human immunoglobulin.
d. Tetanus toxoid plus human immunoglobulin and antibiotics with aerobic
and anaerobic coverage.
e. Wide debridement of the wound.

83.Which of the following measures is MOST likely to reduce the risk of


postoperative wound infection with MRSA?
a. 5 days of broad spectrum prophylactic antibiotics.
b. Ensure that the patient showers with chlorhexidine wash prior to surgery.
c. A policy of staff handwashing between patients.
d. Screening patients for MRSA carriage prior to surgery.
e. Preoperative shaving the area of incision.

84.Treatment of wounds with negative pressure wound dressings (VAC dressings)


may result in all of the following EXCEPT:
a. Increased apoptosis.
b. Increased granulation tissue.
c. Improved microvascular blood flow to wound edges.
d. Removal of excess fluid and debris.
e. Lower bacterial counts.
P a g e 21 | 26
SurgiNotes 2nd Ed. (2022)

85.A 45-year-old woman undergoes an uneventful laparoscopic cholecystectomy


for which she receives one dose of cephalosporin. One week later, she returns
to the emergency room with fever, nausea and copious diarrhea and is
diagnosed with pseudomembranous colitis. Which one of the following
statements is correct?
a. Surgical intervention is frequently required.
b. After appropriate antibiotic therapy, the relapse rate is less than 5%.
c. Tissue culture assay for Clostridium difficile toxin B is neither sensitive nor
specific; therefore diagnosis should be based on clinical findings.
d. If surgery is performed a left hemicolectomy is usually adequate to treat
pseudo membranous colitis.
e. Indications for surgical treatment include intractable disease, failure of
medical therapy, toxic megacolon and colonic perforation.

86.What is the oral antibiotic MOST likely to be effective against a pseudomonas


urinary tract infection?
a. Ciprofloxacin.
b. Trimethoprim-sulfamethoxazole.
c. Cephalexin.
d. Nitrofurantoin.
e. Amoxicillin.

87.Which of the following is TRUE regarding necrotizing fasciitis?


a. Skin manifestations may be totally absent.
b. It is commonly a single microbial infection.
c. The muscles are usually involved.
d. Broad spectrum antibiotics are the most essential measure of treatment.
e. It commonly occurs in previously healthy people.

88.Renal transplant patients have an increased incidence of:


a. Adenocarcinoma of the uterus.
b. Nonmelanoma skin cancer.
c. Lobular breast cancer.
d. Colonic carcinoid.
e. Lung cancer.
P a g e 22 | 26
SurgiNotes 2nd Ed. (2022)

89.Which of the following statements about the treatment of necrotizing fasciitis


is TRUE?
a. The tissue underlying necrosis is reflected by the extent of skin necrosis.
b. Intravenous immune globulin (IVIG) is the first line of therapy.
c. Penicillin provides sufficient antibiotic coverage.
d. Hyperbaric oxygen has been shown to improve survival.
e. Exploratory incisions over normal-appearing skin are effective in
determining the extent of the necrosis.
90.The clinical course of the majority of patients with HCV infection is
characterized by which one of the following?
a. Acute constitutional symptoms and jaundice.
b. Acute fulminant hepatic failure.
c. Development of chronic hepatitis.
d. Progression to cirrhosis.
e. Development of hepatocellular carcinoma .
91.A surgical resident sustains a needle stick with a hollow-bore needle
contaminated with the blood of a patient who is hepatitis B antigen positive.
The resident completed a series of three hepatitis B vaccines one-year ago, but
his antibody response was not checked. Which of the following statements
BEST describe management of this case?
a. Observation only is indicated since the source does not have active HBV
infection.
b. The resident needs a booster of hepatitis B vaccine.
c. The resident should receive HBIG immediately.
d. The resident should receive HBIG and a hepatitis B vaccine immediately.
e. The resident needs to be tested for anti-hepatitis B antibody immediately.
If the test insult is negative, proceed as in alternative D.
92.Which of the following statements about the usage of antibiotics for the
prevention of surgical site infection is NOT true?
a. Antibiotics should be administered within 60 minutes of incision.
b. The therapeutic dose of the antibiotic should be administered
intravenously.
c. Adequate tissue concentrations should be maintained during operation
by re-dosing as necessary.
d. Antibiotics should be continued for 48 hours after operation.
e. Vancomycin is the agent of choice for patients from nursing homes
undergoing hip replacement.

P a g e 23 | 26
SurgiNotes 2nd Ed. (2022)

93.The earliest manifestations of serious gram-negative infection may consist of


a triad of signs that includes:
a. Tachypnea, hypotension and an altered sensorium.
b. Tachypnea, hypotension and lactic acidosis.
c. Thrombocytopenia, hypotension and lactic acidosis.
d. Mild hyperventilation, respiratory alkalosis and an altered sensorium.
e. Tachycardia, hypotension and metabolic acidosis.

94.Postoperative wound infections after operations for head and neck cancer are
MOST frequently caused by:
a. Bacillus fragilis.
b. Escherichia coli.
c. Gram negative anaerobes.
d. Staphylococcus aureus.
e. Pseudomonas aeruginosa.

95.What is the MOST common bacterial organism present in the colon?


a. Bacteroides.
b. Clostridium difficile.
c. Escherichia coli.
d. Salmonella.
e. Streptococci.

96.After being struck by a moving truck, a 23-year-old woman undergoes a


splenectomy during diagnostic laparotomy. She leaves the hospital against
medical advice on postoperative day 4, after refusing vaccination. Infection
with which of the following organisms is MOST likely to result in her
developing sepsis?
a. Beta-hemolytic streptococcus.
b. Candida albicans.
c. Clostridium difficile.
d. Escherichia coli.
e. Pseudomonas aeruginosa.
P a g e 24 | 26
SurgiNotes 2nd Ed. (2022)

97.Septic shock is characterized by:


a. Increased capillary permeability.
b. Vasoconstriction.
c. A low cardiac output.
d. A high systemic vascular resistance.
e. Bradycardia.

98.Following urinary tract infection associated with extraction of a stone, a 64-


year-old woman developed gram-negative septicemia. Which statement is
TRUE for gram-negative bacterial septicemia?
a. Pseudomonas is the most common organism isolated.
b. Many of the adverse changes can be accounted for the endotoxin release.
c. The cardiac index is low.
d. Central venous pressure (CVP) is high.
e. Endotoxin is mainly a long-chain peptide.

99.Which of the following is NOT characteristic of aminoglycosides?


a. Active against a broad spectrum of gram-negative aerobes and useful for
synergy against some gram-positive cocci.
b. Emergence of resistant bacterial stains.
c. Narrow margin between therapeutic and toxic blood levels.
d. Nephrotoxicity, ototoxicity and neuromuscular paralysis.
e. Excellent activity in abscesses in which gram negative organisms are
involved.

100. The MOST common source of bacteria in wound infection after groin hernia
repair is:
a. The patient skin.
b. The patient nasopharynx.
c. Operating room air.
d. Surgical instruments.
e. Gastrointestinal flora.
P a g e 25 | 26
SurgiNotes 2nd Ed. (2022)

Egyptian Board Of General Surgery


1st Part Exam – Paper (1) MCQ "ANSWER"
October 2020

1 c 26 a 51 c 76 d
2 e 27 d 52 d 77 e
3 d 28 c 53 c 78 d
4 b 29 c 54 e 79 c
5 d 30 b 55 b 80 d
6 d 31 b 56 c 81 a
7 e 32 e 57 a 82 d
8 c 33 d 58 e 83 c
9 c 34 c 59 b 84 a
10 a 35 c 60 e 85 e
11 a 36 b 61 b 86 a
12 b 37 a 62 b 87 a
13 c 38 a 63 e 88 b
14 c 39 d 64 d 89 e
15 d 40 c 65 c 90 c
16 e 41 a 66 b 91 e
17 d 42 e 67 c 92 d
18 b 43 d 68 e 93 d
19 c 44 d 69 d 94 d
20 b 45 c 70 d 95 a
21 d 46 b 71 b 96 a
22 a 47 c 72 d 97 a
23 c 48 e 73 c 98 b
24 a 49 b 74 e 99 e
25 e 50 a 75 a 100 a

P a g e 26 | 26
SurgiNotes 2nd Ed. (2022)

Egyptian Board Of General Surgery


1st Part Exam – Paper (2) MCQ
October 2020
1. The arterial blood gas analysis of a patient was pH 7.5, PCO2 47 mmHg and
HCO3- 35 mmol/L. This patient MOST likely is suffering from:
a. Chronic obstructive pulmonary disease.
b. Diabetic ketoacidosis.
c. Persistent diarrhea.
d. Profound vomiting.
e. Salicylate poisoning.
2. Five days after an uneventful cholecystectomy, an asymptomatic middle aged
woman is found to have a serum sodium level of 120 mEq/L. What is the
proper management?
a. Administration of hypertonic saline solution.
b. Administration of Ringer's lactate solution.
c. Aggressive diuresis with furosemide.
d. Plasma ultra-filtration.
e. Restriction of free water.
3. A 65-year-old diabetic woman with chronic steroid-dependent bronchospasm
had hemicolectomy for a perforated cecum. The patient was intubated and
maintained on broad-spectrum antibiotics, renal-dose dopamine and a rapid
steroid taper. On postoperative day 2, she developed a fever of 39.2 oC,
hypotension and lethargy. Laboratory investigations revealed hypoglycemia
and hyperkalemia. What is the MOST likely diagnosis of this acute event?
a. Adrenal insufficiency.
b. Diabetic ketoacidosis.
c. Hypovolemia.
d. Insulin overdose.
e. Sepsis.
4. What is the MOST common fluid disorder in the surgical patient?
a. Extracellular fluid deficit.
b. Hyperkalemia.
c. Hyponatremia.
d. metabolic acidosis.
e. Metabolic alkalosis.

P a g e 1 | 26
SurgiNotes 2nd Ed. (2022)

5. The osmolarity of the extracellular fluid space is determined primary by a


concentration of:
a. Bicarbonate.
b. Chloride ion.
c. Phosphate radicals.
d. Potassium ion.
e. Sodium ion.

6. Which of the following is a characteristic of neurogenic shock?


a. Cool moist skin.
b. Decreased blood volume.
c. Increased peripheral vascular resistance.
d. Increased cardiac output.
e. Bradycardia.

7. A 28-year-old male was injured in a motorcycle accident. On admission he was


in severe respiratory distress and appeared cyanotic (with blood pressure of
80/40 mmHg). The patient was bleeding profusely from the nose and had an
obviously open femur fracture with exposed bone. Breath sounds were
decreased on the right side of the chest. What should be the initial
management?
a. Control of hemorrhage with anterior and posterior nasal packing.
b. Tube thoracostomy in the right hemithorax.
c. Endotracheal intubation with in-line cervical traction.
d. Obtain intravenous access and begin emergency blood transfusion.
e. Obtain cross-table cervical spine film and chest film.

8. What is the MOST common symptom after major pulmonary embolism?


a. Cough.
b. Dyspnea.
c. Hemoptysis.
d. Pleural pain.
e. Palpitation.
P a g e 2 | 26
SurgiNotes 2nd Ed. (2022)

9. Following an operation and extubation, a patient was restless. His arterial


blood gases revealed: pH 7.36, PaO2 65 mmHg, PaCO2 55 mmHg and HCO3- 36
mmol/L. The physiologic status can best be described as which of the
following?
a. Respiratory alkalosis.
b. Respiratory acidosis.
c. Metabolic acidosis.
d. Metabolic alkalosis.
e. Combined respiratory and metabolic acidosis.

10.What is the predominant metabolic abnormality in patients with pyloric


obstruction?
a. Respiratory alkalosis.
b. Hyperchloremic alkalosis.
c. Salt-losing enteropathy.
d. Intrinsic renal disease.
e. Metabolic acidosis.

11.A 68-year-old man has a history of myocardial infarction. He undergoes


uneventful left hemicolectomy for carcinoma of the colon. In the recovery
room, he is hypotensive and is given a fluid bolus of 500 ml Ringer's lactate
over 30 minutes. He is intubated and his neck veins are distended. His HR is
130 bpm, his BP is 80/60 mmHg and his urine output is 20 ml over the last
hour. What should be the next step in his management?
a. Administration of Ringer's lactate 500 ml over 1 hour.
b. Administration of dopamine.
c. Insertion of a Swan-Ganz catheter.
d. Administration of lasix.
e. Extubation of the patient.

12.A 60-year-old male patient had a colectomy operation. Post-operatively the


patient had tachycardia, hypertension and shallow respiration. Which of the
following is an indication to intubate the patient?
a. PaCO2 of 45 mmHg.
b. Respiratory rate 25/min.
c. PO2 of 55 mmHg at room temperature.
d. Heart rate of 130/min.
e. High pulmonary capillary wedge pressure.
P a g e 3 | 26
SurgiNotes 2nd Ed. (2022)

13.Which of the following metabolic effects is present in shock?


a. Increase in sodium and water excretion.
b. Increase in renal perfusion.
c. Decrease in cortisol levels.
d. Hyperkalemia.
e. Hypoglycemia.

14.A 65 year old man is noted to have a blood pressure of 90/62 mmHg on the
evening after an uncomplicated small bowel resection for obstruction. His
heart rate is 110/mm, respiratory rate 24/mm and temperature 37.4 oC, urine
output is only 20 ml over 2 hours and oxygen saturation by pulse oximetry is
95%. His pre-operative hemoglobin was 12.6 g/dl. Which of the following
statements is the MOST accurate regarding the patient?
a. Initial therapy should be intravenous crystalloid fluid bolus.
b. Intravenous furosemide (lasix) should be administered.
c. This patient is most likely affected by anxiety and a mild anxiolytic and
careful observation should be initiated.
d. A hemoglobin level performed in the recovery room after surgery of 12.4
g/dl is good evidence against active hemorrhage.
e. Urgent CT abdomen is required.

15.A patient who had gastrectomy 10 days ago developed massive pulmonary
embolism proved by CT angiography. What is the recommended treatment?
a. IV heparin.
b. Fibrinolytic agent.
c. Warfarin.
d. IV acetylsalicylic acid.
e. Corticosteroids.

16.Which of the following is the commonest cause of ARDS?


a. Sepsis syndrome.
b. Aspiration.
c. Acute pancreatitis.
d. DIC.
e. Fat embolism.
P a g e 4 | 26
SurgiNotes 2nd Ed. (2022)

17.A 70-year-old man with chronic obstructive pulmonary disease is admitted for
elective hemicolectomy. What is a preoperative arterial blood gas analysis
likely to show?
Arterial PCO2: Bicarbonate:
a. Decreased. Decreased.
b. Decreased. Increased.
c. Decreased. Normal.
d. Increased. Decreased.
e. Increased. Increased.

18.A 39-year-old woman making a slow but adequate recovery after sustaining a
40% surface area burn injury. on the sixth postoperative day she becomes
unwell. She vomits intermittently, has painless abdominal distension and
starts to hiccup. What is the MOST likely causes of these symptoms?
a. Acute gastric dilatation.
b. Acute intestinal obstruction.
c. Clostridium difficile infection.
d. Fecal impaction.
e. Systemic sepsis.

19.A 60-year-old man had undergone exploratory laparotomy for perforated


gastric ulcer with severe peritoneal contamination. Six hours after surgery, he
is tachycardic, hypertensive and has shallow respirations. Intubation and
institution of ventilatory support is indicated in the presence of which of the
following?
a. Respiratory rate of 23 breaths/min.
b. PaCO2 of 45 mmHg.
c. PaO2 of 55 mmHg on room air.
d. Heart rate of 140 bpm.
e. BP of 100/70 mmHg.

20.Which of the following types of shock is associated with high pulmonary


wedge pressure?
a. Hypovolemic shock.
b. Cardiogenic shock.
c. Early septic shock.
d. Late septic shock.
e. Neurogenic shock.
P a g e 5 | 26
SurgiNotes 2nd Ed. (2022)

21.A common drug that can trigger an episode of malignant hyperthermia:


a. Propofol.
b. Dantrolene.
c. Ketamine.
d. Succinylcholine.
e. Neostigmine.

22.A middle-aged woman is brought to the emergency department following a


head injury in a car accident. The patient is unconscious and there is bruising
over the upper abdomen. Blood pressure is 80 mmHg systole, pulse
120/minute. What is the MOST important initial step in the management of
this patient?
a. X-ray (three views) of abdomen.
b. Immediate laparotomy.
c. Check airway, breathing and circulation.
d. Perform emergency burr holes.
e. Diagnostic peritoneal lavage.

23.Choose the TRUE statement about septic shock:


a. Gram positive organisms do not cause septic shock because they lack the
cell endotoxin.
b. Leucopenia is not a feature of septic shock.
c. The second most common source of gram-negative bacteremia is the
urinary tract.
d. Escherichia coli is the second most common organism identified in gram
negative bacteremia.
e. The toxic effects of gram-negative endotoxin appear to be the result of
abnormal activation of normal physiologic pathways.

24.The anion gap will increase with an increase in the plasma concentration of:
a. Sodium.
b. Potassium.
c. Chloride.
d. Bicarbonate.
e. Lactate.
P a g e 6 | 26
SurgiNotes 2nd Ed. (2022)

25. A 78-year-old woman with emphysema receiving 28% oxygen by mask has the
following blood gas results:
PH PO2 PaCO2 Bicarbonate Base excess
Finding: 7.28 70 mmHg 48 mmHg 36 mmol/L +5
Normal: 7.35-7.45 90-110 35-45 22-26 -2 to +2
The MOST likely interpretation is:
a. Mixed respiratory and metabolic acidosis.
b. Partially compensated metabolic acidosis.
c. Partially compensated respiratory acidosis.
d. Uncompensated metabolic acidosis.
e. Uncompensated respiratory acidosis.
26. A 28-year-old motorcyclist is admitted following a road traffic accident, having
sustained bilateral femoral fractures and a ruptured spleen. Three days
postoperatively he is noted to be agitated, hypoxaemic and difficult to
ventilation with a BP of 120/80 mmHg, regular pulse of 88 beats/minute and he
is apyrexial. A chest X-ray shows bilateral diffuse lung infiltrates. What is the
MOST likely underlying diagnosis?
a. Adult respiratory distress syndrome (ARDS).
b. Atelectasis.
c. Bronchopneumonia.
d. Pulmonary oedema.
e. Pulmonary thrombo-embolism.
27. A 22-year-old man involved in a motor vehicle accident is found to have a thoracic
spine fracture (T6) and paraplegia. The patient is hypotensive with a systolic BP
of 70 mmHg, is bradycardiac with a pulse of 48 beats/min and is breathing
comfortably. Which of the following would the MOST appropriate initial
treatment?
a. Isotonic fluid administration.
b. Steroid administration within 24 hours of the injury.
c. Immediate intubation.
d. Alpha-agonist administration.
e. Immediate magnetic resonance imaging.
28. Which of the following is TRUE regarding nosocomial pneumonia among
intensive care unit patients?
a. Has the same mortality rate as does community-acquired pneumonia.
b. Is the most common nosocomial infection.
c. Can be avoided by early tracheostomy.
d. Is directly related to the duration of intubation.
e. Can be prevented by early institution of prophylactic antibiotics.
P a g e 7 | 26
SurgiNotes 2nd Ed. (2022)

29.With regard to complications of blood transfusion, which of the following


statements is TRUE?
a. Febrile reactions are regarded rare.
b. Transfusions lasting more than 6 hours increase the risk of infection by
contaminated blood.
c. One hundred ml of intravenous air is well tolerated.
d. Gram positive organisms are the commonest contaminants of stored
blood.
e. Larval stage of hydatid disease can be transmitted by blood transfusion.

30.A 55-year-old male patient has been receiving TPN for prolonged time and
developed hypomagnesaemia which of the following conditions clinically
resembles hypomagnesaemia?
a. Hypoglycemia.
b. Hypokalemia.
c. Hypophosphatemia.
d. Hypocalcemia.
e. Hyponatremia.

31.Which of the following are effects of epinephrine in response to injury?


a. It enhances the adherence of leukocytes to vascular endothelial
membranes.
b. It stimulates the release of aldosterone.
c. It inhibits the secretion of thyroid hormones.
d. It increases glucagon secretion.
e. It decreases lipolysis in adipose tissue.

32.The gold standard for diagnosing pulmonary embolism (PE) is:


a. CT pulmonary angiogram.
b. MRI.
c. Ventilation perfusion nuclear scans (VQ scans).
d. Duplex ultrasound U/S.
e. ECG.
P a g e 8 | 26
SurgiNotes 2nd Ed. (2022)

33.Which of the following is the best parameter for monitoring septic shock?
a. Central venous pressure (CVP).
b. Vasopressor requirement.
c. Urine Output.
d. Serum lactate.
e. Mental status changes.

34.Which of the following is TRUE about the catabolic response to trauma?


a. Intravenous hyperalimentation can prevent the catabolic response to
trauma.
b. Liver glycogen is the source of dextrose in the first week.
c. The catabolic response is initiated by the thyroid hormones.
d. The catabolic response is the same regardless of the severity of trauma.
e. There is inevitable loss of muscle mass.

35.Which of the following statements regarding hypervolemia in postoperative


patients is TRUE?
a. Hypervolemia can be produced by the administration of isotonic salt
solutions in amounts that exceed the loss of volume.
b. Acute overexpansion of the ECF space is typically not well tolerated in
healthy individuals.
c. Excess administration of normal saline can result in metabolic
derangement, most commonly hyperchloremic metabolic alkalosis.
d. The most reliable sign of volume excess is peripheral edema.
e. Daily weight measurement in the postoperative period does not help
determine fluid status.

36.Which of the following substances has been shown to be useful as a


measurable marker of the response to injury?
a. Tumor necrosis factor- α (TNF-α).
b. Interleukin-2 (IL-2).
c. IL-6.
d. IL-10.
e. C-reactive protein (CRP).
P a g e 9 | 26
SurgiNotes 2nd Ed. (2022)

37.A 35-year-old woman presents to you after running her first marathon with
complaints of muscle aches. Which of the following is NOT an action of cortisol
in this metabolically stressed patient?
a. It stimulates release of insulin by the pancreas.
b. It induces insulin resistance in muscles and adipose tissue.
f. It stimulates release of lactate from skeletal muscle.
c. It induces release of glycerol from adipose tissue.
d. It leads to immunosuppression.

38.Which of the following metabolic changes is MOST prominent during times of


physiologic stress?
a. Increase in growth hormone (GH) release.
b. Increase in TSH.
c. Increased levels of T4 and T3.
d. Initial insulin increase and then suppression.
e. Increase in cortisol excretion.

39.Which of the following statements relating to large volume blood loss in


trauma is incorrect?
a. Tranexamic acid reduces the incidence of rebleeding following surgery.
b. Hypocalcaemia may complicate resuscitation.
c. Colloids are preferred initially as they reduce the incidence of
coagulopathy.
d. When patients receive over 5 units of whole blood, mortality increases
when blood products greater than 3 weeks old are utilized.
e. In the battlefield setting a ratio of one unit fresh blood to one unit plasma
is often utilized.

40.Maintaining perioperative serum glucose levels between 80 and 110 mg/dl in


diabetic patients undergoing cardiac surgery may:
a. Have no effect on postoperative complications.
b. Increase the incidence of deep sternal wound infections.
c. Increase the incidence of hypoglycemia.
d. Promote osmotic diuresis.
e. Decrease in-hospital mortality.
P a g e 10 | 26
SurgiNotes 2nd Ed. (2022)

41.A 45-year-old male patient had renal transplantation operation. Thirty


minutes after the operation the urinary output markedly dropped. The blood
pressure of was 120/80 mmHg. Duplex scanning of the transplanted kidney
revealed patent renal vessels and normal pelvicalyceal system. Which of the
following is TRUE regarding the present problem?
a. It is less liable to occur in females who had repeated pregnancies.
b. It is more liable to occur in patients who receive liver transplantation.
c. Stimulated CD4 lymphocytes are mainly responsible for the problem.
d. The patient needs immediate removal of the transplanted kidney.
e. The problem can be corrected by increasing the dose of cyclosporine.

42.Which of the following cells is responsible for cellular mediated immunity?


a. B-lymphocytes.
b. Dendritic cells.
c. Monocytes.
d. Natural killer cells.
e. T-lymphocytes.

43.A 40-year-old male patient underwent a renal transplant from a cadaveric


donor. Immediately after finishing the vascular anastomosis, the kidneys
became cyanotic and flaccid. Histological examination revealed deposition of
immunoglobulins and complement in the vessel walls. What is the
immunological background of the problem?
a. Donor cytotoxic T-lymphocytes directed against the front antigens.
b. Donor natural killer cells directed against host antigens.
c. Host natural killer cells against donor antigens.
d. Preformed donor antibodies against the host antigens.
e. Preformed host antibodies against the donor antigens.

44.Which of the following is a major disadvantage of living donor organ


transplantation?
a. Elective procedure.
b. Immediate graft function.
c. Improved graft survival.
d. Mortality and morbidity of donors.
e. Shorter hospital stay.
P a g e 11 | 26
SurgiNotes 2nd Ed. (2022)

45.Which of the following is commonest vascular complication which may follow


hepatic living related donor transplantation?
a. Portal vein stenosis.
b. Hepatic artery thrombosis.
c. Hepatic artery stenosis.
d. Inferior vena cava thrombosis.
e. Hepatic vein stenosis.

46.Which of the following allogeneic grafts does NOT require


immunosuppression?
a. Kidney.
b. Heart.
c. Liver.
d. Bone marrow.
e. Cartilage.

47.One week after receiving a cadaver renal allograft, the recipient remains
oliguric and dialysis dependent. Ultrasonography reveals a large perigraft fluid
collection. Appropriate next step in the management of this patient is:
a. Observation.
b. Aspiration of the perigraft fluid and installation of fibrosis inducing agent
to obliterate the dead space.
c. Aspiration of the perigraft fluid collection for chemical analysis.
d. CT of the abdomen.
e. Angiography for localization of the bleeding site.

48.A 37-year-old woman with end-stage renal disease had a cadaveric renal
transplant 18 months previously. She tolerated her transplantation well and
has been receiving a stable regiment of tacrolimus and corticosteroids with a
creatinine of 1.0 mg/dl. She now presents with fatigue, bone pain and
osteoporosis. Her serum calcium is 11.5 mg/dl. The next step in management
should be:
a. Subtotal parathyroidectomy.
b. An increase in corticosteroid dose.
c. Hydration and diuresis.
d. Calcitonin injections.
e. Phosphate binders.
P a g e 12 | 26
SurgiNotes 2nd Ed. (2022)

49.Which of the following statements about acute rejection is NOT true?


a. It is T-cell mediated.
b. It is related to organ-host human leukocyte antigen disparity.
c. Treatment can save the grafted organ in 90% to 95% of cases.
d. It does not occur with living related donors.
e. It is associated with an increased risk of chronic rejection.

50.Which of the following statements is TRUE about Cushing's disease?


a. ACTH is high.
b. Blood sugar is low.
c. It is due to a pituitary adenoma in 10% of cases.
d. Serum Na is low.
e. Surgery is not successful in the treatment.

51.A 40-year-old male patient is complaining of persistent headache, recurrent


attacks of palpitation, sweating and visual disturbances. Blood pressure of the
patient is 180/110 mmHg. Which of the following is MOST accurate in the
diagnosis of the problem?
a. 24 hour urinary catecholamines.
b. 24 hour urinary VMA.
c. CT scan of the abdomen.
d. Plasma aldosterone level.
e. Plasma metanephrine level.

52.A 30-year-old pregnant female patient (first trimester) developed


palpitations, tremors, excessive sweating, nervousness and failure to gain
weight. Which of the following investigations is the MOST accurate to
diagnose her condition?
a. Free thyroxine level.
b. Level of thyroid peroxidase enzyme.
c. Technetium scan.
d. Thyroglobulin level.
e. Total thyroid level.
P a g e 13 | 26
SurgiNotes 2nd Ed. (2022)

53.What is the mode of action of carbimazole?


a. It diminishes the blood supply of the thyroid gland.
b. It prevents the release of thyroid hormone from the gland.
c. It prevents the union between iodine and tyrosine.
d. It prevents the transformation of iodides to iodine.
e. It prevents the uptake of iodides by the thyroid gland.

54.Which of the following is the MOST common sign of Cushing’s syndrome?


a. Acne.
b. Hirsutism.
c. Hypertension.
d. Purple striae.
e. Truncal obesity.

55.A 23-year-old woman undergoes total thyroidectomy for carcinoma of the


thyroid gland. On the second postoperative day, she begins to complain of
tingling sensation in her hands. She appears quite anxious and later complains
of muscle cramps. What is the initial therapy?
a. 10 ml of 10% magnesium sulfate intravenously.
b. 22-dihydrotachysterol orally.
c. Continuous infusion of calcium gluconate.
d. Oral calcium gluconate.
e. Oral vitamin D.

56.What is the best screening test to evaluate a patient to diagnose Cushing's


syndrome?
a. Fasting a.m. cortisol.
b. 24-hour urine cortisol.
c. Low-dose dexamethasone suppression test.
d. Serum adrenocorticotrophic hormones levels.
e. High-dose dexamethasone suppression test.
P a g e 14 | 26
SurgiNotes 2nd Ed. (2022)

57. Which of the following statements regarding adrenal cortical insufficiency is


TRUE?
a. Treatment with exogenous steroids is usually ineffective.
b. It is commonly seen as a consequence of metastasis of distant tumors, such
as lungs or breast, to the adrenal glands.
c. Chronic adrenal insufficiency (Addison's disease) in the preoperative patient
should be recognized by constellation of findings, including the
hyperglycemia, hypernatremia and hypokalemia.
d. Death from untreated chronic adrenal insufficiency may occur within hours
of surgery.
e. The most common underlying cause today is infection with resistant
microorganisms.
58. A 55-year-old female patient who has metastatic breast cancer presents with
weakness, anorexia, malaise, constipation and back pain and lethargy.
Laboratory studies include a normal chest X-ray, serum albumin 3.2 mg/dl, serum
calcium 14 mg/dl, serum phosphorus 2.6 mg/dl, serum chloride 108 mg/dl, BUN
32 mg/dl and creatinine 2.0 mg/dl. What is the appropriate initial management?
a. Intravenous normal saline infusion.
b. Administration of thiazide diuretics.
c. Administration of intravenous phosphorus.
d. Use of mithramycin.
e. Neck exploration and parathyroidectomy.
59. You have a patient who has pheochromocytoma and his blood pressure 200/120
mmHg. You are preparing this patient for surgery. Which of the following drugs
should be used first for the control of blood pressure?
a. Phenoxybenzamine.
b. Propranolol.
c. Nifedipine
d. Atenolol.
e. Captopril.
60. After undergoing a thyroidectomy operation, a 42-year-old opera singer has no
change in speech, but she has difficulty in singing high-pitched voices. Which
nerve is MOST likely to be injured?
a. Recurrent laryngeal.
b. Internal laryngeal.
c. External laryngeal.
d. Pharyngeal branch of vagus.
e. Phrenic.
P a g e 15 | 26
SurgiNotes 2nd Ed. (2022)

61.A 50-year-old woman presents with lethargy, weight gain, cold intolerance
and loss of interest for the past six months. Which is the MOST appropriate
initial investigation?
a. Erythrocyte sedimentation rate estimation.
b. Radioactive isotope scan of thyroid.
c. Thyroid antibodies screen.
d. Thyroid stimulating hormone estimation.
e. Ultrasound scan of thyroid gland.
62.40-year-old man is admitted to the surgical day case unit for repair of his left
inguinal hernia. On examination he is noted to have diffuse skin tanning,
spotty pigmentation of the elbows, nipples and buttocks and pigmentation of
the scar from a previous right inguinal hernia repair. Three hours after the
operation he becomes severely hypotensive. What is the MOST likely cause?
a. ACTH deficiency.
b. Adrenal insufficiency.
c. Growth hormone deficiency.
d. Potassium deficiency.
e. Thyroxin deficiency.
63.A pregnant mother in her first trimester comes to her clinician office with a
diagnosis of primary hyperparathyroidism. What is the correct management?
a. Parathyroidectomy during the second trimester.
b. Parathyroidectomy during the third trimester.
c. Prescribing a calcimimetic agent to help reduce hypercalcemia until after
delivery, when definitive surgery can be offered safely.
d. Close observation and parathyroidectomy following delivery.
e. Weekly injections of calcitonin until delivery, when definitive surgery can
be offered safely.
64.A patient who had adrenalectomy to remove a pheochromocytoma
demonstrates signs of confusion and complains of sweating and headache
several hours following his operation. His blood pressure is 130/65 mmHg, his
heart rate is 100 beats/min and his respiratory rate is 12 breaths/min. What is
the MOST likely cause of his symptoms?
a. Dehydration.
b. Postoperative bleeding.
c. Hypoglycemia.
d. Narcotic overdose.
e. Incomplete removal of the pheochromocytoma.

P a g e 16 | 26
SurgiNotes 2nd Ed. (2022)

65.With regard to calcium homeostasis, which of the following statements is


FALSE?
a. Calcium is the most abundant cation in human beings.
b. Approximately 50% of calcium is free or ionized and is metabolically
active.
c. Hypoalbuminemia can make the measured total calcium concentration
appear artificially low.
d. Hypoventilation can decrease ionized calcium levels and in turn
exacerbate symptoms of hypocalcemia.
e. Calcium is bound to citrate and is biologically inactive. 

66.Calcitonin helps mediate calcium homeostasis by which of the following


actions?
a. Stimulates osteoblast mediated bone formation and inhibits the renal
resorption of calcium and phosphate.
b. Directly inhibits the secretion of parathyroid hormones (PTH).
c. Inhibits intestinal absorptions of calcium.
d. Simulates hydroxylation of vitamin D.
e. Stimulates osteoclast mediated bone resorption.

67.What is the MOST common cause of primary adrenal insufficiency (Addison's


disease)?
a. Tuberculosis.
b. Kaposi sarcoma.
c. Cytomegalovirus.
d. Lymphoma.
e. Autoimmune disorder.

68.A 42-year-old woman is in the intensive care unit immediately following


removal of a left adrenal pheochromocytoma. Her blood pressure is 80/40
mmHg. The MOST appropriate treatment of the patient's hypotension is:
a. Epinephrine.
b. IV bolus of lactated Ringer solution.
c. Methylprednisolone.
d. Phenoxybenzamine.
e. Phenylephrine.
P a g e 17 | 26
SurgiNotes 2nd Ed. (2022)

69.A 75-year-old woman presents with acute back pain after a minor fall.
Radiological examination reveals an osteoporotic crush fracture. What is the
expected laboratory finding?
a. Hypercalcaemia.
b. Hyperphosphataemia.
c. Hypokalaemia.
d. Hyponatraemia.
e. Normocalcaemia.

70.Which one of the following clinical scenarios is associated with


hypercalcemia?
a. Fluid resuscitation of shock.
b. Rapid infusion of blood products.
c. Improper administration of phosphates.
d. Malignancy.
e. Acute pancreatitis.

71.Which of the following is the first sign or symptom of hypocalcemia?


a. Shortened QT interval.
b. Trousseau sign.
c. Circumoral numbness.
d. Anxiety.
e. Laryngospasm.

72.Which of the following occurs in starvation?


a. Increase in plasma glucose.
b. Decrease in urinary nitrogen excretion.
c. Increase in plasma ketone bodies.
d. Increase in glucose utilization by the brain.
e. Metabolic alkalosis.
P a g e 18 | 26
SurgiNotes 2nd Ed. (2022)

73.A complication that enteral and parenteral feeding have in common is:
a. Increased incidence of sepsis.
b. Intestinal villous atrophy.
c. Elevated liver transaminases.
d. Hyperosmolar non-ketotic coma.
e. Diarrhea.

74.A 50-year-old male diabetic patient whose diabetes has been controlled by
regimen alone is going to have repair of an incisional hernia. What is your plan
of management?
a. Continuation of diet and determination of serum glucose level before
surgery.
b. Subcutaneous administration of regular insulin.
c. Oral hypoglycemic agents for 3 days atter surgery.
d. Insulin infusion beginning one for before surgery.
e. Increased oral carbohydrate intake to prevent ketosis.

75.Which of the following with regard to metabolism during fasting is TRUE?


a. The main source of fuel in short-term fasting (< 5 days) is derived from
hepatic glycogen stores.
b. Norepinephrine, vasopressin and angiotensin II promotes the assembly of
glycogen during fasting.
c. In prolonged starvation, ketone bodies become the primary fuel for the
brain.
d. Lipid stores in adipose tissue provide 50% of the caloric expenditure
during starvation.
e. Release of fatty acids is stimulate by an increase in serum insulin levels.

76.Extracellular fluid has a:


a. Higher protein content than intracellular fluid.
b. Higher potassium content than intracellular fluid.
c. Higher sodium content than intracellular fluid.
d. Higher magnesium content than intracellular fluid.
e. Higher number of large organic molecules than intracellular fluid.

P a g e 19 | 26
SurgiNotes 2nd Ed. (2022)

77.A 35-year-old man is admitted to the ICU following a diagnosis of acute


pancreatitis. After initial resuscitation, the patient's condition improves and
enteral tube feedings are started through a postpyloric tube. Initial
intolerance to a tube feeding regimen requires the clinician to:
a. Immediately discontinue the tube feeding regimen and start total
parenteral nutrition (TPN).
b. Add water to feeding regimen to dilute the feedings for better tolerance.
c. Consider slowing the tube feeding regimen and progress to the goal rate
less aggressively.
d. Immediately change the tube feeding formula.
e. Increase the tube feeding rate per hour.

78.Hyperglycemia in a surgical patient receiving TPN may best be managed by:


a. Oral hypoglycemics.
b. Decreasing the dextrose load and doubling the amount of fat.
c. Adding regular insulin to the TPN.
d. Discontinuing TPN for 2 weeks and then trying to start TPN again.
e. Increasing the concentration of protein and carbohydrate calories and
decreasing that of lipids.

79.A 50-year-old man with small-bowel fistula has been receiving TPN for the
previous 3 weeks. He is scheduled for exploratory laparotomy and closure of
fistula. On the day of surgery, TPN is discontinued and intravenous infusion
with Ringer's lactate is started. An hour later, the patient was found to be
anxious, sweating and tachycardic. What is the MOST likely cause?
a. Anxiety.
b. Hypoglycemia.
c. Hypovolemia.
d. Unexplained hemorrhage.
e. Hyperglycemia.

80.In which of the following conditions is the enteral route appropriate for
nutrition?
a. Upper gastrointestinal bleed.
b. Complete small bowel obstruction.
c. Acute flare up of Crohn's disease.
d. Low output colonic fistula.
e. High output small bowel fistula.
P a g e 20 | 26
SurgiNotes 2nd Ed. (2022)

81.If a patient with pulmonary failure is receiving total parenteral nutrition.


Difficulty in weaning because of respiratory muscle weakness is MOST likely
to be associated with:
a. Hypocalcemia.
b. Hypochloremia.
c. Hyponatremia.
d. Hypomagnesemia.
e. Hypophosphatemia.

82.Which of the following conditions causes the greatest increase in energy


expenditure?
a. Pyloric obstruction from chronic duodenal ulcer.
b. Fractured femur.
c. Perforated diverticulitis of colon.
d. Thermal burns more than 30% total body surface area.
e. Right inguinal herniorrhaphy for incarcerated inguinal hernia.

83.A 75-year-old female presents with ductal carcinoma in situ for simple
mastectomy. She has marked muscle wasting on examination and admits to a
poor diet. Which of the following values is MOST predictive of postoperative
mortality?
a. Serum sodium.
b. Serum albumin.
c. Serum protein.
d. Serum creatinine.
e. Serum glucose.

84.A patient who presented with severe diarrhea as a result of clostridium


difficile, develops a metabolic acidosis. This is MOST likely due to loss of which
one of the following ions in the diarrhea?
a. Cl-.
b. HCO3-.
c. K+.
d. NH4+.
e. OH-.
P a g e 21 | 26
SurgiNotes 2nd Ed. (2022)

85.Bacterial pressure sores are best prevented by:


a. Routine use of air-flow mattresses.
b. Frequent re-positioning.
c. Aggressive nutritional support.
d. Infection control.
e. Drying powders to moist skin surfaces.

86.Which of the following statements is TRUE regarding beta-hemolytic


streptococci?
a. They produce a characteristic beta-lactamase.
b. They are one of the causes of necrotizing fasciitis.
c. They are the most common cause of infective endocarditis.
d. They are resistant to ampicillins.
e. They are part of the normal vaginal flora.

87.Which of the following statements regarding MRSA is TRUE?


a. The treatment of choice is clindamycin.
b. MRSA can only be found in the health care setting.
c. MRSA is more virulent than methicillin sensitive S. aureus.
d. Treatment of surgical patients with intranasal mupirocin decreases
wound infection rates with MRSA.
e. Hospitalized patients colonized with MRSA require contact isolation.

88.Which of the following statements regarding prevention of surgical site


infection is TRUE?
a. All hair should be removed by shaving from the operative site.
b. Smoking has no effect on postoperative surgical site infection.
c. Prophylactic antibiotics should be administered within 30 minutes of
incision.
d. Administration of 80% oxygen reduces surgical site infection in vascular
procedures.
e. Chlorhexidine containing skin preparations have a clear benefit in
reducing surgical site infection compared with povidone-iodine solutions.
P a g e 22 | 26
SurgiNotes 2nd Ed. (2022)

89.With regard to potassium, which of the following statements is NOT true?


a. Normal dietary intake of potassium is 50 to 100 mEq/day.
b. In patients with normal renal function, most ingested potassium is
excreted in urine.
c. More than 90% of the potassium in the body is located in the extracellular
compartment.
d. Critical hyperkalemia (> 6 mEq/L) is rarely encountered if renal function
is normal.
e. Administration of sodium bicarbonate shifts potassium from the
extracellular space (ECF) to the intracellular space (ICF).

90.Surgical safety checklist before induction of anesthesia includes all of the


following EXCEPT?
a. Does the patient have a known allergy?
b. Difficult airway or aspiration risk?
c. Risk of > 500 ml blood loss (7 ml/kg in children)?
d. Is the site of surgery shaved?
e. Is the site marked?

91.A 77-year-old man is admitted for a radical prostatectomy. On postoperative


day 3, a productive cough and fever develop. A chest radiograph shows a right
lower lobe infiltrate. Which of the following is correct regarding immune
function in older patients?
a. WBC counts increase significantly even with mild infections.
b. The T-cell response to new antigens is impaired.
c. Normal neutrophil counts decline with age.
d. Normal acute phase protein levels are decreased.
e. Normocytic anemia is uncommon in older patients.

92.Which site or venue is the area that is at greatest risk for surgical errors?
a. Operating room.
b. Surgical intensive care unit.
c. Hospital wards/floors.
d. Emergency department.
e. Ambulatory care sites.
P a g e 23 | 26
SurgiNotes 2nd Ed. (2022)

93.An unconscious accident victim is hypotensive from intra-abdominal


hemorrhage and needs an emergency laparotomy. His identity is unknown
and therefore, no family is available. Which of the following should be done?
a. Nothing, it is illegal to operate on a patient without consent.
b. The surgeon should document the need for the surgery in the chart and
proceed.
c. Three doctors should document the need for the surgery in the chart and
the surgeon should then proceed.
d. A court order for surgery should be obtained prior to proceeding.
e. Inform local health authority and then proceed.

94.The MOST common indication for surgery in the elderly is:


a. Obstructive vascular disease.
b. Thrombotic vascular disease.
c. Biliary tract disease.
d. Colorectal disease.
e. Pancreatic disease.

95.The nurse prepared a unit of blood to be transfused to a patient with lower


gastrointestinal bleeding. Just before transfusion she noticed that the blood
unit didn't belong to the patient and she sent it back to the blood bank and
received the right one. This situation is called:
a. Adverse event.
b. Near miss event.
c. Never event.
d. No harm event.
e. Sentinel event.

96.World health organization established the surgical safety checklist in order to


maintain patient safety in the operating theatre. One of the measures that
eliminate wrong site surgery is to perform a time-out check. The timing of this
check is:
a. Before referral to the operating theatre.
b. Before induction of anesthesia.
c. Before skin incision.
d. Before removal of an important organ.
e. Before closure of the wound.

P a g e 24 | 26
SurgiNotes 2nd Ed. (2022)

97. An 80-year-old woman with advanced Alzheimer's disease has been ill for 4 days
and is transferred from the nursing home with fever, hypotension and abdominal
swelling. Computed tomographic (CT) scan reveals a superior mesenteric artery
thrombosis, bowel ischemia and pneumatosis. She is acidotic and in acute renal
failure. Surgical intervention would necessitate resection of a significant length
of small and large bowel. Which of the following statements about this patient's
care is TRUE?
a. She is dying and palliative care is indicated.
b. The decision for further care should be left solely with the family.
c. Operation is the only chance for cure o the bowel ischemia and should be
performed.
d. Aggressive ICU resuscitation is warranted and operation should be
performed if the patient stabilizes.
e. Operation should be performed, but dialysis should not be contemplated.
98. Duty to intervene on the patient's behalf to increase comfort, health and well-
being:
a. Non-maleficence.
b. Autonomy.
c. Euthanasia.
d. Beneficence.
e. Justice.
99. Which of the following statements regarding the role of collagen in wound
healing is TRUE?
a. Collagen synthesis in the initial phase of injury is the sole responsibility of
endothelial cells.
b. Net collagen content increases for up to 2 years after injury.
c. At 3 weeks after injury, more than 50% of the tensile strength of the wound
has been restored.
d. Tensile strength of the wound increases gradually for up to 2-years after
injury; however, it generally reaches a level of only about 80% of that of
uninjured tissue
e. Tensile strength is the force necessary to reopen a wound.
100. Respiratory alkalosis can occur as a result of:
a. Asphyxia.
b. Asthma.
c. Severe emphysema.
d. Hyperventilation.
e. Hypoventilation.
P a g e 25 | 26
SurgiNotes 2nd Ed. (2022)

Egyptian Board Of General Surgery


1st Part Exam – Paper (2) MCQ "ANSWER"
October 2020

1 d 26 a 51 e 76 c
2 e 27 a 52 a 77 c
3 a 28 d 53 c 78 c
4 a 29 d 54 e 79 b
5 e 30 d 55 c 80 d
6 e 31 d 56 b 81 d
7 c 32 a 57 d 82 d
8 b 33 d 58 a 83 b
9 b 34 e 59 a 84 b
10 b 35 a 60 c 85 b
11 c 36 e 61 d 86 b
12 c 37 a 62 b 87 e
13 d 38 e 63 a 88 c
14 a 39 c 64 c 89 c
15 b 40 c 65 d 90 d
16 a 41 c 66 a 91 b
17 e 42 e 67 e 92 a
18 a 43 e 68 b 93 b
19 c 44 d 69 e 94 c
20 b 45 b 70 d 95 b
21 d 46 e 71 c 96 c
22 c 47 c 72 c 97 b
23 e 48 a 73 a 98 d
24 a 49 d 74 a 99 d
25 c 50 a 75 c 100 d

P a g e 26 | 26
SurgiNotes 2nd Ed. (2022)

Egyptian Board Of General Surgery


1st Part Exam – Paper (2) MCQ
March 2021
1. Which of the following statements is TRUE about Cushing's disease?
a. ACTH is high.
b. Blood sugar is low.
c. It is due to a pituitary adenoma in 10% of cases.
d. Serum Na is low.
e. Surgery is not successful in the treatment.
2. A 40-year-old male patient is complaining of persistent headache, recurrent
attacks of palpitation, sweating and visual disturbances. Blood pressure of the
patient is 180/110 mmHg. Which of the following is MOST accurate in the
diagnosis of the problem?
a. 24 hour urinary catecholamines.
b. 24 hour urinary VMA.
c. CT scan of the abdomen.
d. Plasma aldosterone level.
e. Plasma metanephrine level.
3. What is the hormone that acts on the intestines to increase calcium
absorption?
a. Calcitonin.
b. Corticotrophin releasing factor (CRF).
c. Pancreatic polypeptide.
d. Parathormone.
e. Thyroxine.
4. A 30-year-old female patient complains of fatigue, generalized bony aches and
depression. The patient gives a history of 2 previous operations for urinary
stones. Which of the following laboratory tests is MOST accurate in the
diagnosis?
a. 24 hours urinary calcium.
b. Ionizable serum calcium.
c. Parathormone hormone.
d. Plasma chloride.
e. Serum phosphate.
P a g e 1 | 26
SurgiNotes 2nd Ed. (2022)

5. Regarding the anatomy of the thyroid gland, which of the following TRUE?
a. In about 80% of persons, the recurrent laryngeal nerve traverses anterior
to the inferior thyroid artery.
b. The recurrent laryngeal nerve has an oblique course around the
subclavian artery on the left side.
c. The superior laryngeal nerve provides both sensory and motor function
to the larynx.
d. The thyroid gland is innervated only by parasympathetic fibers from the
vagus nerve.
e. Unilateral recurrent laryngeal nerve injury usually results in airway
compromise that necessitates tracheotomy.

6. Which of the following tests will be helpful in the early management of a


patient who had total thyroidectomy?
a. Antithyroglobulin antibody.
b. Serum calcium.
c. Serum iodine.
d. T4.
e. TSH.

7. A 23-year-old woman undergoes total thyroidectomy for carcinoma of the


thyroid gland. On the second postoperative day, she begins to complain of
tingling sensation in her hands. She appears quite anxious and later complains
of muscle cramps. What is the initial therapy?
a. 10 ml of 10% magnesium sulfate intravenously.
b. 22-dihydrotachysterol orally.
c. Continuous infusion of calcium gluconate.
d. Oral calcium gluconate.
e. Oral vitamin D.

8. You have a patient who has pheochromocytoma and his blood pressure is
200/120 mmHg. You are preparing this patient for surgery. Which of the
following drugs should be used first for the control of blood pressure?
a. Phenoxybenzamine.
b. Propranolol.
c. Nifedipine.
d. Atenolol.
e. Captopril.
P a g e 2 | 26
SurgiNotes 2nd Ed. (2022)

9. A 45-year-old female is found to have a 2-cm solid nodule in her right adrenal
gland at the time of an abdominal CT scan following an auto-accident. With
regard to the adrenal lesion, the patient is asymptomatic and the nodule is
found to be non-functional on evaluation. What is the recommended
management?
a. Extraperitoneal right adrenalectomy through either a flank of posterior
approach.
b. Suppression with 5 mg prednisone PO t.d.s.
c. Follow-up CT scan after 3 months.
d. Excision biopsy via laparoscopic approach.
e. Arterial embolization.

10.What is the recommended treatment of acute adrenal insufficiency?


a. Normal saline, potassium and glucose.
b. Hypertonic saline and potassium.
c. Normal saline and potassium.
d. Intravenous mineralocorticoids.
e. Normal saline, glucose and intravenous glucocorticoids.

11.A 40-year-old female patient complains of tiredness, putting on weight,


intolerance to cold weather and hoarseness of voice. Investigations revealed
low levels of T3, T4 and TSH. What is the diagnosis?
a. Hashimoto's thyroiditis.
b. Previous thyroidectomy.
c. Pituitary hypothyroidism.
d. Primary hypothyroidism.
e. Hypothyroidisms following 131I intake.

12.A 50-year-old woman presents with lethargy, weight gain, cold intolerance
and loss of interest for the past six months. Which is the MOST appropriate
initial investigation?
a. Erythrocyte sedimentation rate estimation.
b. Radioactive isotope scan of thyroid.
c. Thyroid antibodies screen.
d. Thyroid stimulating hormone estimation.
e. Ultrasound scan of thyroid gland.

P a g e 3 | 26
SurgiNotes 2nd Ed. (2022)

13.A patient with chronic renal failure attends the emergency department
complaining of increasing confusion, muscle weakness, nausea, vomiting and
fatigue. The serum calcium level is 12.4 mg/dl. The first step in management
of this patient should be:
a. Emergency parathyroidectomy.
b. Aggressive intravenous hydration.
c. Initiation of furosemide infusion.
d. Continuous calcitonin infusion.
e. Initiation of bisphosphonates.

14.What is the MOST common cause of primary adrenal insufficiency (Addison's


disease)?
a. Tuberculosis.
b. Kaposi sarcoma.
c. Cytomegalovirus.
d. Lymphoma.
e. Autoimmune disorder.

15.A patient who had adrenalectomy to remove a pheochromocytoma


demonstrates signs of confusion and complains of sweating and headache
several hours following his operation. His blood pressure is 130/65 mmHg, his
heart rate is 100 beats/min and his respiratory rate is 12 breaths/min. What is
the MOST likely cause of his symptoms?
a. Dehydration.
b. Postoperative bleeding.
c. Hypoglycemia.
d. Narcotic overdose.
e. Incomplete removal of the pheochromocytoma.

16.A 42-year-old woman is in the intensive care unit immediately following


removal of a left adrenal pheochromocytoma. Her blood pressure is 80/40
mmHg. The MOST appropriate treatment of the patient's hypotension is:
a. Epinephrine.
b. IV bolus of lactated Ringer solution.
c. Methylprednisolone.
d. Phenoxybenzamine.
e. Phenylephrine.

P a g e 4 | 26
SurgiNotes 2nd Ed. (2022)

17.A 50-year-old woman scheduled for parathyroidectomy presents to the


emergency department with nausea, vomiting and altered mental status.
Laboratory analysis reveals serum calcium of 16 mg/dl. In addition to IV fluids,
the best medication to give immediately would be:
a. Calcitonin.
b. Furosemide.
c. Mithramycin.
d. Prednisone.
e. Zoledronic acid.
18.An otherwise healthy 52-years-old male is in the postoperative care unit,
hours after undergoing an uncomplicated left-sided laparoscopic
adrenalectomy for a 0.5-cm cortisol-producing tumor when he is noted to
have a heart rate of 82 beats per minute and blood pressure of 90/40 mmHg.
After administration of 2 L of normal saline, the blood pressure is unchanged.
The MOST critical next step in management will be to:
a. Administer 100 mg IV hydrocortisone.
b. Begin a norepinephrine drip.
c. Check troponin levels.
d. Obtain a 12 lead EKG.
e. Obtain a portable chest X-ray.
19.A 75-year-old woman presents with acute back pain after a minor fall.
Radiological examination reveals an osteoporotic crush fracture. What is the
expected laboratory finding?
a. Hypercalcaemia.
b. Hyperphosphataemia.
c. Hypokalaemia.
d. Hyponatraemia.
e. Normocalcaemia.
20.Calcitonin helps mediate calcium homeostasis by which of the following
actions?
a. Stimulates osteoblast-mediated bone formation and inhibits renal
resorption of calcium and phosphate.
b. Directly inhibits secretion of parathyroid hormone (PTH).
c. Inhibits intestinal absorption of calcium.
d. Stimulates hydroxylation of vitamin D.
e. Stimulates osteoclast-mediated bone resorption.
P a g e 5 | 26
SurgiNotes 2nd Ed. (2022)

21.In addition to determining blood glucose levels, the perioperative


management of a patient with diet-controlled diabetes mellitus should
include which of the following considerations?
a. Determination of glycosylated hemoglobin (HBA1c) level before surgery.
b. Subcutaneous administration of regular insulin for glucose levels above
140 mg%.
c. Metformin initiated 3 days prior surgery.
d. Intravenous (IV) insulin therapy 1 h prior to surgery.
e. Oral liquid carbohydrate initiated 3 h prior to surgery.

22.Which one of the following clinical scenarios is associated with


hypercalcemia?
a. Fluid resuscitation for shock.
b. Rapid infusion of blood products.
c. Improper administration of phosphates.
d. Malignancy.
e. Acute pancreatitis.

23.Which of the following is the first sign or symptom of hypocalcemia?


a. Shortened QT interval.
b. Trousseau sign.
c. Circumoral numbness.
d. Anxiety.
e. Laryngospasm.

24.On workup, a patient is found to have elevated free cortisol and plasma ACTH
levels. Further testing reveals that both low- and high-dose dexamethasone
administration fail to suppress cortisol production. What is the MOST likely
diagnosis?
a. Bilateral adrenal hyperplasia.
b. Pituitary tumor.
c. Adrenal adenoma.
d. Ectopic ACTH-producing tumor.
e. Exogenous corticosteroids.

P a g e 6 | 26
SurgiNotes 2nd Ed. (2022)

25.Which of the following cancers is MOST common in organ transplant


recipients?
a. Bronchogenic carcinoma.
b. Lung cancer.
c. Pancreatic cancer.
d. Prostatic cancer.
e. Skin cancer.

26.Antilymphocytic globulin is useful to treat:


a. Acute rejection.
b. Chronic rejection.
c. Graft-versus-host reaction.
d. Hyperacute rejection.
e. Immunosuppressive drug toxicity.

27. A 40-year-old male patient underwent a renal transplant from a cadaveric


donor. Immediately after finishing the vascular anastomosis, the kidney
became cyanotic and flaccid. Histological examination revealed deposition of
immunoglobulins and complement in the vessels walls. What is the
immunological background of the problem?
a. Donor cytotoxic T-lymphocytes directed against the host antigens.
b. Donor natural killer cells directed against host antigens.
c. Host natural killer cells against donor antigens.
d. Preformed donor antibodies against the host antigens.
e. Preformed host antibodies against the donor antigens.

28.Which of the following allogeneic grafts does NOT require


immunosuppression?
a. Kidney.
b. Hearn.
c. Liver.
d. Bone marrow.
e. Cartilage.
P a g e 7 | 26
SurgiNotes 2nd Ed. (2022)

29.Which of the following is the MOST commonly used primary maintenance


immunosuppressive agent?
a. Tacrolimus (prograf).
b. Azathioprine (imuran).
c. Mycophenolate (cellcept).
d. Rapamycin (sirolimus).
e. Pirfenidone.

30.Which of the following statements regarding rejection of solid organ


transplants is TRUE?
a. Hyper-acute rejection begins in the operating room with reperfusion of
the transplanted organ.
b. Liver transplants are especially susceptible to hyper-acute rejection.
c. Most immunosuppressive medications are used to prevent chronic
rejection.
d. The major cause of graft failure is acute rejection.
e. Chronic rejection is characterized histologically by lymphocyte
infiltration.

31.Which of the following is NOT an indication for liver transplant?


a. Cirrhosis complicated by ascites and esophageal varices.
b. Acute liver failure secondary to acetaminophen overdose.
c. NASH (nonalcoholic steatohepatitis) complicated by hepatic
encephalopathy.
d. Hepatocellular carcinoma with underlying cirrhosis.
e. Hepatocellular carcinoma with biopsy-proven lymph node metastasis.

32.Which of the following statements about acute rejection is NOT true?


a. It is T-cell mediated.
b. It is related to organ-host human leukocyte antigen disparity.
c. Treatment can save the grafted organ in 90% to 95% of cases.
d. It does not occur with living related donors.
e. It is associated with an increased risk of chronic rejection.
P a g e 8 | 26
SurgiNotes 2nd Ed. (2022)

33. Which of the following transplants is MOST susceptible to donor-recipient HLA


mismatches?
a. Skin.
b. Renal.
c. Liver.
d. Corneal.
e. Cardiac valves.

34. You review a 42-year-old woman six weeks following a renal transplant for focal
segmental glomerulosclerosis. Following the procedure she discharged on a
combination of tacrolimus, mycophenolate and prednisolone. She has now
presented with a five day history of feeling generally unwell with anorexia,
fatigue and arthralgia. On examination she has a temperature of 37.9 oC and has
widespread lymphadenopathy. What is the MOST likely diagnosis?
a. Hepatitis C.
b. Epstein-Barr virus.
c. HIV.
d. Hepatitis B.
e. Cytomegalovirus.

35. Five days after an uneventful cholecystectomy, an asymptomatic middle aged


woman is found to have a serum sodium level of 120 mEq/L. What is the proper
management?
a. Administration of hypertonic saline solution.
b. Administration of Ringer's lactate solution.
c. Aggressive diuresis with furosemide.
d. Plasma ultra-filtration.
e. Restriction of free water.

36. A 28-year-old male was injured In a motorcycle accident. On admission he was in


severe respiratory distress and appeared cyanotic (With blood pressure of 80/40
mmHg). The patient was bleeding profusely from the nose and had an obviously
open femur fracture with exposed bone. Breath sounds were decreased on the
right side of the chest. What should be the initial management?
a. Control of hemorrhage with anterior and posterior nasal packing.
b. Tube thoracostomy in the right hemithorax.
c. Endotracheal intubation with in-line cervical traction.
d. Obtain intravenous access and begin emergency blood transfusion.
e. Obtain cross-table cervical spine film and chest film.
P a g e 9 | 26
SurgiNotes 2nd Ed. (2022)

37.A 24-year-old woman has acute renal failure following postpartum


hemorrhage. Laboratory studies showed serum glucose 150 mg/dl, sodium
135 mEq/L, potassium 6.5 mEq/L, chloride 105 mEq/L and bicarbonate 15
mEq/L. Which of the following is recommended?
a. Decrease potassium chloride to 10 mEq/L.
b. Intravenous 0.9% sodium chloride.
c. 100 ml of 50% glucose water with 10 U insulin.
d. Intravenous calcitonin.
e. Intravenous magnesium sulfate.

38.Following an operation and extubation, a patient was restless. His arterial


blood gases revealed: pH 7.36, PaO2 65 mmHg, PaCO2 55 mmHg and PaCO3- 38
mmol/L. The physiologic status can best be described as which of the
following?
a. Respiratory alkalosis.
b. Respiratory acidosis.
c. Metabolic acidosis.
d. Metabolic alkalosis.
e. Combined respiratory and metabolic acidosis.

39.What is the predominant metabolic abnormality in patients with pyloric


obstruction?
a. Respiratory alkalosis.
b. Hyperchloremic alkalosis.
c. Salt-losing enteropathy.
d. Intrinsic renal disease.
e. Metabolic acidosis.

40.A primary tumor of which of these organs is the LEAST likely to metastasize to
bone:
a. Breast.
b. Colon.
c. Kidney.
d. Lung.
e. Prostate.
P a g e 10 | 26
SurgiNotes 2nd Ed. (2022)

41.A patient who had gastrectomy 10 days ago developed massive pulmonary
embolism proved by CT angiography. What is the recommended treatment?
a. IV heparin.
b. Fibrinolytic agents.
c. Warfarin.
d. IV acetylsalicylic acid.
e. Corticosteroids.

42.A 28-year-old motorcyclist sustained multiple fractures and a


multifragmented fracture of his left femoral shaft. He underwent reamed
intramedullary nailing the next day. Twenty-four hours postoperatively, he
became increasingly confused and hypoxaemic. The MOST likely diagnosis is:
a. Fat embolism.
b. Hypovolemia.
c. Systemic sepsis.
d. Tension pneumothorax.
e. Unrecognized head injury.

43.A 39-year-old woman is making a slow but adequate recovery after sustaining
a 40% surface area burn injury. On the sixth postoperative day she becomes
unwell. She vomits intermittently, has painless abdominal distension and
starts to hiccup. What is the MOST likely cause of these symptoms?
a. Acute gastric dilatation.
b. Acute intestinal obstruction.
c. Clostridium difficile infection.
d. Fecal impaction.
e. Systemic sepsis.

44.The arterial blood gases of a patient who had an operation for ruptured aortic
aneurysm are: pH 7.54, PO2 100 mmHg, PCO2 30 mmHg and HCO3- 30 mmol/L.
What is the acid-base disturbance?
a. Respiratory acidosis.
b. Respiratory alkalosis.
c. Metabolic alkalosis.
d. Combined respiratory and metabolic alkalosis.
e. Compensated respiratory acidosis.

P a g e 11 | 26
SurgiNotes 2nd Ed. (2022)

45.A 78-year-old man with history of coronary artery disease requests an elective
epigastric hernia repair. Which of the following is valid reason for delaying the
proposed surgery?
a. Coronary artery bypass surgery 3 months earlier.
b. A history of cigarette smoking.
c. Jugular venous distension.
d. Hypertension.
e. Hyperlipidemia.

46.A 50-year-old man presented with severe repeated vomiting as a result of


gastric outlet obstruction is admitted to the hospital. There is marked
dehydration, with urine output 20 ml/h and the hematocrit is 48%. Initial
treatment for this patient should include which of the following?
a. Administration of 10% dextrose (D10W) in one-third saline solution IV.
b. Antiemetics.
c. Ringer's lactate solution.
d. Hemodialysis to correct azotemia.
e. Saline fluid replacement with appropriate potassium administration.

47.Which of the following types of shock is associated with high pulmonary


wedge pressure?
a. Hypovolemic shock.
b. Cardiogenic shock.
c. Early septic shock .
d. Late septic shock.
e. Neurogenic shock.

48.A common drug that can trigger an episode of malignant hyperthermia:


a. Propofol.
b. Dantrolene.
c. Ketamine.
d. Succinylcholine.
e. Neostigmine.
P a g e 12 | 26
SurgiNotes 2nd Ed. (2022)

49.What is the correct management of the commonest acid-base imbalance seen


in long standing or severe hemorrhagic shock?
a. Intravenous sodium bicarbonate.
b. Component blood therapy.
c. Increased fluid administration.
d. Vasopressors.
e. Hyperventilation adenoma.
50. A 65-year-old man undergoes trans-sphenoidal surgery for a pituitary macro-
adenoma. On the first postoperative day he is noted to be confused. The
MOST likely cause is:
a. Hyperkalemia.
b. Hyperuricemia.
c. Hypoglycemia.
d. Hyponatremia.
e. Hypoxia.
51.A 38-year-old man in end-stage renal failure resulting from polycystic kidney
disease receives a cadaveric renal transplant. Good renal function is
established but four weeks later deteriorates, the serum creatinine rising by
25%. Which of the following processes is MOST likely to be responsible for this
deterioration?
a. B-cell mediated rejection.
b. Circulating immune complex disease.
c. IgG antibody mediated rejection.
d. Post-transplant lymphoproliferative disorder.
e. T-cell mediated rejection.
52.A 78-year-old woman with emphysema receiving 28% oxygen by mask has the
following blood gas results:
PH PO2 PaCO2 Bicarbonate Base excess
Finding: 7.28 70 mmHg 48 mmHg 36 mmol/L +5
Normal: 7.35-7.45 90-110 35-45 22-26 -2 to +2
The MOST likely interpretation is:
a. Mixed respiratory and metabolic acidosis.
b. Partially compensated metabolic acidosis.
c. Partially compensated respiratory acidosis.
d. Uncompensated metabolic acidosis.
e. Uncompensated respiratory acidosis.
P a g e 13 | 26
SurgiNotes 2nd Ed. (2022)

53.A 56-year-old motorcyclist presents to the emergency department after he


had involved in a road traffic accident. He is conscious and is maintaining his
normal airway and breathing. He is found to have an open right femoral
fracture with palpable distal pulses and normal sensation. Resuscitation is
started. Urethral catheterization drained 250 ml immediately but over the
following 60 minutes he drained only 10 ml. Which of the following is the
MOST likely cause of his low urine output?
a. Blocked catheter.
b. Cardiogenic shock.
c. Hypovolemic shock.
d. Ruptured bladder.
e. Septic shock.
54.A 55-year-old man presents to the emergency department after collapsing. On
examination his pulse is 124 beats/minute, blood pressure is 60/30 mmHg,
respiratory rate is 30 breaths/minute and his peripheries are warm. What is
the MOST likely diagnosis for this patient?
a. Cardiac failure.
b. Hemorrhage.
c. Hypovolemia.
d. Pulmonary embolus.
e. Sepsis.
55.A 22-year-old man involved in a motor vehicle accident is found to have a
thoracic spine fracture (T6) and paraplegia. The patient is hypotensive with a
systolic BP of 70 mmHg, is bradycardiac with a pulse of 48 beats/min and is
breathing comfortably. Which of the following would the MOST appropriate
initial treatment?
a. Isotonic fluid administration.
b. Steroid administration within 24 hours of the injury.
c. Immediate intubation.
d. Alpha-agonist administration.
e. Immediate magnetic resonance imaging.
56.The pathogenesis of stress-related gastric mucosal injury is:
a. Elevated gastric acid levels.
b. Bile reflux.
c. Reduced gastric blood flow.
d. Helicobacter pylori infection.
e. Use of nonsteroidal anti-inflammatory drugs.

P a g e 14 | 26
SurgiNotes 2nd Ed. (2022)

57.A 55-year-old male patient has been receiving TPN for prolonged time and
developed hypomagnesaemia. Which of the following conditions clinically
resembles hypomagnesaemia?
a. Hypoglycemia.
b. Hypokalemia.
c. Hypophosphatemia.
d. Hypocalcemia.
e. Hyponatremia.

58.Which of the following are effects of epinephrine in response to injury?


a. It enhances the adherence of leukocytes to vascular endothelial
membranes.
b. It stimulates the release of aldosterone.
c. It inhibits the secretion of thyroid hormones.
d. It increases glucagon secretion.
e. It decreases lipolysis in adipose tissues.

59.A 21-year-ald man undergoes major abdominal surgery after a motor vehicle
collision. He has a cardiac arrest in the intensive care unit shortly after
returning from surgery. Select the MOST appropriate pharmacologic agent for
the patient:
a. Epinephrine.
b. Norepinephrine.
c. Phenylephrine.
d. Dopamine.
e. Dobutamine.

60.Which of the following is the best parameter for monitoring septic shock?
a. Central venous pressure (CVP).
b. Vasopressor requirement.
c. Urine output.
d. Serum lactate.
e. Mental status changes.
P a g e 15 | 26
SurgiNotes 2nd Ed. (2022)

61.Which of the following is an important goal of therapy at 24 hours after the


onset of hypovolemic shock that may improve mortality?
a. Normalization of blood pressure and heart rate.
b. Normalization of urine output and base deficit.
c. Discontinuation of vasopressors and inotropes.
d. Ensuring the hematocrit is at least 30% (0.3).
e. Normalization of body and skin temperature.

62.Which of the following is TRUE about the catabolic response to trauma?


a. Intravenous hyperalimentation can prevent the catabolic response to
trauma.
b. Liver glycogen is the source of dextrose in the first week.
c. The catabolic response initiated by the thyroid hormones.
d. The catabolic response is the same regardless of the severity of trauma.
e. There is inevitable loss of muscle mass.

63.Effective management of gastric acid aspiration includes which of the


following?
a. Suctioning and controlled ventilation with positive end-expiratory
pressure (PEEP).
b. Tracheal intubation and saline lavage of the lungs.
c. Prophylactic antibiotic therapy.
d. Prophylactic steroid therapy.
e. Diuresis.

64.The arterial blood gas results of a 40-year-old male patient who had an
accident one week ago are: pH 7.54, PO2 80 mmHg, PCO2 30 mmHg and PCO3-
18 mmol/L. The MOST likely cause is:
a. Excessive vomiting.
b. Pulmonary embolism.
c. Diabetic ketoacidosis.
d. CVA.
e. Persistent diarrhea.
P a g e 16 | 26
SurgiNotes 2nd Ed. (2022)

65.Which of the following metabolic changes is MOST prominent during times of


physiologic stress?
a. Increase in growth hormone (GH) release.
b. Increase in TSH.
c. Increased levels of T4 and T3.
d. Initial insulin increase and then suppression.
e. Increase in cortisol excretion.

66.Which of the following options is MOST appropriate for a type II diabetic


patient undergoing bilateral hernia repair as a day-case procedure under
general anesthesia?
a. Their usual diabetic medication should be omitted on the night before
surgery.
b. They should be prescribed an insulin/dextrose sliding scale starting at 6
a.m. on the day of surgery.
c. They should be placed first on the operating list whenever possible.
d. They should be kept nil by mouth at least 6 hours before the procedure.
e. They should receive long acting insulin at the night of the operation.

67.Regarding hypokalemia, which of the following statements is NOT true?


a. Potassium and hydrogen are exchanged for sodium in the renal tubule.
b. Respiratory acidosis is associated with increased renal potassium loss.
c. Metabolic acidosis is usually associated with hypokalemia.
d. Flattened T wave and a prolonged QT interval are associated with
hypokalemia.
e. Intravenous potassium administration should not exceed 40 to 60 mEq/h.

68.A 70-year-old man undergoes anterior resection for carcinoma of the rectum.
He is extubated in the operating room (OR). In the recovery room, he is found
to be restless with an HR of 136 bpm and a BP of 144/80 mmHg. ABG analysis
on room air reveals pH 7.24, PCO2 60 mmHg, PO2 54 mmHg, HCO3- 25 mEq/L
and SaO2 90%. Appropriate management for this patient should be which of
the following?
a. To administer 40% oxygen by mask.
b. Morphine 2 mg IV.
c. Ringer's lactate 250 mL over 1 hour.
d. Intubation and ventilatory support.
e. Deep breathing and coughing.
P a g e 17 | 26
SurgiNotes 2nd Ed. (2022)

69. Clinical studies have shown that administration of lactated Ringer's solution
to patients with hypovolemic shock may:
a. A Increase serum lactate concentration.
b. Impair liver function.
c. Improve hemodynamics by alleviating the deficit in the interstitial fluid
compartment.
d. It increases metabolic acidosis.
e. Increase the need for blood transfusion.

70.Central venous access lines is a minor procedure, but may associated with
many complications, the commonest is:
a. Haematoma.
b. Air embolism.
c. Pneumothorax.
d. Arrhythmias.
e. Lost wire or catheter.

71. The gold standard for diagnosing pulmonary embolism (PE) is:
a. CT pulmonary angiogram.
b. MRI.
c. Ventilation perfusion nuclear scan (VQ scan).
d. Duplex ultrasound.
e. ECG.

72.What is the MOST common tumor that metastasizes to the adrenal gland?
a. Breast cancer.
b. Bronchogenic cancer.
c. Melanoma.
d. Renal cell carcinoma.
e. Thyroid cancer.
P a g e 18 | 26
SurgiNotes 2nd Ed. (2022)

73.Which of the following the commonest cause of metastatic hypercalcaemia?


a. Breast cancer.
b. Prostatic cancer.
c. Lung cancer.
d. Testicular tumors.
e. Thyroid malignancy.

74.Which of the following is LEAST likely to cause metastasis to the brain?


a. Prostate cancer.
b. Melanoma.
c. Small cell carcinoma of the lung.
d. Breast cancer.
e. Renal carcinoma.

75.A 40-year-old man presents to his doctor with a swelling in the neck of eight
weeks' duration. Clinical examination demonstrates that he has multiple
swellings in the neck, axillae and both inguinal regions. What is the MOST
likely diagnosis?
a. Acute lymphadenitis.
b. Cat-scratch disease.
c. Malignant lymphoma.
d. Metastatic carcinoma.
e. Reactive Lymphadenopathy.

76.Which of the following is NOT characteristic of poor differentiation?


a. Pleomorphism (variability in the size and shape of cells).
b. Giant cells.
c. Decreased mitotic activity.
d. High nuclear : cytoplasmic ratio.
e. Clumped chromatin.
P a g e 19 | 26
SurgiNotes 2nd Ed. (2022)

77.Which of the following tumor marker-disease association is NOT correct?


a. PSA and prostate cancer.
b. CEA and colon cancer.
c. CA 19-9 and pancreatic cancer.
d. AFP and breast cancer.
e. CA 125 and ovarian cancer.

78.A patient undergoing chemotherapy for acute leukemia develops bilateral


ureteric stones. What is the MOST likely composition of these stones?
a. Calcium oxalate.
b. Xanthine.
c. Cysteine.
d. Triple phosphate.
e. Uric acid.

79.A 43-year-old woman is undergoing external beam radiation therapy for


invasive breast cancer. Biopsy of the tumor shows a relatively high mitotic
index, indicative of active growth. Which portion of the cell cycle in actively
dividing cells is MOST sensitive to ionizing radiation?
a. S phase.
b. M phase.
c. G1 phase.
d. G2 phase.
e. All phases are equally radiosensitive.

80.Which of the following malignancies is the commonest to produce metastases


to the spine?
a. Bronchogenic carcinoma.
b. Breast cancer.
c. Prostatic cancer.
d. Renal cell carcinoma.
e. Colon cancer.
P a g e 20 | 26
SurgiNotes 2nd Ed. (2022)

81.In examining a biopsy of a primary tumor, the clearest evidence of malignancy


is provided by:
a. Absence of a capsule.
b. Basophilia of the cytoplasm.
c. Invasion of surrounding structures.
d. Excess of mitoses.
e. Nuclear aberrations.

82.A 56-year-old lady presents with a pathological fracture of the proximal femur.
Which of the following primary sites is the MOST likely source of her disease?
a. Thyroid.
b. Breast.
c. Kidney.
d. Endometrium.
e. Colon.

83.Which of these options is the MOST common cause of death in patients with
ovarian cancer?
a. Uremia.
b. Anemia.
c. Liver failure.
d. Bowel obstruction.
e. Respiratory failure.

84.Which of the following genes is a protooncogene?


a. BRCA-I.
b. DCC.
c. RAS.
d. APC.
e. P53.
P a g e 21 | 26
SurgiNotes 2nd Ed. (2022)

85.Which of the following occurs in starvation?


a. Increase in plasma glucose.
b. Decrease in urinary nitrogen excretion.
c. Increase in plasma ketone bodies.
d. Increase in glucose utilization by the brain.
e. Metabolic alkalosis.

86.The perioperative management of a patient whose diabetes has been


controlled by diet alone consists of blood glucose level determinations and
which of the following?
a. Continuation of diet and determination of serum glucose level before
surgery.
b. Subcutaneous administration of regular insulin.
c. Oral hypoglycemic agents discontinued 3 days before surgery.
d. Insulin infusion beginning one hr before surgery.
e. Increased oral carbohydrate intake to prevent ketosis.

87.What is the MOST deleterious effect of excessive glucose in the formula of


longstanding TPN?
a. Hyperglycemia.
b. Excessive diuresis.
c. Hepatic steatosis.
d. Hyponatremia.
e. Hypoglycemia.

88.Which of the following with regard to metabolism during fasting is TRUE?


a. The main source of fuel in short-term fasting (< 5 days) is derived from
hepatic glycogen stores.
b. Norepinephrine, vasopressin, and angiotensin II promote the assembly of
glycogen chains during fasting.
c. In prolonged starvation, ketone bodies become the primary fuel source
for the brain.
d. Lipid stores in adipose tissue provide 40% of the caloric expenditure
during starvation.
e. Release of tree fatty acids is stimulated by an increase in serum insulin
levels.
P a g e 22 | 26
SurgiNotes 2nd Ed. (2022)

89.The gold standard for diagnosing pulmonary embolism (PE) is:


a. CT pulmonary angiogram.
b. MRI.
c. Ventilation perfusion nuclear scan (VQ scan).
d. Duplex ultrasound.
e. ECG.

90.The chest X-ray of a 68-year-old intubated man in the surgical intensive care
unit reveals an infiltrate in his right lower lobe. Concurrent with this finding,
the patient has an elevated leukocyte count and increased endotracheal
secretions. Which of the following procedures would have been MOST
effective in preventing ventilator-associated pneumonia (VAP)?
a. Elevation of the head of bed above 30 degrees.
b. Initiation of ranitidine.
c. Replacement of nasogastric tube with nasoduodenal tube for post pyloric
enteral feeding.
d. Selective decontamination of the digestive tract.
e. Use of prophylactic topical antibiotics (Intratracheal or oral).

91.What is the single MOST important treatment measure in preventing acute


renal failure following crush injury?
a. Maintenance of an alkaline urine (pH > 6) to prevent cast formation.
b. Maintenance of adequate urine output with mannitol.
c. Vigorous intravenous fluid replacement.
d. Administration of xanthine oxidase inhibitors to prevent hyperuricemia.
e. Emergent dialysis to remove myoglobin from the circulation.

92.A 35-year-old male patient was subjected to a car accident and had multiple
rib fractures and fracture of the right shaft femur. The pulse is 120/min and
the BP is 90/60 mmHg. The patient is in severe agony. What is the MOST
appropriate parenteral analgesic for this patient?
a. Morphine.
b. Fentanyl.
c. Midazolam.
d. Paracetamol.
e. Propofol.
P a g e 23 | 26
SurgiNotes 2nd Ed. (2022)

93.Which site or venue is the area that is at greatest risk for surgical errors?
a. Operating room.
b. Surgical intensive care unit.
c. Hospital wards/floors.
d. Emergency department.
e. Ambulatory care sites.
94.An unconscious accident victim is hypotensive from intra-abdominal
hemorrhage and needs an emergency laparotomy. His identity is unknown
and therefore, no family is available. Which of the following should be done?
a. Nothing, it is illegal to operate on a patient without consent.
b. The surgeon should document the need for the surgery in the chart and
proceed.
c. Three doctors should document the need for the surgery in the chart and
the surgeon should then proceed.
d. A court order for surgery should be obtained prior to proceeding.
e. Inform local health authority and then proceed.
95.Surgical "never events" are errors in medical care that are clearly identifiable,
preventable and serious in their consequences for patients and that indicate a
real problem in the safety and credibility of a health care facility. Which
surgical error does NOT belong to this category?
a. Surgery performed on the wrong body part.
b. Surgery performed on the wrong patient.
c. Immediately postoperative death in an ASA class II patient.
d. Wrong surgical procedure performed on a patient.
e. A foreign body left in a patient after surgery.
96.Before skin incision "time out" is a phase of the surgical safety check list. What
is the correct step that should be done at this phase?
a. The Checklist coordinator will verbally review with the patient his
identity.
b. The Checklist coordinator will verbally review with the patient that the
procedure and site are correct.
c. The Checklist coordinator will verbally review with the patient that
consent for surgery has been given.
d. The team will confirm that prophylactic antibiotics have been
administered within the previous 60 minutes.
e. The team will review together the operation that was performed,
completion of sponge and instrument counts.

P a g e 24 | 26
SurgiNotes 2nd Ed. (2022)

97.The nurse prepared a unit of blood to be transfused to a patient with lower


gastrointestinal bleeding. Just before transfusion she noticed that the blood
unit didn't belong to the patient and she sent it back to the blood bank and
received the right one. This situation is called:
a. Adverse event.
b. Near miss event.
c. Never event.
d. No harm event.
e. Sentinel event.
98.An 80-year-old woman with advanced Alzheimer's disease has been ill for 4
days and is transferred from the nursing home with fever, hypotension and
abdominal swelling. Computed tomographic (CT) scan reveals a superior
mesenteric artery thrombosis, bowel ischemia and pneumatosis. She is
acidotic and in acute renal failure. Surgical intervention would necessitate
resection of a significant length of small and large bowel. Which of the
following statements about this patient's care is TRUE?
a. She is dying and palliative care is indicated.
b. The decision for further care should be left solely with the family.
c. Operation is the only chance for cure o the bowel ischemia and should be
performed.
d. Aggressive ICU resuscitation is warranted and operation should be
performed if the patient stabilizes.
e. Operation should be performed, but dialysis should not be contemplated.
99.Which of the following denotes the duty of the doctor to intervene on the
patient's behalf to increase comfort, health and well-being?
a. Nonmaleficence.
b. Autonomy.
c. Euthanasia.
d. Beneficence.
e. Justice.
100. Which of the following is NOT considered as a "never event"?
a. Wrong side surgery.
b. Venous thromboembolic disease.
c. Medication errors.
d. Unintentionally retained foreign materials.
e. Wrong patient.
P a g e 25 | 26
SurgiNotes 2nd Ed. (2022)

Egyptian Board Of General Surgery


1st Part Exam – Paper (2) MCQ "ANSWER"
March 2021

1 a 26 a 51 e 76 c
2 e 27 e 52 c 77 d
3 d 28 e 53 c 78 e
4 b 29 a 54 e 79 b
5 c 30 a 55 a 80 b
6 b 31 e 56 c 81 c
7 c 32 d 57 d 82 b
8 a 33 b 58 d 83 d
9 c 34 e 59 a 84 c
10 e 35 e 60 d 85 c
11 c 36 c 61 b 86 a
12 d 37 c 62 e 87 a
13 b 38 b 63 a 88 c
14 e 39 b 64 b 89 a
15 c 40 b 65 e 90 a
16 b 41 b 66 c 91 c
17 b 42 a 67 b 92 b
18 a 43 a 68 d 93 a
19 e 44 d 69 c 94 b
20 a 45 c 70 a 95 c
21 a 46 e 71 a 96 d
22 d 47 b 72 a 97 b
23 c 48 d 73 a 98 b
24 d 49 c 74 a 99 d
25 e 50 d 75 c 100 b

P a g e 26 | 26
SurgiNotes 2nd Ed. (2022)

Egyptian Board Of General Surgery


1st Part Exam – Paper (1) MCQ
September 2021
1. A 34-year-old man is undergoing an emergency appendectomy. After the
appendectomy has been performed successfully, the patient undergoes an
exploratory laparoscopy. Which of the following anatomic features are the MOST
useful to distinguish the jejunum from the ileum?
a. Jejunum has thinner walls compared with the ileum.
b. Jejunum has less mesenteric fat compared with the ileum.
c. Jejunum has more numerous vascular arcades compared with the ileum.
d. Jejunum has more numerous lymphatic follicles beneath the mucosa compared
with the ileum.
e. Jejunum has fewer villi compared with the ileum.
2. A 48-year-old female complains of symptoms of carpal tunnel syndrome for almost
a year. Which muscles MOST typically become weakened in this condition?
a. Dorsal interossei.
b. Lumbricals III and IV.
c. Thenar.
d. Palmar interossei.
e. Hypothenar.
3. A 34-year-old woman had partial thyroidectomy. Twenty-four hours postoperatively
it was noted that the patient frequently aspirates fluid into her lungs. Upon
examination it was determined that the area of the piriform recess above the vocal
fold of the larynx was anesthetized. Which of the following nerves was MOST likely
iatrogenically injured?
a. External branch of the superior pharyngeal.
b. Hypoglossal.
c. Internal branch of the superior laryngeal.
d. Lingual.
e. Recurrent laryngeal.
4. Which of the following is consistently the largest artery to the stomach?
a. Left gastric artery.
b. Right gastric artery.
c. Left gastroepiploic artery.
d. Right gastroepiploic artery.
e. Short gastric vessels.
P a g e 1 | 26
SurgiNotes 2nd Ed. (2022)

5. The principal blood supply to the parathyroid glands is which of the following?
a. Superior thyroid arteries.
b. Inferior thyroid arteries.
c. Thyroid ima arteries.
d. Parathyroid arterial branches directly from the external carotid artery.
e. Vertebral artery.

6. A 73-year-old lady suffers a fracture at the surgical neck of the humerus.


During surgical correction, which of the following vessels lie posterior to the
fracture?
a. Axillary artery.
b. Brachial artery.
c. Thoracoacromial artery.
d. Transverse scapular artery.
e. Posterior circumflex humeral artery.

7. The femoral canal is a point of weakness in the abdominal wall. Which of the
following statements is the MOST accurate regarding the anatomy of the
femoral canal?
a. The femoral artery forms the anterior border of the femoral canal.
b. The femoral vein forms the lateral border.
c. The inguinal ligament lies posterior to the canal.
d. The pectineal part of the inguinal ligament is anterior to the canal.
e. The pectineal ligament forms the medial border.

8. Which of the following is the main blood supply to the breast?


a. Acromio-thoracic artery.
b. Intercostal arteries.
c. Internal thoracic artery.
d. Lateral thoracic artery.
e. Superior thoracic artery.
P a g e 2 | 26
SurgiNotes 2nd Ed. (2022)

9. A 74-year-old woman presents to the outpatient clinic with pelvic pain. A CT


scan reveals enlarged para-aortic lymph nodes. These are MOST likely to be
involved in secondary spread from a tumor in which of the following organs?
a. Cervix.
b. Ovary.
c. Uterus.
d. Vagina.
e. Vulva.
10.A 44-year-old man is admitted to the emergency department with excessive
vomiting and dehydration. Radiographic images demonstrate that part of the
bowel is being compressed between the abdominal aorta and the superior
mesenteric artery. Which of the following intestinal structures is MOST likely
being compressed?
a. Second part of duodenum.
b. Transverse colon.
c. Third part of duodenum.
d. First part of duodenum.
e. Jejunum.
11.A 6-year-old boy is admitted to the hospital with high fever and pain over the
parotid gland. A diagnosis of parotiditis (mumps) is established. Which of the
following nerves is responsible for painful sensations at the region of the
parotid gland?
a. Facial.
b. Auriculotemporal.
c. Lesser petrosal.
d. Lingual.
e. Chorda tympani.
12.A 64-year-old woman is reviewed in the emergency department with an acute
right groin swelling, below and lateral to the pubic tubercle. Which of the
following forms the medial boundary of the ring through which this hernia is
protruding?
a. Conjoint tendon.
b. Inferior epigastric artery.
c. Inguinal ligament.
d. Lacunar ligament.
e. Pectineal ligament.
P a g e 3 | 26
SurgiNotes 2nd Ed. (2022)

P a g e 4 | 26
SurgiNotes 2nd Ed. (2022)

18.Which group of lymph nodes will be inflamed secondary to paronychia of the


big toe?
a. Deep inguinal lymph nodes.
b. External iliac lymph nodes.
c. Lateral group of superficial inguinal lymph nodes.
d. Medial group of superficial inguinal lymph nodes.
e. Vertical group of superficial inguinal lymph nodes.

19.Pain referred to the right side of the neck and extending laterally from the
right clavicle to the tip of the right shoulder is MOST likely due to involvement
of:
a. Cervical cardiac nerves.
b. Posterior vagal trunk.
c. Right intercostal nerves.
d. Right phrenic nerve.
e. Right recurrent laryngeal nerve.

20.Which of the following nerves is MOST likely to be injured following fracture


shaft of the humerus?
a. Medial cord of the brachial plexus.
b. Median nerve.
c. Musculo-cutaneous nerve.
d. Radial nerve.
e. Ulnar nerve.

P a g e 5 | 26
SurgiNotes 2nd Ed. (2022)

21.During an operation for abdominal aortic aneurysm, the inferior mesenteric


artery was ligated, but still the vascularity of the descending colon was
preserved. From which of the following arteries came the blood supply of the
descending colon?
a. Ileocolic artery.
b. Left gastroepiploic artery.
c. Marginal artery.
d. Middle rectal.
e. Superior rectal.

22.A patient who had an injury to the right lateral side of the neck was unable to
raise his right shoulder. What is the injured nerve?
a. Accessory.
b. Dorsal scapular.
c. Greater occipital.
d. Thoracodorsal.
e. Transverse cutaneous nerve of the neck.

23.A 40-year-old male patient had a superficial parotidectomy operation. Post-


operatively he complained of numbness at the lobule of the ear. What was the
injured nerve?
a. Buccal branch of the facial nerve.
b. Cervical branch of the facial nerve.
c. Great auricular nerve.
d. Lesser occipital nerve.
e. Marginal mandibular nerve.

24.A 30-year-old male patient was subjected to a motor car accident and he had
only a fracture of the upper end of the fibula. At examination the patient had
sensory loss at the dorsum of the right foot. Which else may be found on
clinical examination of the patient?
a. Sensory loss at the middle side of the foot.
b. Sensory loss at the sole of the foot.
c. Weakness dorsiflexion of the foot.
d. Weakness in flexion of the knee.
e. Weakness in planter flexion of the foot.
P a g e 6 | 26
SurgiNotes 2nd Ed. (2022)

25.A 20-year-old man presented with a stab injury to the anterior aspect of the
right wrist. On examination he was found to have a deep 2 cm long transverse
laceration at the front of the right wrist with loss of sensation in the ring and
little fingers. The patient was unable to abduct and adduct his fingers on the
affected side. Radial pulse was palpable. What was the MOST likely structure
to have been injured?
a. Anterior interosseous nerve.
b. Median nerve.
c. Musculocutaneous nerve.
d. Radial nerve.
e. Ulnar nerve.

26.A 30-year-old male had a stab wound at the front of the forearm. Examination
revealed loss of flexion of the distal phalanges of the thumb and the lateral 3
fingers. Which of the following was the injured nerve?
a. Ulnar nerve.
b. Posterior interosseous nerve.
c. Anterior interosseous nerve.
d. Radial nerve.
e. Musculocutaneous nerve.

27.A 42-year-old construction worker noted a swelling in the right submandibular


region. Biopsy reveals malignancy and surgical excision is advised. The patient
is informed that one of the risks of this operation is which of the following?
a. Horner syndrome.
b. Excessive sweating in the temporal region.
c. Deformity of the angle of the mouth.
d. Submandibular duct calculus.
e. Trismus.

28.A CT scan of a 63-year-old man reveals that the left renal vein is occluded as it
crosses the aorta. The occlusion is due to compression by an arterial aneurysm
anterior to the vein. What is the MOST likely location of this aneurysm?
a. Coeliac artery.
b. Inferior mesenteric artery.
c. Left colic artery.
d. Middle colic artery.
e. Superior mesenteric artery.
P a g e 7 | 26
SurgiNotes 2nd Ed. (2022)

29.A 24-year-old female received a small-caliber bullet wound to the popliteal


fossa. The surgeons recognized that the bullet had severed the tibial nerve.
Such an injury would MOST likely result in which of the following?
a. Inability to extend the leg at the knee.
b. Foot drop.
c. A dorsiflexed and everted foot.
d. A plantar flexed and inverted foot.
e. Total inability to flex the leg at the knee joint.

30.An 80-year-old man presents with a prostatic malignancy. Imaging reveals a


sclerotic lesion in a thoracic vertebra. Which is the MOST likely vascular route
of metastatic spread?
a. Cremasteric vein.
b. Inferior mesenteric vein.
c. Internal iliac vein.
d. Testicular vein.
e. Vertebral veins.

31.A 25-year-old man is playing football when he complains of posterior thigh


pain whilst sprinting with the ball. On examination he has tenderness in the
lower lateral posterior thigh. He is unable to fully extend the knee due to pain.
What is the MOST likely injury?
a. Biceps femoris tear.
b. Gracilis tear.
c. Gastrocnemius tear.
d. Semimembranosus tear.
e. Semitendinosus tear.

32.A 35-year-old man presents to the emergency department following a sports


injury. He has gross deformity of his arm and X-rays reveal a supracondylar
fracture of the humerus. Examination also reveals some paresthesia over the
thenar eminence. Which nerve has been damaged with the fracture?
a. Axillary.
b. Median.
c. Musculocutaneous.
d. Radial.
e. Ulnar.

P a g e 8 | 26
SurgiNotes 2nd Ed. (2022)

33.A 65-year-old man presents with haematuria and right loin pain.
Computerized tomography demonstrates a right renal tumor and he
undergoes a right radical nephrectomy. The right testicular vein drains into
which of the following?
a. Inferior mesenteric vein.
b. Inferior vena cava.
c. Right adrenal vein.
d. Right lumbar vein.
e. Right renal vein.

34.A 58-year-old male farmer was accidentally injured by a sharp tool. During
physical examination the doctor noted that the patient has foot drop,
although sensation was present over the dorsum of the foot and the skin of
the posterior calf. Which of the following nerve was injured?
a. Femoral nerve.
b. Sciatic nerve.
c. Superficial fibular (peroneal) nerve.
d. Deep fibular (peroneal) nerve.
e. Common fibular (peroneal) nerve.

35.Which of the following cell types is essential for normal wound healing?
a. Erythrocytes.
b. Leukocytes.
c. Lymphocytes.
d. Monocytes.
e. Platelets.

36.Which of the following can lower the serum potassium level?


a. Calcium gluconate.
b. Dextrose 25%.
c. Ringer's lactate.
d. Saline.
e. Sodium bicarbonate.
P a g e 9 | 26
SurgiNotes 2nd Ed. (2022)

37.Which of the following cells is responsible for the secretion of pepsinogen?


a. Chief cells.
b. Enterochromaffin cells.
c. Mucous cells.
d. Paneth cells.
e. Parietal cells.

38.The cervical esophagus receives its blood supply primarily from the:
a. Internal carotid artery.
b. Inferior thyroid artery.
c. Superior thyroid artery.
d. Inferior cervical artery.
e. Facial artery.

39.Which of the following is the MOST potent stimulus for ADH hormone
secretion?
a. Decreased plasma osmolarity.
b. Decreased plasma volume.
c. Hypothalamic releasing factor.
d. Increased plasma osmolarity.
e. Increased plasma volume.

40.Which of the following increases the tone of the lower esophageal sphincter?
a. Gastrin.
b. Secretin.
c. Cholecystokinin.
d. Caffeine.
e. Calcium channel blocker.
P a g e 10 | 26
SurgiNotes 2nd Ed. (2022)

41.A patient who presented with severe diarrhea as a result of clostridium


difficile, develops a metabolic acidosis. This is MOST likely due to loss of which
one of the following ions in the diarrhea?
a. Cl-.
b. HCO3-.
c. K+.
d. NH4+.
e. ОН-.

42.Which of the following is an excitatory transmitter for small bowel motility?


a. Nitric oxide.
b. Vasoactive intestinal peptide.
c. Adenosine triphosphate.
d. Acetylcholine.
e. Catecholamines.

43.Primary malignant tumors of which of the following endocrine glands does


NOT present with hyperfunction?
a. Thyroid.
b. Parathyroid.
c. Endocrine pancreas.
d. Adrenal medulla.
e. Adrenal cortex.

44.A 26-year-old patient presents with abdominal pain, weight loss and
steatorrhea. He is found to have severe terminal ileal involvement and
structuring . Which of the following vitamins is the LEAST likely to be deficient?
a. Vitamin A.
b. Vitamin D.
c. Vitamin E.
d. Vitamin C.
e. Vitamin K.
P a g e 11 | 26
SurgiNotes 2nd Ed. (2022)

45.Which of the following cytokines has anti-inflammatory properties?


a. Granulocyte macrophage colony-stimulating factor.
b. IL-10.
c. IL-2.
d. IL-3.
e. Tumor necrosis factor.

46.Which of the following microscopic features MOST likely indicates that a


neoplasm is malignant?
a. Atypia.
b. Increased nuclear : cytoplasmic ratio.
c. Invasion.
d. Necrosis.
e. Pleomorphism.

47.Which of the following can adverse effects of steroids on wound healing?


a. Copper.
b. Vitamin A.
c. Vitamin C.
d. Vitamin D.
e. Vitamin E.

48.A 45-year-old man with a long history of ulcerative colitis undergoes


colonoscopy. Biopsy shows that the mucosal architecture is abnormal. The
epithelial cells have enlarged hyperchromatic nuclei and there is failure of
maturation towards the surface. There is no evidence of invasion. Which of
the following pathological processes best fits this histological description?
a. Carcinoma.
b. Dysplasia.
c. Hyperplasia.
d. Hypertrophy.
e. Metaplasia.
P a g e 12 | 26
SurgiNotes 2nd Ed. (2022)

49.In the presence of acute inflammation, when an abscess has formed, which of
the following cell types is MOST likely to be primarily involved in phagocytic
activity?
a. Erythrocytes.
b. Lymphocytes.
c. Macrophages.
d. Mast cells.
e. Plasma cells.

50.Which of the following disturbances in associated with tumor lysis syndrome?


a. Hypocalcemia.
b. Hypouricemia.
c. Hypokalemia.
d. Hypomagnesemia.
e. Hypophosphatemia.

51.Which of the following is the MOST effective way in preventing surgical site
infection?
a. Antibiotic prophylaxis.
b. Bowel preparation.
c. Drains and irrigation.
d. Skin preparation.
e. Surgical technique.

52.Which of the following abnormalities indicate the possibility of


hyperaldosteronism?
a. Hyperkalaemia, hyponatremia, hypochloremia.
b. Hypokalaemia, hypernatremia, high pH.
c. Hypokalaemia, hypochloremia, high pH.
d. Hyperkalaemia, hypernatraemia, low pH.
e. Hyperkalemia, hyponatremia, hyperglycemia.
P a g e 13 | 26
SurgiNotes 2nd Ed. (2022)

53.The medial mammary artery supplying the medial aspect of the breast is a
tributary of the:
a. 2nd, 3rd and 4th intercostal arteries.
b. Internal mammary artery.
c. Thoracoacromial artery.
d. Posterior intercostal arteries.
e. Musculo-phrenic artery.

54.Which of the following is TRUE regarding the physiology of the thyroid gland?
a. Absorption of iodides occurs by passive diffusion.
b. Potassium perchlorate prevents the union between iodine and tyrosine.
c. T3 is four times more active than T4.
d. The half-life of thyroxine is 4 days.
e. The necessary intake of iodide is 50 ug/day.

55.Which of the following is the major stimulus for aldosterone secretion?


a. ACTH.
b. Angiotensin II.
c. Cortisol.
d. Hypernatraemia.
e. Hypokalaemia.

56.Which of the following can lower the serum potassium level?


a. Calcium gluconate.
b. Dextrose 25%.
c. Ringer's lactate.
d. Saline.
e. Sodium bicarbonate.
P a g e 14 | 26
SurgiNotes 2nd Ed. (2022)

57.In compensating for respiratory alkalosis, the body excretes more:


a. Ammonium ions.
b. Bicarbonate ions.
c. Dihydrogen phosphate ions.
d. Carbonic acid.
e. Hydrogen ions.

58.Which of the following clinical situations can be associated with hypovolemic


hyponatremia?
a. Congestive heart failure.
b. SIADH (syndrome of inappropriate ADH).
c. Cirrhosis.
d. Hyperglycemia.
e. Gastrointestinal losses.

59.Which of the following statements is TRUE about Cushing's disease?


a. ACTH may be high.
b. Blood sugar is low.
c. It is due to a pituitary adenoma in 10% of cases.
d. Serum Na is low.
e. Surgery is not successful in the treatment.

60.A 40-year-old male patient is complaining of persistent, headache, recurrent


attacks of palpitation, sweating and visual disturbances. Blood pressure of the
patient is 180/110 mmHg. Which of the following is MOST accurate in the
diagnosis of the problem?
a. 24-hour urinary catecholamines.
b. 24-hour urinary VMA.
c. CT scan of the abdomen.
d. Plasma aldosterone level.
e. Plasma metanephrine level.
P a g e 15 | 26
SurgiNotes 2nd Ed. (2022)

61.Which of the following tests will be helpful in the early management of a


patient who had total thyroidectomy?
a. Antithyroglobulin antibody.
b. Serum calcium.
c. Serum iodine.
d. T4.
e. TSH.

62.What is the recommended treatment of acute adrenal insufficiency?


a. Normal saline, potassium and glucose.
b. Hypertonic saline and potassium.
c. Normal saline and potassium.
d. Intravenous mineralocorticoids.
e. Normal saline, glucose and intravenous glucocorticoids.

63.A patient who had adrenalectomy to remove a pheochromocytoma


demonstrates signs of confusion and complains of sweating and headache
several hours following his operation. His blood pressure is 130/65 mmHg, his
heart rate is 100 beats/min and his respiratory rate is 12 breaths/min. What is
the MOST likely cause of his symptoms?
a. Dehydration.
b. Postoperative bleeding.
c. Hypoglycemia.
d. Narcotic overdose.
e. Incomplete removal of the pheochromocytoma.

64.Preventing acute renal failure following crush injury:


a. Maintenance of an alkaline urine (pH > 6) to prevent cast formation.
b. Maintenance of adequate urine output with mannitol.
c. Vigorous intravenous fluid replacement.
d. Administration of xanthine oxidase inhibitors to prevent hyperuricemia.
e. Emergent dialysis to remove myoglobin from the circulation.
P a g e 16 | 26
SurgiNotes 2nd Ed. (2022)

65.Which of the following is the MOST common cause of death in patients with
ovarian cancer?
a. Uremia.
b. Anemia.
c. Liver failure.
d. Bowel obstruction.
e. Respiratory failure.

66.Which of the following occurs in starvation?


a. Increase in plasma glucose.
b. Decrease in urinary nitrogen excretion.
c. Increase in plasma ketone bodies.
d. Increase in glucose utilization by the brain.
e. Metabolic alkalosis.

67.A 56-year-old lady presents with a pathological bone fracture, which of the
following is the likely primary source?
a. Thyroid.
b. Breast.
c. Kidney.
d. Endometrium.
e. Colon.

68.Central venous access lines are a minor procedure, but may associated with
many complications, the commonest is:
a. Haematoma.
b. Air embolism.
c. Pneumothorax.
d. Arrhythmias.
e. Lost wire or catheter.
P a g e 17 | 26
SurgiNotes 2nd Ed. (2022)

69.Which of the following metabolic changes is MOST prominent during times of


physiologic stress?
a. Increase in growth hormone (GH) release.
b. Increase in TSH.
c. Increased levels of T4 and T3.
d. Initial insulin increases and then suppression.
e. Increase in cortisol excretion.

70.Which of the following is the best parameter for monitoring septic shock?
a. Central venous pressure (CVP).
b. Vasopressor requirement.
c. Urine output.
d. Serum lactate.
e. Mental status changes.

71.A 55-year-old male patient is receiving 150 mg of aspirin daily because he has
coronary artery disease. Which of the following tests will be affected in this
patient?
a. Bleeding time.
b. Coagulation time.
c. INR.
d. PTT.
e. Thrombin time.

72.A 40-year-old male patient had a car accident which led to fracture pelvis. The
patient received 4 litres of stored blood following which he had evidence of
generalized coagulopathy. Deficiency of which of the following factors is the
cause of the problem?
a. Fibrinogen.
b. II.
c. V, VIII.
d. XI.
e. XII.
P a g e 18 | 26
SurgiNotes 2nd Ed. (2022)

73.What is the main mode of action of heparin?


a. It increases the level of protein C.
b. It is a cofactor of antithrombin III.
c. It prevents clot retraction.
d. It prevents platelets aggregation.
e. It prevents the synthesis of fibrinogen.

74.What is the principal mechanism of the antithrombotic action of administering


aspirin in low dose?
a. Induction of capillary vasodilatation.
b. Induction of endothelial cell prostacyclin production.
c. Induction of endothelial heparin production.
d. Inhibition of factor V production by the liver.
e. Inhibition of platelet production of thromboxane A2.

75.What does increased level of fibrin degradation products (FDP) in the blood
denote?
a. Diminished fibrinogen synthesis.
b. Diminished platelet count.
c. Diminished platelet function.
d. Disseminated intravascular coagulation (DIC).
e. Heparin overdose.

76.A cirrhotic patient with abnormal haemostatic studies requires an urgent


cholecystectomy. A transfusion of fresh frozen plasma is planned to minimize
the risk of bleeding. What is the optimal timing of this transfusion?
a. In the recovery room.
b. Intraoperatively.
c. On call to surgery.
d. The night before surgery.
e. Two days before surgery.
P a g e 19 | 26
SurgiNotes 2nd Ed. (2022)

77.A 22-year-old man is brought into the emergency department in profound


shock after a fall from the fourth floor of a building. After resuscitation, small
bowel resection and hepatic segmentectomy are performed at laparotomy.
He receives 15 unit of packed RBCs, 4 unit of fresh-frozen plasma and 8 L of
Ringer’s lactate. On closure, diffuse oozing of blood is noted. What is the
MOST likely cause?
a. Hepatic failure.
b. Hypersplenism.
c. Platelet deficiency.
d. Factor IX (Christmas factor) deficiency.
e. Congenital hypoprothrombinemia.

78.Which of the following substances is produced by the vascular endothelium


and inhibits platelets aggregation?
a. Thromboxane A2.
b. Prostacyclin.
c. Nitric oxide.
d. ADP.
e. Serotonin.

79.Which of the following types of shock is associated with high pulmonary


wedge pressure?
a. Hypovolemic shock.
b. Cardiogenic shock.
c. Early septic shock.
d. Late septic shock.
e. Neurogenic shock.

80.What is the mode of action of carbimazole?


a. It diminishes the blood supply of the thyroid gland.
b. It prevents the release of thyroid hormone from the gland.
c. It prevents the union between iodine and tyrosine.
d. It prevents transformation of iodides to iodine.
e. It prevents uptake of iodides by the thyroid gland.
P a g e 20 | 26
SurgiNotes 2nd Ed. (2022)

81.Which of the following is the MOST common sign of Cushing's syndrome?


a. Acne.
b. Hirsutism.
c. Hypertension.
d. Purple striae.
e. Truncal obesity.

82.Which of the following is the first sign or symptom of hypocalcemia?


a. Shortened QT interval.
b. Trousseau sign.
c. Circumoral numbness.
d. Anxiety.
e. Laryngospasm.

83.Which of the following occurs in starvation?


a. Increase in plasma glucose.
b. Decrease in urinary nitrogen excretion.
c. Increase in plasma ketone bodies.
d. Increase in glucose utilization by the brain.
e. Metabolic alkalosis.

84.A 70-year-old patient with diabetes and paraplegia is undergoing an elective


laparoscopic cholecystectomy after an episode of biliary pancreatitis. Shortly
after induction, blood pressure is normal, but ECG shows peaked P waves and
a widened QRS complex. The MOST likely diagnosis is:
a. Ketoacidosis.
b. Hyperkalemia.
c. Hypoglycemia.
d. Hypocalcemia.
e. Acute myocardial infarction.
P a g e 21 | 26
SurgiNotes 2nd Ed. (2022)

85.A 40-years-old dark-skinned male had an excision of a parotid tumor that


resulted in an ugly scar. Which information applies to his condition?
a. Keloids contain an overabundance of fibroblasts.
b. A hypertrophic scar extends beyond the boundaries of the original
wound.
c. Improvement is usually seen with keloid excision followed by intralesional
steroid injection.
d. An incision placed perpendicular to the lines of natural skin tension will
result in the least obvious scar.
e. Hypertrophic scars occur most commonly on the lower extremities.

86.A 78-year-old woman with emphysema on oxygen mask has the following
blood gas results:
PH PO2 PaCO2 Bicarbonate Base excess
Finding: 7.28 70 mmHg 48 mmHg 36 mmol/L +5
What is the MOST likely interpretation?
a. Mixed respiratory and metabolic acidosis.
b. Partially compensated metabolic acidosis.
c. Partially compensated respiratory acidosis.
d. Uncompensated metabolic acidosis.
e. Uncompensated respiratory acidosis.

87.What is the main brain's fuel during prolonged starvation?


a. Amino acid.
b. Glucose.
c. Ketones.
d. Lactose.
e. Short-chain fatty acid.

88.Which of the following hormones is primarily responsible for differentiation


of the breast ductal epithelium?
a. Estrogen.
b. Testosterone.
c. Progesterone.
d. Prolactin.
e. Gonadotrophic hormone.
P a g e 22 | 26
SurgiNotes 2nd Ed. (2022)

89.Marjolin's ulcer arises in areas exposed to:


a. External beam radiation.
b. Thermal injury.
c. Pressure.
d. Lymphedema.
e. Chronic irritation.

90.Shock following severe carbon monoxide poisoning is MOST commonly:


a. Hypovolemic shock.
b. Neurogenic shock.
c. Cardiogenic shock.
d. Vasodilatory shock.
e. Anaphylactic shock.

91.Vasoconstriction is one of the initial physiologic responses to hypovolemic


shock. This is mediated by:
a. Activation of alpha-adrenergic receptors on the arterioles.
b. Down regulation of alpha-adrenergic receptors on the arterioles.
c. Activation of beta-adrenergic receptors on the arterioles.
d. Release of angiotensin II hormone.
e. Down regulation of beta-adrenergic receptors on the arterioles.

92.Bleeding in patients with thrombasthenia is treated with:


a. Whole blood transfusion.
b. Factor V injection.
c. Factor VII injection.
d. Fresh frozen plasma transfusion.
e. Platelet transfusion.
P a g e 23 | 26
SurgiNotes 2nd Ed. (2022)

93.Hemophilia C is caused by a deficiency of:


a. Factor VIII.
b. Factor IX.
c. Factor X.
d. Factor XI.
e. Factor VII.

94.An abnormal aPTT (partial thromboplastin time) is associated with an


abnormality in which portion of the clotting mechanism?
a. Platelet aggregation.
b. Intrinsic pathway.
c. Extrinsic pathway.
d. Both intrinsic and extrinsic pathway.
e. Coagulation (clot formation).

95.The primary lymphatic drainage of the midline of the upper lip is:
a. Submandibular nodes.
b. Submental nodes.
c. Intraparotid nodes.
d. Preauricular nodes.
e. Jugulo-omohyoid nodes.

96.Pancreas divisum occurs as a result of:


a. Pancreatitis.
b. Carcinoma of the pancreas.
c. Penetrating Trauma to the pancreas.
d. Blunt trauma to the pancreas.
e. Abnormal fusion of the pancreatic ducts.
P a g e 24 | 26
SurgiNotes 2nd Ed. (2022)

97.More than 80% of accessory spleens are found in the splenic hilum. What is
the second MOST common location for an accessory spleen?
a. The Lienorenal ligament.
b. The greater omentum.
c. The gastrocolic ligament.
d. The tail of the pancreas.
e. The splenocolic ligament.

98.Which of the following is the primary function of the spleen in human adults?
a. Production of red cells.
b. Production of white cells.
c. Destruction of platelets.
d. Storage of blood.
e. Host defense.

99.The inguinal ligament is a part of which abdominal wall muscle?


a. Transversalis.
b. Internal oblique.
c. External oblique.
d. Rectus abdominis.
e. Pyramidalis.

100. The MOST common position of the right recurrent laryngeal nerve is:
a. Anterior to the inferior thyroid artery.
b. Posterior to the inferior thyroid artery.
c. Between the branches of the inferior thyroid artery.
d. Transversely parallel to inferior thyroid artery.
e. Absent (nonrecurrent) laryngeal nerve.
P a g e 25 | 26
SurgiNotes 2nd Ed. (2022)

Egyptian Board Of General Surgery


1st Part Exam – Paper (1) MCQ "ANSWER"
September 2021

1 b 26 c 51 a 76 c
2 c 27 c 52 b 77 c
3 c 28 e 53 a 78 b
4 a 29 c 54 c 79 b
5 b 30 e 55 b 80 c
6 e 31 a 56 e 81 e
7 b 32 b 57 b 82 c
8 c 33 b 58 e 83 c
9 b 34 d 59 a 84 b
10 c 35 d 60 e 85 c
11 b 36 e 61 b 86 c
12 d 37 a 62 e 87 c
13 38 b 63 c 88 c
14 39 d 64 c 89 b
15 40 a 65 d 90 d
16 41 b 66 c 91 a
17 42 d 67 b 92 e
18 a 43 c 68 a 93 d
19 d 44 d 69 e 94 b
20 d 45 b 70 d 95 d
21 c 46 c 71 a 96 e
22 a 47 b 72 c 97 c
23 c 48 b 73 b 98 e
24 c 49 c 74 e 99 c
25 e 50 a 75 d 100 b

P a g e 26 | 26
SurgiNotes 2nd Ed. (2022)

Egyptian Board Of General Surgery


1st Part Exam – Paper (1) MCQ
March 2022
1. A neurosurgeon started his brain tumor operation by a scalp incision. Which
of the following does NOT constitute part of the scalp layers?
a. Skin.
b. Subcutaneous fat.
c. Dense connective tissue.
d. Galea aponeurotica.
e. Loose areolar tissue.

2. A 21-year-old man developed "black eye" after being subjected to head


trauma. Where has this hemorrhage MOST likely occurred?
a. Just under the skin of the scalp.
b. Under the epicranial aponeurosis.
c. Under the pericranium.
d. Within the cranial cavity.
e. Within a fractured nasal sinus.

3. The superficial layer of cervical fascia splits into two sheets to enclose which
of the following muscles?
a. Sternothyroid.
b. Anterior scalene.
c. Trapezius.
d. Mylohyoid.
e. Semispinalis capitis.

4. The innervation of the masseter muscle is provided by which of the following


nerves?
a. Mandibular division of the trigeminal.
b. Buccal branch of the facial.
c. Buccal branch of the trigeminal.
d. Maxillary division of the trigeminal.
e. Inferior alveolar.
P a g e 1 | 26
SurgiNotes 2nd Ed. (2022)

5. The inferior thyroid artery is a branch of which of the following arteries?


a. Dorsal scapular.
b. Costocervical.
c. External carotid.
d. Thyrocervical.
e. Vertebral.

6. The lateral to medial (superficial to deep) relationship of facial nerve, external


carotid artery and retromandibular vein, as they pass through parotid gland,
is:
a. Nerve, artery, vein.
b. Nerve, vein, artery.
a. Vein, artery, nerve.
c. Vein, nerve, artery.
d. Artery, nerve, vein.

7. Which of the following structures lies just behind the carotid sheath?
a. Vagus nerve.
b. Superior root of ansa cervicalis.
c. Inferior root of ansa cervicalis.
d. Sympathetic trunk.
e. Phrenic nerve.

8. Enlarging the femoral ring medially to relieve strangulation of femoral hernia


is sometimes met with serious hemorrhage due to rupture of:
a. Femoral vein.
b. Femoral artery.
c. Accessory obturator artery.
d. Obturator artery.
e. Inferior epigastric vein.
P a g e 2 | 26
SurgiNotes 2nd Ed. (2022)

9. Which of the following muscles is attached to the lesser trochanter of the


femur?
a. Pectineus.
b. Iliopsoas.
c. Adductor longus.
d. Adductor brevis.
e. Sartorius.

10.Which of the following is NOT a branch of the arch of aorta?


a. Left common carotid.
b. Right common carotid.
c. Brachiocephalic trunk.
d. Left subclavian.
e. Thyroidea ima.

11.The trachea is separated from the bodies of the upper four thoracic vertebrae
by the:
a. Aorta.
b. Esophagus.
c. Azygos vein.
d. Thoracic duct.
e. Heart.

12.Which statement is FALSE regarding the pancreas?


a. It develops from ventral and dorsal buds.
b. Is a retroperitoneal organ.
c. Is supplied by the branches of the coeliac trunk only.
d. Gives attachment to the transverse mesocolon.
e. Lies anterior to the splenic vein.
P a g e 3 | 26
SurgiNotes 2nd Ed. (2022)

13.The accessory pancreatic duct:


a. Drains the body and tail of the pancreas.
b. Opens into the minor duodenal papilla.
c. Opens in common with the bile duct.
d. Is surrounded by the sphincter of Oddi.
e. Has none of the above properties.

14.The pancreatica magna artery is a branch of which of the following arteries?


a. Common hepatic.
b. Inferior mesenteric.
c. Superior mesenteric.
d. Left gastroepiploic.
e. Splenic.

15.A 2-day-old newborn male is cyanotic after attempts to swallow milk result in
collection of the milk in his mouth. After 2 days he develops pneumonia. A
tracheoesophageal fistula is suspected. Which of the following structures has
failed to develop properly?
a. Esophagus.
b. Trachea.
c. Tongue.
d. Tracheoesophageal septum.
e. Pharynx.

16.The lesser peritoneal sac communicates with the greater peritoneal sac
through the:
a. Aortic hiatus.
b. Esophageal hiatus.
c. Caval foramen.
d. Deep inguinal ring.
e. Epiploic foramen.
P a g e 4 | 26
SurgiNotes 2nd Ed. (2022)

17.Which structure is NOT related to the right kidney posteriorly?


a. Diaphragm.
b. 11th rib.
c. Transversus abdominis muscle.
d. Psoas major muscle.
e. Iliohypogastric nerve.

18.The structures in the hilum of kidney are arranged from anterior to posterior
side as:
a. Vein, ureter, artery.
b. Artery, vein, ureter.
c. Vein, artery, ureter.
d. Ureter, artery, vein.
e. Artery, ureter, vein.

19.The right suprarenal vein drains into which of the following veins?
a. Right renal.
b. Inferior mesenteric.
c. Superior mesenteric.
d. Portal.
e. Inferior vena cava.

20.The medial arcuate ligament of the diaphragm bridges over:


a. Right crus of diaphragm.
b. Quadratus lumborum.
c. Psoas major.
d. Pectineus.
e. Erector spinae.
P a g e 5 | 26
SurgiNotes 2nd Ed. (2022)

21.Which of the following statements is FALSE regarding the oesophageal


opening of the diaphragm?
a. It lies at the level of 12th thoracic vertebra.
b. It is situated through the right crus.
c. The 2 vagi pass through this opening.
d. It transmits branches of left gastric artery and vein.
e. Lies exactly in the midline.

22.A 4-year-old male child is admitted to the hospital with severe vomiting.
Radiographic examination and history taking reveals that the boy suffers from
an annular pancreas. Which of the following structures is MOST typically
obstructed by this condition?
a. Pylorus of the stomach.
b. First part of the duodenum.
c. Second part of the duodenum.
d. Third part of the duodenum.
e. Jejunum.

23.Which of the following arteries is NOT a direct branch from the abdominal
aorta?
a. Superior mesenteric.
b. Renal.
c. Inferior phrenic.
d. Inferior suprarenal.
e. Gonadal.

24.The gastroduodenal artery is a branch of the:


a. Coeliac trunk.
b. Pancreaticoduodenal.
c. Common hepatic.
d. Superior mesenteric.
e. Right gastric.
P a g e 6 | 26
SurgiNotes 2nd Ed. (2022)

25.A 45-year-old male construction worker slips and falls on a nail protruding
from a board. The nail penetrates the skin overlying the submental triangle
lateral to the midline. Which of the following muscles would be the last to be
penetrated?
a. Platysma.
b. Mylohyoid.
c. Anterior belly of the digastric.
d. Geniohyoid.
e. Genioglossus.

26.Which of the following features does NOT apply to the right renal artery?
a. It passes behind the inferior vena cava.
b. It lies posterior to right renal vein.
c. It lies in front of pelvis of ureter.
d. It is shorter than left renal artery.
e. It gives out the inferior adrenal artery.

27.Which of the following muscles does NOT form any boundary of the axilla?
a. Serratus anterior.
b. Pectoralis major.
c. Subclavius.
d. Subscapularis.
e. Deltoid.

28.Which statement is FALSE regarding the axillary tail of the breast?


a. Lies in superficial fascia.
b. Lies close to axillary vessels.
c. Pierces the deep fascia.
d. It arises from supero-lateral quadrant of mammary gland.
e. Contains lactiferous glands.
P a g e 7 | 26
SurgiNotes 2nd Ed. (2022)

29.A 5-day-old male infant is diagnosed with Hirschsprung disease. CT scan


examination reveals an abnormally dilated colon. Which of the following is the
MOST likely embryologic mechanism responsible for Hirschsprung disease?
a. Failure of neural crest cells to migrate into the walls of the colon.
b. Incomplete separation of the cloaca.
c. Failure of recanalization of the colon.
d. Defective rotation of the hindgut.
e. Oligohydramnios.
30.A 38-year-old pregnant woman is admitted to the emergency department
with severe vaginal bleeding. Ultrasound examination confirms the initial
diagnosis of ectopic pregnancy. Which of the following is the MOST common
site of an ectopic pregnancy?
a. Uterine tubes.
b. Cervix.
c. Mesentery of the abdominal wall.
d. Lower part of uterine body overlapping the internal cervical os.
e. Fundus of the uterus.
31.A 32-year-old male is admitted to the emergency department with groin pain.
Examination reveals that the patient has an indirect inguinal hernia. Which of
the following nerves is compressed by the herniating structure in the inguinal
canal to give the patient pain?
a. Iliohypogastric.
b. Lateral femoral cutaneous.
c. Ilioinguinal.
d. Subcostal.
e. Pudendal.
32.A 54-year-old male is admitted to the emergency department with severe
upper abdominal pain. Gastroscopy reveals a tumor in the antrum of the
stomach. A CT scan is ordered to evaluate lymphatic drainage of the Stomach.
Which of the following lymph nodes is MOST likely to be involved in a
malignancy of the stomach?
a. Celiac.
b. Superior mesenteric.
c. Inferior mesenteric.
d. Lumbar.
e. Hepatic.
P a g e 8 | 26
SurgiNotes 2nd Ed. (2022)

33. A 58-year-old male alcoholic is admitted to the hospital after vomiting dark red
blood (hematemesis). Endoscopy reveals ruptured esophageal varices resulting
from portal hypertension. Which of the following venous tributaries to the portal
system anastomosis with caval veins to cause the varices?
a. Splenic.
b. Left gastro-epiploic.
c. Left gastric.
d. Left hepatic.
e. Right gastric.

34. A 35-year-old male is admitted to the hospital with an indirect inguinal hernia.
During an open hernioplasty (in contrast to a laparoscopic procedure), the spermatic
cord and the internal abdominal oblique muscles are identified. Which component
of the spermatic cord is derived from the internal abdominal oblique muscle?
a. External spermatic fascia.
b. Cremaster muscle.
c. Tunica vaginalis.
d. Internal spermatic fascia.
e. Dartos fascia.

35. A 34-year-old man is undergoing an emergency appendectomy. After the


appendectomy has been performed successfully, the patient undergoes an
exploratory laparoscopy. Which of the following anatomic features are the MOST
useful to distinguish the jejunum from the ileum?
a. Jejunum has thinner walls compared with the ileum.
b. Jejunum has less mesenteric fat compared with the ileum.
c. Jejunum has more numerous vascular arcades compared with the ileum.
d. Jejunum has more numerous lymphatic follicles beneath the mucosa compared
with the ileum.
e. Jejunum has fewer villi compared with the ileum.

36. After a mastectomy, a musculo-cutaneous flap is used to restore the thoracic


contour in a 34-year-old female patient. The ipsilateral (same side) rectus abdominis
muscle was detached carefully from the surrounding structures and transposed to
the thoracic wall. Which of the following landmarks is MOST often used to locate
the inferior end of the posterior tendinous layer of the rectus sheath?
a. Intercristal line.
b. Linea alba.
c. Arcuate line.
d. Pectineal line.
e. Semilunar line.
P a g e 9 | 26
SurgiNotes 2nd Ed. (2022)

37. A 45-year-old female is admitted to the hospital with symptoms of an upper bowel
obstruction. Upon CT examination it is found that the third (transverse) portion of
the duodenum is being compressed by a large vessel. Which of the following vessels
will MOST likely be causing the compression?
a. Inferior mesenteric artery.
b. Superior mesenteric artery.
c. Inferior mesenteric vein.
d. Portal vein.
e. Splenic vein.

38. A 36-year-old man was brought to the emergency department with a bullet wound
to the abdomen. The bullet penetrated the anterior abdominal wall superior to the
umbilicus. If the bullet passed directly posterior in the midline, which of the
following structures was MOST likely to have been struck first by the bullet?
a. Abdominal aorta.
b. Transverse colon.
c. Stomach.
d. Gallbladder.
e. Pancreas.

39. A 23-year-old man is admitted to the hospital with a bulge in his scrotum. Physical
examination reveals an indirect inguinal hernia. During the open hernia repair the
internal spermatic fascia is identified and reflected to expose the ductus deferens
and testicular vessels. Which of the following provides the internal spermatic fascial
layer of the spermatic cord?
a. External abdominal oblique aponeurosis.
b. Internal abdominal oblique aponeurosis.
c. Transversus abdominis aponeurosis.
d. Transversalis fascia.
e. Processus vaginalis.

40. Two days after an appendectomy on a 45-year-old male patient, the patient has
developed an elevated temperature (39 °C), is hypotensive and complains of
abdominal pain. An exploratory laparotomy reveals large amounts of blood in the
peritoneal cavity due to an injury to a vessel that occurred during the
appendectomy. Which of the following vessels must be ligated to stop the bleeding?
a. Right colic artery.
b. Right colic artery and superior rectal artery.
c. Superior mesenteric artery.
d. Ileocolic artery.
e. Ileocolic artery and middle colic artery.
P a g e 10 | 26
SurgiNotes 2nd Ed. (2022)

41.A 45-year-old woman is being examined as a candidate for cosmetic breast


surgery. The surgeon notes that both of her breasts sag considerably. Which
structure has MOST likely become stretched to result in this condition?
a. Scarpa's fascia.
b. Pectoralis major muscle.
c. Pectoralis minor muscle.
d. Suspensory (Cooper's) ligaments.
e. Serratus anterior muscle.
42.A 27-year-old man was admitted to the emergency department after an
automobile collision in which he suffered a fracture of the lateral border of
the scapula. Six weeks after the accident, physical examination reveals
weakness in medial rotation and adduction of the humerus. Which nerve was
MOST likely injured?
a. Lower subscapular.
b. Axillary.
c. Radial.
d. Spinal accessory.
e. Ulnar.
43.While walking to his classroom building, a first-year medical student slipped
on the wet floor and fell against the desk, injuring his right arm. Radiographic
images showed a midshaft fracture of the humerus. Which pair of structures
was MOST likely injured at the fracture site?
a. Median nerve and brachial artery.
b. Axillary nerve and posterior humeral circumflex artery.
c. Radial nerve and deep brachial artery.
d. Median and ulnar nerves.
e. Radial nerve and radial artery.
44.A 36-year-old man is brought to the emergency department because of a deep
knife wound on the medial side of his distal forearm. He is unable to hold a
piece of paper between his fingers and has sensory loss on the medial side of
his hand and little finger. Which nerve is MOST likely injured?
a. Axillary.
b. Median.
c. Musculocutaneous.
d. Radial.
e. Ulnar.
P a g e 11 | 26
SurgiNotes 2nd Ed. (2022)

45.A 60-year-old male butcher accidentally slashed his wrist with his butcher's
knife, partially dividing the ulnar nerve. Which of the following actions would
MOST likely be lost as a result of this injury?
a. Flexion of the proximal inter-phalangeal joint of the little finger.
b. Extension of the thumb.
c. Adduction of the fifth digit.
d. Abduction of the thumb.
e. Opposition of the thumb.
46.A 45-year-old male is treated at the hospital after he fell from his bicycle.
Radiographic examination reveals fractures both of the tibia and the fibula.
On physical examination the patient has a foot drop, but normal eversion.
Which of the following nerves is MOST likely injured?
a. Tibial.
b. Common fibular (peroneal).
c. Superficial fibular (peroneal).
d. Saphenous.
e. Deep fibular (peroneal).
47.A 49-year-old lady is admitted to the emergency department with a painful
lump on the proximal medial aspect of his thigh. Radiographic and physical
examinations reveal that the patient has a herniation of abdominal viscera
beneath the inguinal ligament into the thigh. Through which of the following
openings will a hernia of this type initially pass to extend from the abdomen
into the thigh?
a. Femoral ring.
b. Superficial inguinal ring.
c. Deep inguinal ring.
d. Fossa ovalis.
e. Obturator canal.
48.A 75-year-old woman is admitted to the hospital after falling in her bathroom.
Radiographic examination reveals an extracapsular fracture of the femoral
neck. Which artery is MOST likely at risk for injury?
a. Inferior gluteal.
b. First perforating branch of deep femoral.
c. Medial circumflex femoral.
d. Obturator.
e. Superior gluteal.
P a g e 12 | 26
SurgiNotes 2nd Ed. (2022)

49.The followings are contents of the carpal tunnel EXCEPT:


a. Ulnar nerve.
b. Median nerve.
c. Flexor pollicis longus.
d. Flexor carpi radialis.
e. Flexor digitorum profundus.

50.A 35-year-old woman is admitted to the emergency department after a violent


automobile crash. The patient's upper airway is obstructed with blood and
mucus and a midline tracheotomy inferior to the thyroid isthmus is
performed. Which of the following vessels are MOST likely to be present at
the site of incision and will need to be cauterized?
a. Middle thyroid vein and inferior thyroid artery.
b. Inferior thyroid artery and inferior thyroid vein.
c. Inferior thyroid vein and thyroidea ima artery.
d. Cricothyroid artery and inferior thyroid vein.
e. Left brachiocephalic vein and inferior thyroid artery.

51.Which of the following is the best definition of physiology?


a. The microscopic study of tissues and cells.
b. The study of how the body functions.
c. All the chemical processes that take place in the organelles of the body's
cells.
d. The body's automatic tendency to maintain a relatively constant internal
environment.
e. The biochemical effects on organ functions.

52.Which structure within the cell produces ATP (adenosine triphosphate)?


a. The nucleus.
b. Peripheral proteins.
c. The endoplasmic reticulum.
d. The ribosomes.
e. The mitochondria.
P a g e 13 | 26
SurgiNotes 2nd Ed. (2022)

53.Which form of transport through the plasma membrane requires the


expenditure of energy by the cell?
a. Diffusion.
b. Facilitated diffusion.
c. Osmosis.
d. Pump.
e. Imbibition.

54.What is the composition of the intercellular matrix in connective tissue?


a. Cells and fibres.
b. Serous and mucus membranes and lamina propria.
c. Protein fibres and ground substance.
d. Interstitial fluid.
e. Serous fluid.

55.Which of the following is NOT a cell found in connective tissue?


a. Adipocytes.
b. Chondroblasts.
c. Osteoblasts.
d. Fibroblasts.
e. Keratinocytes.

56.What is the function of phospholipids in the plasma membrane?


a. To maintain the intracellular fluid at a similar composition to that of the
interstitial fluid.
b. To form channels to selectively allow passage of small molecules.
c. To act as receptors for signaling chemicals.
d. To present a barrier to the passage of water-soluble molecules.
e. To allow for cell replication.
P a g e 14 | 26
SurgiNotes 2nd Ed. (2022)

57.Which of the listed terms is described by "All the chemical processes that take
place in the organelles and cytoplasm of the cells of the body"?
a. Metabolism.
b. Cellular respiration.
c. Homeostasis.
d. Physiology.
e. Milieu.

58.Many drugs are neutralized to form salts and administered in this form. What
is the main advantage of administering the salt form of the drug? It is usually:
a. Less toxic.
b. More soluble in water.
c. More pleasant to taste.
d. More soluble in lipid.
e. Potentiates the drug effect.

59.The chief determinant of plasma osmolality is:


a. Plasma Na+.
b. Plasma K+.
c. Plasma glucose.
d. Blood urea.
e. Plasma albumin.

60.What is the MOST reliable RBCs index?


a. MCV.
b. MCH.
c. MCHC.
d. Mean cell diameter.
e. Hematocrit.
P a g e 15 | 26
SurgiNotes 2nd Ed. (2022)

61.Which of the following clotting factors is NOT Vit K dependent?


a. Factor II.
b. Factor V.
c. Factor VII.
d. Factor IX.
e. Factor X.

62.An abnormal aPTT (partial thromboplastin time) is associated with an


abnormality in which portion of the clotting mechanism?
a. Platelet aggregation.
b. Intrinsic pathway.
c. Extrinsic pathway.
d. Both intrinsic and extrinsic pathway.
e. Coagulation (clot formation).

63.Hemophilia C is caused by a deficiency of:


a. Factor VIII.
b. Factor IX.
c. Factor X.
d. Factor XI.
e. Factor VII.

64.A 70-year-old man arrived to the emergency department in a critically ill


condition. His arterial blood gasses report showed the following result: PO2 84
mmHg, pH 7.37, PCO2 28 mmHg and bicarb. 32 mEq/L. What is the best
description for his acid base status?
a. Respiratory acidosis.
b. Respiratory alkalosis.
c. Metabolic acidosis.
d. Metabolic alkalosis.
e. Normal acid base status.
P a g e 16 | 26
SurgiNotes 2nd Ed. (2022)

65.Which mutation of the following does NOT result in a hypercoagulable state?


a. Protein C.
b. Protein S.
c. Factor V.
d. Antithrombin III.
e. Fibrinogen.

66.In haemostasis, which molecule polymerizes to become the insoluble blood


clot?
a. Factor X.
b. Thrombin.
c. Fibrin.
d. Plasmin.
e. Thromboxane.

67.Which is the MOST abundant plasma protein?


a. Albumin.
b. Alpha globulin.
c. Beta globulin.
d. Ferritin.
e. Haemoglobin.

68.What is the average half-life of neutrophils in the circulation?


a. 6 hours.
b. 3 days.
c. 7 days.
d. 2 weeks.
e. 1 month.
P a g e 17 | 26
SurgiNotes 2nd Ed. (2022)

69.Where is MOST of the iron in the body present?


a. Haemoglobin.
b. Myoglobin.
c. Ferritin.
d. Transferrin.
e. Kupffer cells.

70.Which characteristic of blood refers to the concentration of solutes?


a. Salinity.
b. PH.
c. Osmolality.
d. Viscosity.
e. Tonicity.

71.Which molecule or ion dissolved in blood can stimulate the central


chemoreceptors of the brain's respiratory centre?
a. CO2.
b. H+.
c. HCO3-.
d. Lactate.
e. Pyruvate.

72.The major adrenal androgen is:


a. Etiocholanolone.
b. Testosterone.
c. Dihydrotestosterone.
d. Dehydroepiandrosterone (DHEA) sulphate.
e. Androstenedione.
P a g e 18 | 26
SurgiNotes 2nd Ed. (2022)

73.Which of the following hormones are NOT energy releasing?


a. Growth hormone.
b. Cortisol.
c. Epinephrin.
d. Insulin.
e. Glucagon.

74.Insulin secretion from β cells of the pancreas is inhibited by:


a. Acetylcholine.
b. Activation of beta-adrenergic receptors.
c. Thyroxin.
d. Glucagon.
e. Somatostatin.

75.Which of the following inhibits gastric acid secretion by parietal cells in the
stomach?
a. Prostaglandin E2.
b. Gastrin.
c. Acetylcholine.
d. Histamine.
e. Secretin.

76.What is the MOST important enzyme for digestion of fat?


a. Salivary lipase.
b. Gastric lipase.
c. Pancreatic lipase.
d. Intestinal lipase.
e. Lipase in bile.
P a g e 19 | 26
SurgiNotes 2nd Ed. (2022)

77.The major humoral mediator of pancreatic secretion of bicarbonate rich juice


is:
a. Secretin.
b. Cholecystokinin.
c. Somatostatin.
d. Gastrin.
e. Glucagon.

78.Normally there is increased secretion than absorption of which ion in the


colon?
a. Sodium.
b. Potassium.
c. Calcium.
d. Magnesium.
e. Chloride.

79.Twenty-four of the chemical elements are essential to the human body. Four
bulk elements, 7 are macrominerals and 13 are trace elements. Which are the
four bulk elements?
a. Calcium, carbon, hydrogen & oxygen.
b. Nitrogen, carbon, hydrogen & oxygen.
c. Calcium, nitrogen, carbon & oxygen.
d. Carbon, oxygen, phosphorus & iron.
e. Calcium, nitrogen, carbon & hydrogen.

80.The term "capacitance vessels" is applied to:


a. Pulmonary capillaries.
b. Arterial tree.
c. Veins and venules.
d. Shunts.
e. Thoroughfare channels.
P a g e 20 | 26
SurgiNotes 2nd Ed. (2022)

81.Why do capillaries can withstand high internal pressures without bursting?


a. Because they lack smooth muscles.
b. Because the blood flow through them is slow.
c. Because the haematocrit in them is lower than in arteries and veins.
d. Because they have small diameter.
e. Because they are supported by surrounding connective tissue.

82.During muscle cell contraction, what happens because of Ca++ binding to


troponin?
a. The binding site on actin is uncovered.
b. Acetylcholine is released.
c. The cross-bridge disengages from the thin filament.
d. ATP hydrolyses to ADP.
e. Sodium potassium pump is activated.

83.What is the role of gastrin in the digestive system?


a. To stimulate release of bile and pancreatic juice.
b. To stimulate acid secretion.
c. Activate pepsinogen.
d. Hydrolyse proteins to polypeptides.
e. Stimulate gastric contractions.

84.The secretion of gastrin from G cells is inhibited by:


a. Vagal stimulation of the stomach.
b. Circulating epinephrine.
c. Blood borne calcium.
d. Mechanical distension of the stomach.
e. Somatostatin.
P a g e 21 | 26
SurgiNotes 2nd Ed. (2022)

85.What is the purpose of "intrinsic factor" in gastric juice?


a. To promote gastric emptying.
b. To assist with the absorption of vitamin B12.
c. To protect the stomach lining against hydrochloric acid.
d. To allow absorption of iron.
e. To promote reabsorption of bile salts.

86.Why are the blood capillaries in the liver lobules so permeable?


a. To allow the products of digestion to leave the blood for processing in the
liver.
b. To allow fatty acids to leave the liver cells to enter the blood.
c. To allow plasma proteins that are synthesised in the liver to enter the
blood.
d. To allow red blood cells at the end of their life to leave the blood to be
recycled in the liver.
e. To allow detoxication of toxic molecules.

87.What is the function of bile salts?


a. To assist the digestion of lipids.
b. To assist the absorption of digested lipids.
c. To hydrolyse lipids.
d. To emulsify lipids.
e. To protect the mucosa from fatty acids.

88.Which of the functions below is NOT performed by the liver?


a. Production of glucagon.
b. Synthesis of lipoproteins to transport fatty acids.
c. Deamination of amino acids to form keto-acids.
d. Conversion of non-carbohydrate molecules to glucose.
e. Production of ammonia.
P a g e 22 | 26
SurgiNotes 2nd Ed. (2022)

89.Fatty acids are transported around the body by the blood in structures known
as:
a. Micelles.
b. Chylomicrons.
c. Triglycerols.
d. Low density lipoproteins.
e. Microlipids.

90.In the digestion of food molecules, the process known as "hydrolysis" is


described as which of the following?
a. Breaking one or more chemical bonds using a molecule of water.
b. Splitting a molecule into two smaller molecules using hydrogen.
c. The metabolism of glucose to produce water and energy.
d. Splitting of triglycerols into molecules that are soluble in water.
e. Combining a small molecule to a water molecule to facilitate its
absorption.

91.Which of the following is a function of the normal flora of the large intestine?
a. To hydrolyse cellulose.
b. To produce peristaltic stimulants.
c. To synthesise B vitamins and vitamin K.
d. To secrete protective mucosal substances.
e. To add bulk to stools.

92.Which gland or organ releases erythropoietin?


a. The kidneys.
b. The adrenal glands.
c. The anterior pituitary.
d. The pancreas.
e. The spleen.
P a g e 23 | 26
SurgiNotes 2nd Ed. (2022)

93.What effect does aldosterone have?


a. It promotes calcium excretion by the kidney.
b. It promotes Na+ excretion by the kidney.
c. It promotes hydrogen ions to be excreted in urine.
d. It promotes K+ excretion by the kidney.
e. It promotes K+ re-absorption from the kidney filtrate.

94.What is the normal urine output per hour?


a. 10-20 ml.
b. 20-30 ml.
c. 30-50 ml.
d. 50-70 ml.
e. 70-100 ml.

95.Which part of the nephron is impermeable to water?


a. Proximal convoluted tubule.
b. Proximal convoluted tubule in the presence of ADH.
c. Distal convoluted tubule in the presence of ADH.
d. Ascending limb of the loop of Henle.
e. Descending limb of the loop of Henle.

96.Which material is actively reabsorbed from the filtrate in the kidney tubule?
a. Na+.
b. HCO3-.
c. Cl-.
d. H₂O.
e. Albumin.
P a g e 24 | 26
SurgiNotes 2nd Ed. (2022)

97.Which of the following mechanisms operates in the nephron to maintain pH


balance in the body?
a. Bicarbonate ions from the tubule cells are secreted into the filtrate, then
H+ + HCO3- → H₂CO3 → CO₂ + H₂O.
b. In tubule cells CO₂ + H₂O → H₂CO3 → H+ + HCO3- then hydrogen ions are
secreted into the filtrate.
c. In tubule cells CO₂ + H₂O → H₂CO3 → H+ + HCO3- then hydrogen ions are
transported into the peritubular capillaries.
d. In tubule cells CO₂ + H₂O → H₂CO3 → H+ + HCO3- then bicarbonate ions are
secreted into the filtrate.
e. H+ is excreted directly from the blood.

98.In which part of the nephron does MOST of the reabsorption of water and
solutes occur?
a. The collecting ducts.
b. The ascending loop of Henle.
c. The descending loop of Henle.
d. The vasa recta.
e. The proximal convoluted tubule.

99.What is the first process that occurs after a blood vessel is damaged?
a. Activation of the intrinsic mechanisms.
b. Activation of extrinsic mechanisms.
c. Formation of platelet plug.
d. Vasoconstriction.
e. Widening of the endothelial junctions.

100. The primary function of the spleen in adults is:


a. Production of RBCs.
b. Production of white cells.
c. Storage of blood.
d. Host defense.
e. Detoxication of blood.
P a g e 25 | 26
SurgiNotes 2nd Ed. (2022)

Egyptian Board Of General Surgery


1st Part Exam – Paper (1) MCQ "ANSWER"
March 2022

1 b 26 d 51 b 76 c
2 d 27 e 52 e 77 a
3 c 28 b 53 d 78 b
4 a 29 a 54 c 79 b
5 d 30 a 55 e 80 c
6 b 31 c 56 d 81 b
7 d 32 a 57 a 82 a
8 d 33 c 58 b 83 b
9 b 34 b 59 a 84 e
10 b 35 b 60 c 85 b
11 b 36 c 61 b 86 c
12 c 37 b 62 b 87 d
13 b 38 b 63 d 88 a
14 e 39 d 64 c 89 b
15 d 40 d 65 e 90 a
16 e 41 d 66 c 91 c
17 c 42 a 67 a 92 a
18 c 43 c 68 a 93 d
19 e 44 e 69 a 94 d
20 c 45 c 70 c 95 d
21 a 46 e 71 a 96 a
22 c 47 a 72 c 97 b
23 d 48 c 73 e 98 e
24 c 49 a 74 e 99 d
25 e 50 c 75 e 100 d

P a g e 26 | 26

You might also like